You are on page 1of 152

MISMO REVIEWER COMPETENCE 1

1. Which of the following stands for SAR?


● System and receiver
● Search and Rescue
● Services and Rescue
● Search and Response

2. What is the best way to test the Inmarsat-C terminal?


● Send a message to a shore terminal and wait for confirmation
● If the “send’ light flashes, proper operation has been confirmed.
● Compose and send a brief message to your Inmarsat C terminal
● Send a message to another ship terminal

3. How many HF DSC Distress watch channels must be guarded by a compulsory vessel underway?
● Two (2)
● Four (4)
● Three (3)
● Six (6)

4. Reference SOLAS IV, Regulation 2, what is meant by an area within the radiotelephone coverage of at
least on VHF coastal station in which continuous DSC alerting is available?
● Sea Area A1
● Sea Area A3
● Sea Area A2
● Sea Area A4

5. What is FALSE about HF communication?


● Not affected by time of day
● Shortwave propagation
● Skywave propagation
● It bounces back to Earth

6. Which of the following numbers indicates a ship station MMSI?


● 030356328
● 003372694
● 623944326
● 3384672

7. Radio laws and regulation of the Philippines are enforced by the?


● Department of Labor and Employment
● Department of Education
● Commission on Higher Education and Culture
● National Telecommunication Commission

8. When experiencing a shadowing of the satellite signal by a shipboard obstruction. How would you show
your immediate remedy?
● To relocate the mast or other obstruction
● To alter the ship’s course
● To raise the transmit power level
● To increase the receiver gain.

9. What does a DSC control unit do if the GMDSS radio operator fails to insert updated information when
initiating a DSC distress alert?
● It will transmit the nature of distress
● It will initiate the DSC distress alert and default information will automatically transmitted
● It will not automatically transmit the time of transmission
● It will not automatically transmit the present position of the vessel

10. For a DSC distress call, the distress information is contained in four messages. What is the content of
Message 1?
● Position
● Time
● Medical transport message
● Nature of distress message

11. In his judgment or if a result of a Radio Medical advice, whom shall the master request a visit to Doctor
ashore if necessary for any crew member?
● CIRM
● Local Agent
● MRCC
● Ship owner

12. What is a class “A” EPIRB?


● A satellite-based maritime distress and safety alerting system
● A high efficiency audio amplifier
● An automatic, battery-operated emergency position indicating radio beacon that floats
free of a sinking ship
● An alerting device notifying mariners of imminent danger

13. What does the DSC control unit do if the GMDSS radio operator fails to insert updated information
when initiating a DSC distress alert?
● It will initiate the DSC distress alert and default information will automatically be
transmitted.
● It will initiate the Dsc distress alert, but any station receiving it will have to establish contact with
the distressed vessel to determine its identity, position and situation
● It will abort the transmission and set off an audible alarm that must be manually rest
● It will initiate the DSC distress alert but, as no information will be transmitted, rescue personnel
will not be able to identify the vessel, its position or its situation.

14. Which statement is TRUE regarding a vessel equipped with GMDSS equipment that will remain in Sea
Area A1 at all times?
● The vessel must be provided with a radio installation capable of initiating the
transmission of ship-to-shore Distress alerting from the position from which the ship is
normally navigated
● HF or MF DSC may satisfy the equipment requirement
● VHF DSC alerting may be the sole means of Distress alerting
● HF SSB with 2182 KHz automatic alarm generator may satisfy the equipment requirement

15. What is the IAMSAR procedural term that is used to separate portions of a message or one message
from another?
● Negative
● Roger
● Affirmative
● Break

16. How can reception of certain NAVTEX broadcasts be prevented?


● Stations are limited to daytime operation only
● Automatic receiver desensitization during night hours
● The receiver can be programmed to reject certain stations and message categories
● Coordinating reception with published broadcast schedules

17. What is the signal transmitted in H3E mode


● A full carrier and both upper and lower sidebands
● A full carrier and two sidebands
● A full carrier and one sideband
● A full carrier and the upper sideband

18. Describe which from the following includes the proper watchkeeping?
● Setting the DSC scanner only to the mandatory 2 MHz and 8 MHz, maintain continuous watch
on 2812.0 KHz or 4125.0 kHz, notify the Master of any distress traffic heard
● After silencing an alarm all displays and/or printouts are read, monitoring all required
frequencies in the proper mode, notifying the Master of any Distress alerts.
● Monitoring all required in the proper mode, setting the DSC scanner to 2 MHz and 8 MHz for
ships in the vicinity, notifying the Master of any distress alerts.
● Notifying the Master of any distress alerts, setting the DSC scanner to 2MHz, 4 MHz and 8 MHz
for ships in the vicinity, monitoring all required frequencies in the proper mode.

19. What can you say about the full implementation date of GMDSS?
● The full implementation was February 1, 1999
● The full implementation was February 1, 2000
● The full implementation was February 1, 1998
● The full implementation was February 2, 1999

20. What is this service publication issued by ITU in pursuant of Article 20 of the Radio Regulation and this
List shall be provided to all ship radiotelegraph stations as prescribed in Appendix 16 to the aforementioned
regulations?
● ITU LIST VIIA LIST OF Call Signs and Numerical Identities
● ITU LIST VI List of Radiodetermination and Special Service Stations
● ITU LIST V List of Ship Stations and Maritime Mobile Service Identity Assignments
● ITU LIST IV Coast Stations/ List of Radiodetermination and Special Services.

21. How would you summarize the number of side bands present in the J3E mode?
● It has one upper sideband
● It has two sidebands and a carrier
● It has two carriers and one sideband
● It has one lower sideband

22. How would you describe the signal transmitted in H3E mode?
● It is a two sidebands upper and lower.
● It is a full carrier and the upper sideband
● It is a full carrier and both upper and lower sidebands
● It is a reduced carrier and the lower sideband

23. Regarding GMDSS radio installation, how would you describe the requirement for emergency and
reserve power?
● An emergency power source for radio communications is not required if a vessel has proper
reserve power (batteries)
● For the compulsory ships must have emergency and reserve power sources for radio
communications
● There is only one of the above is required if a vessel is equipped with a second 406 EPIRB as a
backup means of sending a Distress alert
● The reserve power source is not required for radio communications.

24. How would you compare the frequency range for High Frequency?
● From 10-30 MHz
● Form 30-300 kHz
● From 300-3000 kHz
● From 3-30 MHz

25. What is the most important practice that a radio operator must learn?
● Operate with lowest power necessary
● Monitor the channel before transmitting
● Speak aloud so that he can be heard by the receiving stations
● Transmit with the highest power necessary

26. Choose from the following ITU List that contains important information for the mariner in relation to
radiocommunications, including the GMDSS (Global Maritime Distress and Safety System) and CP (Public
Correspondence) service?
● ITU LIST V List of Ship Station and Maritime Mobile Service Identity
● ITU LIST VI List of Radiodetermination and Special Service Stations
● ITU LIST VIIA List of Call signs and Numerical Identities
● ITU LIST IV Coast Station/ List of Radiodetermination and Special Service

27. If your vessel is in the Sea of Japan and logged-in to the POR satellite. Which Inmarsat Earth station
would you select for routine Inmarsat-C traffic?
● Southbury (USA), Thermopylae (Greece), Psary (Poland), or Tangua (Brazil)
● Auckland (New Zealand), EIK (Norway), Sentosa (Singapore), or Beijing (PRC)
● Beijing (PRC), Santo Paula (USA), Auckland (New Zealand), or Sentosa (Singapore)
● Yamaguchi (Japan), Southbury USA), Pune (India), or Haifa (Israel)

28. What does BDE Stands for?


● Below data Equipment
● Below Deck Equipment
● Below deck Enhancement
● Bits Digital Equipment

29. Regarding the radio operator requirements for a GMDSS equipped ship station, choose from the
following statement is FALSE?
● While at sea, all adjustments or radio installations, servicing or maintenance of such installations
that may affect the proper operation of the GMDSS station must be performed by under the supervision
of a qualified
● Maintaining a record of all incidents connected with radio-communications service that
appear to be of importance to Safety of life is not required
● One of the qualified GMDSS radio operators must be designated to have primary responsibility
for radiocommunications during Distress incidents
● A qualified GMDSS radio operator, and a qualified backup, must be designated to perform
Distress, Urgency and Safety communications

30. How could a SART’s effective range be maximized?


● If possible, the SART should be mounted horizontally so that its signal matches that of
the searching radar signal
● Thor SART into the sea as soon as possible to begin transmitting
● The SART should be placed in water immediately upon activation
● Switch the SART into the \”high\” power position

31. How far away the SART indicated by a series of dots on the radar screen display?
● 3 nautical miles
● 2 nautical miles
● 5 nautical miles
● 4 nautical miles

32. Approximately how does a nearby SART indicated by a series of dots on the radar display?
● 3 nautical miles
● 2 nautical miles
● 5 nautical miles
● 4 nautical miles

33. Approximately how close a SART indicated by rings or circles on the radar display?
● 2 nautical miles
● 4 nautical miles
● 1 nautical miles
● 3 nautical miles

34. What can be defined as radio transmissions intended to facilitate the finding of the mobile unit in
distress of the location of survivors?
● radio propagation
● alerting signals
● Burst form the EPIRB
● locating signals

35. Which of the following ALRS Volume that includes information for: A. Worldwide communication
requirements for distress, search and rescue B. Extracts from SOLAS and ITU Regulations C. Distress and
SAR (incorporating MRCC and MRSC contacts) D. Worldwide NAVTEX and Maritime Safety Information
● Volume 3 (NP283)
● Volume 5 (NP285)
● Volume 6 (NP286)
● Volume 4 (NP284)

36. Which of the following situations would normally use the Safety priority?
● Treatment of crewmember breaking a leg in a Cargo hold
● A fire in the generator flat/spaces
● Conducting shipboard fire drill Heating
● Conducting Compass Deviation Curve

37. Which of the following 3-letter signal means “Give rectal saline slowly to replace fluid loss”?
● MSD
● MSA
● MSB
● MSC
38. Under what condition would you relay a DSC Distress alert?
● When the Coast Station determine further help is necessary
● You could replay anytime if you feel deem necessary
● If the mobile unit in Distress is incapable of sending further distress alert communication.
● You should never relay a distress alert the Coast Station and RCC will do that

39. In the mandatory system under SOLAS Regulation V/11-1, where are the locations of ship reporting
system are shown?
● On the relevant charlets
● On the relevant Admiralty Charts
● On the T & P Notices
● On the Chart Catalogue

40. You are making a ship to shore public correspondence calling and the coast station does not reply to
your VHF DSC calling after the second time, how long must you wait before calling again?
● 25 minutes
● 20 minutes
● 15 minutes
● 10 minutes

41. Your ship is involved in a search and rescue operation, and you are bothered by interference from
other stations, which signal are you going to impose radio silence?
● “PANPAN”
● “MAYDAY”
● “SEELONCE MAYDAY”
● “SEELONCE FEENEE”

42. In facilitating visual signaling, what are the most commonly used important two-letter signals which
specify “I require immediate assistance”?
● AL
● BC
● AN
● CB

43. Which of the following two letter signal are the most commonly used as “I require immediate
assistance”?
● CB
● AL
● MB
● AN

44. What are the two-letter signals which means “I am distress and require immediate assistance”?
● NM
● NC
● CI
● GB

45. What does POR stand for?


● Pacific Ocean Region
● Personal Office Rights
● Pacific Organization Region
● Pacific Ocean Rights

46. Which from the following obligatory requirement for GMDSS?


● SOLAS Convention ships of 300 gross tonnage or more
● Coastal vessel of less than 300 gross tons
● Vessel operating outside of the range of VHF coast radio stations
● All vessels capable of international voyage

47. How is a distress message normally initiated through INMARSAT?


● All INMARSAT units have a dedicated key that can be pressed for immediate action
● Certain INMARSAT units have a dedicated key that can be pressed for immediate action,
while other systems provide menu-driven features
● By transmitting the distress message on the U.S. Coast Guard\’s dedicated monitoring channel
● By addressing the word \”DISTRESS\” in the first line of the message\’s preamble……..

48. What does “PSTN” stands for?


● Public Switched Transport Network
● Personal Services Telephone Network
● Public Switched Telephone Network
● Public Service Telegraph Network

49. Which system is most likely to be affected by atmospheric disturbance?


● MF/HF radiotelephony
● VHF DSC
● SafetyNET
● Inmarsat

50. Reference to Radio Regulation Article 1 section III Radio Services. What is meant by a
radiocommunication service providing links between artificial satellites
● mobile-satellite service
● space operation service
● fixed-satellite service
● inter-satellite service

51. Reference to Radio Regulation Article 1 section III Radio Services. What is meant by a
radiocommunication Service between earth stations at given positions, when one or more satellites are
used?
● mobile-satellite service
● space operation service
● fixed-satellite service
● inter-satellite service

52. How would the third symbol in designation of radio emission under the ITU rules refers to?
● It is a type of information to be transmitted
● It is a nature of the signals modulating the main carrier
● It is a type of the modulation of the main carrier
● It is a bandwidth

53. You are intending to call a particular Coast Station, which publication shall you consult to find out the
frequencies used by the said station?
● ITU list of ship station
● ITU list of coast station or ALRS VOL. 1 Part 1 & 2
● ITU list of radio determination and special service station
● ITU list of call signs and numerical identities

54. In ICS, How would you show the signal for the phrase “I AM SINKING”?
● The code signal “AQ”
● The code signal “DX”
● The code signal “CX”
● The code signal “TI”

55. Which from the following instruments is used to measure the specific gravity of a lead acid battery?
● A voltmeter
● A ammeter
● A galvanometer
● A hydrometer

56. Identify from the following vessel operating in which sea area(s) are required to carry either Inmarsat or
HF equipment or a combination thereof under GMDSS?
● A3
● A4
● All sea area
● A1

57. What should be done on a DSC Relays of DSC Distress alerts received from other ships?
● Only when the original DSC call is not acknowledged and no follow-on Distress traffic has
been heard
● Preferably by MF/HF voice or TELEX directly to the RCC conveying the follow-on Distress traffic
has been heard
● Only by Inmarsat-B voice or TELEX with Distress priority if no follow-on distress traffic has been
heard
● Only by Inmarsat-C TELEX with Distress priority conveying the follow-on distress traffic that has
been heard
58. What should you do when attempting to contact other vessel on VHF Channel 16?
● Limit calling 30 seconds
● If no answer is received, wait 2 minutes before calling vessel again
● Limit Calling to 30 seconds and if no answer is received, wait 2 minutes before calling
vessel again
● Channel 16 is used for emergency calls only

59. What element of a DSC call indicates the type of communication desired during the subsequent
correspondence?
● Message 2 of distress calls
● Message 3 of distress calls
● Message 1 of distress calls
● Message 4 of distress calls

60. What does DSC stands for?


● Data Selective Calling
● Data Service Code
● Digital Selective Calling
● Digital Service Code

61. What does NAU stands for?


● Network Universal Address
● National User Authority
● Network User Authority
● Network User Address

62. What is the emission designation for VHF-FM?


● H3E
● F3C
● F3E
● J2B

63. What is the International Code 3-letter signal that means “General condition of the patient is
unchanged’?
● MAR
● MAQ
● MAT
● MAU

64. GMDSS is required for which of the following?


● All vessels Capable of international voyages
● Coastal vessel of less than 300 gross tons
● SOLAS Convention ships of 300 gross tonnage or more
● Vessels operating outside of the range of VHF coastal radio stations
65. What is the approximate distance of D layer of the ionosphere from earth?
● 160-400 kms
● 95-130 kms
● None of the above
● 50-95 kms

66. In ICS, How would you expressed the code signal for longitude?
● 4 or 5 figures preceded by the letter \”C\”
● 4 or 5 figures preceded by the letter \”G\”
● 4 or 5 figures preceded by the letter \”D\”
● 4 or 5 figures preceded by the letter \”E\”

67. At sea, what must a GMDSS equipped vessel do when she is operating in coastal waters (Sea Area
A1)
● Maintain a continuous DSC Watch on MF 2187.5KHZ
● Maintain a continuous DSC Watch on HF8376.5KHZ
● Maintain a continuous DSC Watch on HF 8414.5KHZ
● Maintain a continuous DSC Watch on VHF channel 70

68. When in standby mode, the SART battery should last up to?
● 24 hours
● 96 hours
● 48 hours
● 8 hours

69. Which of the following frequency bands would most likely provide reliable communications between two
stations that are 100 miles (160 km) apart?
● The Very High Frequency (VHF) band
● The High frequency (HF) band
● The Medium Frequency (MF) band
● The Low frequency (LF) band

70. What does FAX stands for?


● Frequency Shift Keying
● Forward Automatic Machine
● Abbreviation for Facsimile
● Forward Error Correction

71. What does IMO resolution A. 814 (19) embodies guidelines for avoiding?
● Unnecessary usage of VHF
● Unnecessary communication
● Interference
● False distress alert
72. From the following statement, how would you best describes amplitude modulation?
● It is the frequency is varied in synchronization with the modulating signal
● It is the amplitude of the carrier is change but there is still only single frequency being
transmitted
● Its it the character data from the terminal is changed to audio tones
● It is the amplitudes in the sidebands(s) changes but the radio carrier frequency remains
constant

73. An operating method in which only one station can transmit at a time indicates the end of a
transmission by use of the word “over” is called?
● Full duplex Operation
● Semi-duplex operation
● Duplex operation
● Simplex operation

74. Explain what is the effect of temperature on the specific gravity of lead acid battery?
● Higher temperature results in a lower specific gravity reading
● Higher temperature results in a higher specific gravity reading
● Temperature has no effect on the specific gravity reading
● Lower temperature results in a lower specific gravity reading

75. What is the GMDSS equipment which provides a locating signal to SAR vessel?
● PLB
● INMARSAT- E/ L-BAND EPIRB
● EPIRB
● SART

76. Who will collate and disseminate to Contracting Governments all relevant information with regard to
any adopted ship reporting system?
● AMVER
● SOLAS
● USCG
● IMO

77. If your vessel is entering pirate risk area, which ALRS Volume you will refer for anti piracy contact
table?
● Volume 3 (NP283)
● Volume 2 (NP282)
● Volume 5 (NP285)
● Volume 1 (NP281)

78. What does LUT Stands for?


● Local User Terminal
● Local User Transport
● Land User Terminal
● Land User Transport

79. How would you demonstrate your action after sending a false Distress alert on VHF?
● Make a voice announcement to cancel the alert on Ch. 16
● Send a DSC cancellation message on Ch. 70
● Make a voice announcement to cancel the alert on Ch. 22A
● Make a voice announcement to cancel the alert on Ch 13

80. How are NAVTEX broadcast transmitted?


● Regularly, before and after the radio telephone silent periods
● Immediately following traffic lists
● On request of maritime mobile stations
● In categories of messages indicated by a single letter or identifier

81. You receive an urgency call on VHF Channel 70. On what frequency are you going to listen for the
complete message?
● Channel 16
● Channel 12
● Channel 70
● Channel 13

82. What equipment can be used to receive Maritime Safety information?


● HF SITOR (NBDP), Inmarsat B or NAVTEX
● NAVTEX, EGC receiver or HF SITOR (NBDP)
● ECG receiver, Inmarsat-B or F77 terminal
● All of these answers are correct

83. The MMSI number plan is used in all of the following GMDSS equipment “except”?
● EPIRB
● MF/HF DSC
● VHF-DSC
● SART

84. Which of the following is “NOT” a vessel equipment and personnel requirements of GMDSS?
● License radio operators
● Ship Station using Morse Telegraphy
● Equipment carriage requirements
● Distress alerting and response

85. What does DTE stands for?


● Digital Transport Equipment
● Data Terminal equipment
● Data Transport Equipment
● Digital Terminal Equipment

86. Where should the GMDSS radio log be kept on board ship?
● Sea Cabin
● At the GMDSS operating position
● Anywhere on board the vessel
● Captain\’s Office

87. What names are used to prevent the reception of unwanted broadcasts by vessel’s utilizing the
NAVTEX system?
● Operating the receiver Only during daytime hours
● Automatic receiver desensitization during night hours
● Coordinating reception with published broadcast Schedules
● Programming the receiver to reject unwanted broadcasts

88. What is the primary purpose of bridge to bridge Communications?


● Search and rescue emergency call only
● All short-range transmission aboard ship
● Navigational communications
● Transmission of Captain\’s orders from the bridge

89. Regarding a GMDSS equipped ship, choose from the following statement is FALSE?
● That a ships must have the required equipment inspected at least once every five years
● That a conditional or partial exemption may be granted, in exceptional circumstances for a single
voyage outside the sea area for which the vessel is equipped
● That a ships must carry at least two persons holding GMDSS Radio Operator\’s license for
Distress and Safety Communications purposes
● That the regulations apply to all passenger ships and cargo ships of 300 gross tons and
upwards.

90. What is the term for any governmental office responsible in discharging the obligations undertaken in
the convention of ITU and regulation?
● Country
● The Union
● Administration
● Telecommunications office

91. Which of the following statements Concerning testing and maintenance of SART is true?
● An at -Sea GMDSS maintainer is not able to test a SART as it is hermetically sealed
● Testing a SART should be performed only in controlled environment as test signal may
be misinterpreted as a genuine distress situation
● All of the above
● A SARTs\’ battery must be replaced within ninety (90) days after the expiration date imprinted on
the unit

92. How frequent is the DSC controller required to be tested while underway
● Once a day
● Once a week
● Twice a week
● Once a month

93. Which of the following is NOT a provision for maintenance should a ship operating in area A-1 must
have?
● Shore maintenance
● Onboard Radio technician
● At Sea Maintenance
● Duplication of equipment

94. What does NBDP stands for?


● Narrow Band Direct Process
● Narrow Band Direct Printing
● Navigational Band Direct Printing
● Network Band Division Printing

95. Which Regulation in SOLAS Chapter IV states that a record shall be kept, to the satisfaction of the
Administration and as required by the Radio Regulations, of all incidents connected with the radio
communication service which appear to be of importance to safety of life at sea?
● Regulation 16
● Regulation 15
● Regulation 17
● Regulation 18

96. Your ship received a distress relay on DSC VHF channel 70, on what channel would you reply?
● 16
● 14
● 11
● 15

97. Choose from the following is a valid 22-MHz ITU Channel?


● The Ch-22A VTS
● The Ch-70 (DSC only)
● The HF CH-2206
● The VHF Ch-22

98. What does MMSI stands for?


● Mobile Maritime Service Identities
● Maritime Medium Service Integrity
● Maritime Mobile Search Identity
● Maritime Mobile Service Identities
99. Which from the following statements is FALSE for MF/HF DSC Distress call?
● It always requests J3E Voice follow-on communications so other vessel can hear the May
day
● It does not contain all the information normally of interest in on-scene Distress communications
● It will send a more detailed Distress format if time permits and operator data entries are correctly
performed
● It will send the minimal necessary information using the \”Distress Button\” or \”Distress Hot
Key\”

100. In the international radio service which of the following message have the highest priority?
● Weather observations
● Message to agents
● Telephone call to ship’s owner Master report to an agency
● Master report to an agency

101. Which statement concerning the testing of a compulsory radiotelephone station is false?
● Calling the VHF Ch16 or 2182.0 KHz is the most effective method
● Test may be accomplished by using the equipment for normal business
● A daily test is necessary unless the equipment was used for routine traffic
● The test may not interfere with communications in progress and must wait or be suspended of a
request to do so is made

102. What is the code used to obtain information about a subscriber in a country other than where the
services provider is located?
● 13
● 12
● 11
● 00

103. What are the conditions under GMDSS, whereby a ship is NOT allowed to depart from any port?
● The vessel has temporary waiver of its radio license and Safety Certificate
● The vessel is carrying more than the required number of qualified GMDSS radio operators
● The vessel is not capable of performing all required Distress and Safety functions
● The vessel is not carrying a GMDSS radio maintainer, but has provided for shoreside
maintenance plus duplication of equipment if required

104. What is normally received by the master through Inmarsat-C or NAVTEX relative to weather
forecasting, detailing the condition duration and the geographical extent of the advisory?
● Weather advisory
● Weather bulletin
● Weather transmission
● Weather forecast

105. Which channel is utilized for the requirements bridge to bridge watch?
● VHF-FM on channel 16
● The vessel\’s working frequency
● DSC on channel 70
● VHF-Fm on channel 13

106. What is meant by the term “radio silence”?


● Restricted working is lifted
● Stations not directly involved with the ongoing distress communications may not
transmit on the distress frequency or channel
● Stations may transmit every time there is silence in the frequency of channel that is being used
in the distress traffic
● Restricted working is imposed

107. What is define as the area within the radio telephone coverage area of at least one VHF Coast Station
in which continuous DSC alerting is available as defined by the IMO regulation for GMDSS
● SEA AREA A4
● SEA AREA A3
● SEA AREA A2
● SEA AREA A1

108. Which is of the following is functional or carriage requirements for compulsory vessels?
● None of these answers are correct
● A compulsory vessel must be able to transmit and respond to Distress alerts and carry only one
(1) FCC licensed GMDSS Radio Operator in sea areas A1 &A2
● A compulsory vessel must be carry at least two (2) FCC licensed GMDSS Radio Operator in sea
areas A3 & A4
● A compulsory vessel must satisfy certain equipment carriage requirements based on the
intended sea area of operation

109. What causes the SART to begin transmission?


● It is either manually or water activated before radiating
● It begins radiating only when keyed by the operator
● When activated manually, it begins radiating immediately
● After being activated the SART responds to radar interrogation

110. What is defined as an area excluding sea areas A1 and A2, within the coverage of an INMARSAT
geostationary satellite in which continuous alerting is available?
● Sea Area A4
● Sea Area A3
● Ocean Area Regions AOR-E, AOR-W, POR or IOR
● Coastal and Inland Waters

111. Which GMDSS system is least likely affected atmospheric disturbances?


● HF NBDP
● NAVTEX
● MF NBDP
● INMARSAT

112. Choose from the following statements the preparation to be carried out during an annual GMDSS
station inspection?
● Licensed GMDSS operators may not be required to demonstrate equipment competencies but
all required equipment must be fully operational
● All required equipment must be fully operational and nay required publications that are not
current must been ordered
● All required documents and publications might have to be produced and GMDSS
operators may be required to demonstrate equipment competencies
● GMDSS operators may be required to demonstrate equipment competencies and any required
equipment that is not fully operational can be repaired at the next port of call as long as there is
functional duplications

113. SITOR equipment is a full, partial or alternate carriage requirement under GMDSS for vessels operating
in which sea area(s)?
● A1 and A2
● A3 and A4
● A1
● A3

114. Which of the following statements is NOT a basic concept of GMDSS?


● Search and rescue authorities ashore can be alerted to a distress situation
● Shoreside authorities and vessels can assists in a coordinated SAR operation with minimum
delay
● Shipping in the immediate vicinity of a ship will be rapidly alerted
● Shipping correspondence is made easy and concise

115. What is the international VHF digital selective calling channel?


● 156.35 MHz
● 500 MHz
● 156.525 MHz
● 2182 kHz

116. What should an operator do to prevent interference?


● Turn off transmitter when not in use
● Transmission should be as brief as possible
● Monitor the channel before transmitting and make transmission as brief as possible
● Monitor channel before transmitting

117. As soon as details of the incident or accident have become clear enough to indicate SAR involvement ,
what should be transmitted by the Rescue co-ordination Center (RCC) and On scene co-ordinator (OSC) to all
vessels and other stations?
● POSREP
● SHIPREP
● MAREP
● SITREP

118. How would you compare the frequency range for Ultra High Frequency?
● From 3-30 MHz
● From 10-30 MHz
● From 30-300MHz
● From 300-3000MHz

119. What do you call the Radiowave propagation in its distance traveled by a radiowave from one wavetop
to another?
● Magnetic field
● Static duration
● Wavelength
● Frequency

120. Which GMDSS equipment is associated with the land or terrestrial systems?
● GPS
● EPIRB
● VHF-MF-HF
● Inmarsat-C

121. When working aloft, what is the safety precaution to keep in mind?
● To ensure the transmission mode is at H3E
● To ensure the transmitter by removing the supply fuses
● To disable the transmitter is in standby position only
● To lock the frequency on 2812 kHz with transmitter on low power and high

122. When working aloft, what is the safety precaution to keep in mind?
● To ensure the transmission mode is at H3E
● To disable the transmitter by removing the supply fuses
● To ensure the transmitter is in standby position only
● To lock the frequency on 2812 kHz with transmitter on low power and high

123. At what distance is VHF propagation normally limited?


● Around 500 miles
● Around 700 miles
● Around 850 miles
● Around 800 miles

124. Which ADMIRALTY LIST of RADIO SIGNAL (ALRS) publication can you find the frequencies and times
of traffic list by coast stations?
● Vol.1
● Vol. 4
● Vol. 2
● Vol. 3

125. Which of the following licenses meets the requirements to serve as a GMDSS operator?
● Marine Radio operator’s Permit
● General Radiotelephone Operator’s License
● GMDSS Radio Maintainer’s License
● GMDSS Radio Operator’s License

126. If medical evacuations are being considered, what should be weighed for the benefits to both the
person needing assistance and to the rescued personnel?
● Against partial loss of the subject injured, cause by a peril insured
● The claim for cargo damages that may be held against the shipowner
● Against the inherent dangers of such operations
● To maintain a high state of readiness against the possibility of damage and subsequent flooding

127. What is the code use to obtain information from the International Operator about the country where
service provider is located?
● 01
● 00
● 10
● 11

128. What does EPIRB stand for?


● Enhanced Position Indicating Radio Beacon
● Emergency Position Indicating Radio Beacon
● Emergency Position International Radio Beacon
● Exit Position Indicating Radio Beacon

129. What is this code to obtain information about subscribers in the country where the service provider is
located?
● 14
● 00
● 13
● 11

130. How would you explain the Maritime Identification Digits (MID) issued by the International
Telecommunication Union (ITU)?
● It identify Ship Station
● It identify a call sign
● It identify their home country (Nationality)
● It identify a Port of registry
131. The phases of emergency at sea established to classifying incidents and determining the actions to be
taken are the following “EXCEPT”:
● Critical phase
● Alert phase
● Uncertainty phase
● Distress phase

132. What does SRR stands for?


● Search and Rescue Regions
● Safety Regions on Rescue
● Service Rules and Rescue
● Standard Regions
133. In which Sea Area the radio coverage of MF coast stations where a continuous Digital Selective Calling
is available?
● Sea Area A3
● Polar Area
● Sea Area A1
● Sea Area A2

134. What does EGC stands for?


Emergency Group Code
● Enhanced Group Call
● Enhanced Group Code
● Emergency Group Call

135. Who has ultimate control of service at a ship’s radio station?


● An appointed licensed radio operator who agrees to comply with all Radio Regulation in force
● A holder of Fist Class Radiotelegraph Certificate with Six months service endorsement
● The master of the ship
● The Radio Officer-in-Charge authorized by the Captain of the vessel.

136. If a female Oiler was badly injured and you are communicating with a passenger ship that has a
Doctor and hospital onboard. You want to describe her general condition as serious. Which code should your
message contain?
● MAN
● MAO
● MAM
● MAP

137. At sea, all required equipment (other than Survival Craft Equipment how it must be proven operational?
● Weekly testing of all Survival Craft equipment and other compulsory equipment
● Testing at least every 48 hrs
● Daily testing of the Survival Craft equipment and weekly test of the other equipment
● Daily testing or daily successful use of the equipment

138. Which of the following regions lies outside Sea Area A1,A2 and A3?
● Indian Ocean Region
● Pacific Ocean Region
● Atlantic Ocean Region
● Polar Regions

139. What does CCIR stands for?


● International Radio Consultive Committee
● International Radio Consultative Committee
● International Radio Committee Code
● International Radio Code Committee

140. What is the code used to gain access to a store-and-forward unit (sfu) for international calls?
● 21
● 24
● 22
● 17

141. Which of the following are the MF/HF DSC Distress watch frequencies
● 2177.5, 4210.0 , 6314.0, 8416.5, 12579.0, 16806.5 kHz
● 2182.0, 4125.0, 6215.0, 82191.0, 12290.0, 16420.0 kHz
● 2187.5, 4207.5, 6312.0, 8414.5, 12577.0, 16804.5 kHz
● 2174.5, 4177.5, 6268.0, 8376.5, 12520.0, 16695.0 kHz

142. With reference to Radio Regulation Chapter 1 Article Section IV 1.78. Which of the following is a
station in the radio navigation service not intended to be used while in motion?
● Coast earth station
● Ship station
● Ship earth station
● Coast station

143. Reference to radio Regulation Article Section IV Radio station and systems, what is meant by an earth
station in the fixed satellite service or in some cases, in the mobile satellite service located at a specified
fixed point or within a specified area on land to provide a feeder link for the mobile-satellite service?
● Land mobile station
● Land earth station
● Base earth station
● Land mobile earth station

144. What does ITU stands for?


● International Telegraph User
● International Telegraph Union
● International Telecommunications Union
● International Telephony Union
145. If a crew member that was ill has died, which code should your message contain?
● MPL
● MPN
● MPO
● MPR

146. What is the primary reason GMDSS impose carriage requirements for different radio subsystems within
a single sea area?
● Redundancy in duplicating all operational functions in the event of a system failure
● GMDSS vessel must be equipped to communicate in all mode with coast radio stations
● The combined capabilities of redundant subsystems mitigate the risk of a single point of
failure
● Different subsystems4 are required to meet the specific equipment carriage requirements of
national authorities

147. What is the total frequency are available for DSC Distress alerting?
● Seven (7)
● One (1)
● Five (5)
● Two (2)

148. What is the code used to obtain maritime assistance if the vessel requires assistance or towing or has
encounter oil pollution, etc?
● 41
● 42
● 43
● 39

149. Which of the following information is “NOT” vital in a distress follow-on voice transmission after a DSC
alert?
● Company emergency contact information
● Ship’s name, call sign and MMSI number
● Physical description of the vessel and number of person on board
● Ship’s position nature of distress and assistance requested.

150. Select from the following the quantity of GMDSS radio maintainers must be carried aboard a
compulsory vessel if the At-Sea maintenance method is used?
● One (1) regardless of sea areas of operations
● Two (2) in Sea area A1 and A4
● One (1) in Sea Area A1
● Two (2) in Sea area A3 and A4

151. To keep it in a working condition and make the best use of MF DSC equipment, how often should be
tested?
● Weekly, by contacting a coast station
● Monthly, by means provided by the equipment
● Monthly, by contacting a coast station
● Weekly, by means of an internal test
152. Which of the following channels and modes should be used when initiating a distress alert
transmission?
● Channel 70 DSC
● Channel 6 Radiotelephony
● Channel 6 DSC
● Channel 13 Radiotelephony and channel 16 DSC

153. Why is regular updating of a ship’s position in an Inmarsat-C installation is necessary?


● To keep abreast with AMVER reporting
● To enter the correct data to the disk antenna
● To have the correct position in case of accidents
● To keep the correct Inmarsat region

154. Which of the following 3 letter signal means “To induce sleep, give two sedative tablets”?
● MSJ
● MSL
● MSK
● MSM

155. What does MRSC stands for?


● Maritime Region Sub Centre
● Maritime Rescue Sub Centre
● Maritime Rescue Safety Centre
● Maritime Rescue Service Centre

156. With reference to Radio Regulation Chapter Article 1 Section 1.88. Which of the following is a mobile
earth station in the maritime mobile-satellite service located on board ship?
● Radionavigation land station
● Radiodetermination station
● Radiolocation land station
● Radionavigation mobile station

157. Reference to Radio Regulations Article 1 Section III Radio Service. What is meant by a mobile-
satellite service in which mobile earth stations are located on board ships; survival crafts stations and
emergency positions indicating radio beacon stations may also participate in this service?
● Land mobile-satellite
● Maritime mobile service
● Maritime mobile-satellite service
● Land mobile service

158. Reference to Radio Regulations Article 1 Section III Radio Service. What is meant by a mobile service
between coast stations and ship stations, or between shipstations or between associated on-board
communication stations; survival crafts stations and emergency position indicating radiobeacon stations
may also participate in this service?
● Land mobile-satellite
● Maritime mobile service
● Maritime mobile-satellite service
● Land mobile service

159. Reference to Radio Regulations Article 1 Section III Radio Service. What is meant by a
radiocommunication service between mobile and land stations or between mobile stations?
● Land mobile-satellite service
● mobile service
● Maritime mobile service
● Land mobile service

160. What do you call a radio communication operation service between mobile and land stations or
between mobile stations?
● maritime mobile service
● land mobile
● mobile service
● land mobile satellite service

161. What is a ship earth station?


● A communications system which provides line of sight communication between vessels at sea
and coast stations
● A Maritime mobile satellite station located at a coast station
● A mobile satellite station located on board a vessel
● An automated ship-to-shore distress alerting system

162. Which of the following details of additional service supplied according to ITU List IV is excluded? I.
Medical advice II. Navigational and Meteorological warnings III. MSI (Maritime Safety Information) IV. AIS
(Automatic Identification System) V. Radiobeacon Station VI. VTS Station VII. DGNSS Station VIII. Contact
Information of RCC IX. Port information and Pilot station
● I, II, III, VII and VIII only
● I, II, III, IV, VI, VIII and IX only
● V and VII only
● I, II, III, IV, VIII and IX only

163. What can you say about the frequency used in the international NAVTEX system?
● Navtex frequency is 518 KHZ
● Navtex frequency is 2182 KHZ
● Navtex frequency is 4209.5 KHZ
● Navtex frequency is 490KHZ

164. Can you select from the following which system has the least effective radius of operation?
● A HF SITOR
● a NAVTEX
● A VHF DSC
● A MF SITOR

165. Under what International Convention the Automatic Identification system (AIS) was introduced?
● UNCLOS
● STCW-95
● IMO
● SOLAS

166. Which equipment on the bridge is used for locating a SART?


● RADAR
● NAVTEX
● RDF
● AIS

167. In flag signaling, what does a hoist or signal is said to be at the DIP when it is hoisted?
● Both b and c
● Halfway to the mast
● About half of the full extent of the halyards
● To the full extent of the halyards

168. What should be the colour of AIS-SART on all surface where this will assist detection?
● Matte black
● High visible yellow/orange
● High visible green
● White with orange band

169. Select from the following statement the best way to classify a DSC Distress alert?
● Must always be sent on VHF Ch.70 then 2 MHz MF then 8MHz HF
● Will always be sent on one or more of the DSC Distress frequencies
● Must always be sent on one or VHF Ch.70
● Must always be sent on MF 2 MHz and one other DSC Distress frequency

170. In Chapter 3 of the International Code of Signals, Section 2: Request for Medical Assitance, which
code should your message describing that the patient has had no previous illness?
● MBC
● MBD
● MBF
● MBE

171. If a vessel is operating in an area outside Zones A1, A2 and A3 this vessel is operating within what
area for GMDSS purposes
● Sea area A4
● Great Lake area
● Baltic Sea Area
● Persian Gulf Area
172. What does SCADA stands for?
● Supervisory Center and Data Acquisition
● Supervisory Control and Data Acquisition
● Special Control and Data Authority
● Super Control on Data Authority

173. How many MF frequencies are available for DSC distress related calls?
● Five
● One
● Two
● Four

174. Which Regulations from the SOLAS Chapter IV states there shall be available at all times, while the
ship is at sea, a supply of electrical energy sufficient to operate the radio installations and to charge any
batteries used as part of a reserve source or sources of energy for the radio installations?
● Regulation 13
● Regulations 14
● Regulations 12
● Regulations 16

175. How would you best describe the GMDSS Sea Area A2 as being within?
● It is within HF RT range of at least one shore station providing continuous HF DSC alerting
● It is within range of the Inmarsat system with continuous watch
● It is within RT range of an MF shore station providing continuous MF watch
● It is within MF RT range of at least one shore station providing continuous VHF DSC
alerting

176. For VHF-FM voice signals, what is the emission designation?


● It is F3C
● It is H3E
● It is F3E
● It is J2B

177. What is the most important factor in determining the range of a VHF station?
● The height of antenna
● Type of terminals/connectors use
● Quality of the antenna
● Length of wire used

178. What is the code e is used for the retrieval of navigational warnings?
● 51
● 52
● 43
● 42
179. Acknowledgement of a DSC distress alert by use of DSC is normally made by?
● Coast station only
● Navigational lanes
● Inland and Coastal waters
● Coastal and inland waters

180. What is the meaning of “Reserve Source of Energy”?


● High caloric value items for lifeboat, per SOLAS regulations
● The supply of electrical energy sufficient to operate the radio installations for the purpose
of conducting distress and safety Communications in the event of failure of the Ship\’s main
and emergency sources of electrical power
● Diesel fuel stored for the purpose of Operating the powered survival Craft for a period equal to or
exceeding the U.S.C.G and SOLAS requirements
● None of these

181. What does MF stands for?


● Medium Frequency
● Maritime Frequency
● Mobile Frequency
● Marine Frequency

182. Which of the following is not a content of distress relay alert?


● Port of departure and destination of the mobile unit in distress
● Time in which the message is send by the mobile unit
● Nature of Distress of the mobile unit in distress
● Position of the mobile unit in distress

183. The book which aims to provide guidance to those who during emergencies at Sea, may require
assistance or may be able to render assistance is?
● IAMSAR manual
● MERSAR manual
● Admiralty list of radio signals
● ITU list of Coast Station

184. How should the signal from a search and rescue radar transponder appear on radar display
● A series of 13 equally Spaced dots
● A series of 15 equally Spaced dots
● A series of 12 equally Spaced dots
● A series of 11 equally Spaced dots
185. Which of the following equipment should be tested at least once a week?
● SART, EPIRB and handheld VHF sets
● Operation of 2182kHz receiver
● Reserve source of energy if it is not a battery
● Each printer for adequate paper

186. What is automated Mutual-Assistance Vessel Rescue System (AMVER)?


● A satellite-absed distress and safety alerting program operated by the U.S Coast Guard
● A voluntary organization of mariners who maintain radio watch on 500kHz, 2182kHz and
156.800kHz
● An international system operated by the U.S Coast Guard providing coordination of
search and rescue efforts
● A coordinated radio direction finding effort between the Federal Communication Commission
and U.S Coast Guard to assist ships in distress

187. How to alert other ship in the vicinity of the ship in distress and are based on the used on DSC in the
VHF and MF bands?
● Ship to aircraft distress alert
● Ship-to-ship distress alert
● Ship to shore distress alert
● Shore to ship distress alert

188. How long should station logs to be retained when there are no entries relating to distress or disaster
situations?
● For a period of one year from the date of entry
● For period of three years from the date of entry unless notified by the FCC
● Until authorized by the commission in writing to destroy them
● Indefinitely or until destruction is specifically authorized by the U.S Coast Guard

189. In Chapter 3 of the International Code of Signals, Section 2: Request for Medical Assistance, which
code should your message means “ I require a Helicopter urgently with a Doctor”?
● BR 3
● BT 2
● BR 2
● BT 3

190. What would be the following references should be consulted in order to identify the name of a vessel
based on its Maritime Mobile Service Identity?
● ITU List of Radio determination and ship Stations
● ITU Master Plan of Shore Based facilities
● ITU list of Coast Stations
● ITU List of ship stations and Maritime Mobile Service Identity Assignment

191. What does SES stands for?


● Ship Earth Station
● Ship Emergency Station
● Surface Earth Station
● Services Extra Station

192. Which of the following satellite systems if of particular & dedicated importance to search and rescue
missions under GMDSS?
● COSPAS/SARSAT
● GPS
● Inmarsat
● Iridium

193. Which from the following statement regarding 406 MHz EPIRB transmissions is correct?
● Allow immediate voice communications with the RCC
● GMDSS Radio Operator programs an I.D into the SART immediately prior to activation
● Transmits a unique hexadecimal identification number
● Coding permits the SAR authorities to know if manually or automatically activated

194. In Inmarsat operation, what is the two-digit code which corresponds to medical assistance?
● 41
● 42
● 38
● 32

195. Which of the following frequencies and nodes is not allocated for distress alerting in GMDSS?
● 406 MHz via EPIRB
● 1626.5-1645.5 via Inmarsat
● VHF channel 12
● Channel 70 DSC six (6) MF/HF DSC frequencies

196. What element of a DSC call indicates the type of distress situation?
● Message 1 of distress calls
● Message 4 of distress calls
● Message 2 of distress calls
● Message 3 of distress calls

197. In GMDSS, when is the VHF Channel 70 EPIRB can be substitute for a float free Satellite EPIRB?
● SEA AREA A4
● SEA AREA A2
● SEA AREA A1
● SEA AREA A3

198. What is the requirements for the carriage of radio equipment in Sea Area 1?
● A vessel will carry a VHF, EPIRB and MSI receiver
● A vessel will carry a VHF, EPIRB and MF equipment or a satellite EPIRB
● A vessel will carry a VHF, EPIRB, MF & HF equipment and a satellite EPIRB
● A vessel will carry VHF equipment and either a satellite EPIRB or a VHF EPIRB

199. If the watch officer hears “PAN PAN” spoken three times, what does it mean?
● The station is preparing to transmit an urgent message possibly concerning the safety of
a mobile unit or person
● The station is preparing to transmit a safety message possibly concerning the safety of a mobile
unit of person
● The station is preparing to transmit a distress message possibly concerning the safety of a
mobile unit or person
● When the vessel is threatened by grave and imminent danger and required immediate
assistance

200. In case of serious medical emergency to whom should the Master may request Medical advice or
Medical evacuation?
● By sending text message to CES using code 41
● By contracting the nearest CES by Inmarsat phone using phone 32
● Contacting the nearest CES operator using the radiotelephone procedure PAN PAN
● By calling the Designated Person Ashore

201. What is the code to be used in order to charge a telex call using a credit or charge card?
● 32
● 36
● 37
● 33

202. What is the fundamental concept of the GMDSS?


● It is intended to automate and improve emergency communication in the maritime
industry
● It is intended to provide compulsory vessel with a collision avoidance system when they are
operating in waters that are also occupied by non-compulsary vessel
● It is intended to provide more effective but lower cost commercial communications
● It is intended to automate and improve existing digital selective calling procedure and techniques

203. With what system is a NAVTEX message transmitted?


● SELFEC
● FEC
● PLB
● ARQ

204. What does COSPAS/SARSAT stands for?


● Coast Earth Station Search and Rescue Satellite Aided Tracking
● Search And Rescue Satellite Alternating Tracking
● Search and Rescue Satellite Aided Tracking
● Committee on Search and Rescue Aided Tracking

205. Which of the following situations would normally use the voice communication designation “Securite”?
● Message containing information concerning the safety of a mobile unit or person
● Messages concerning On-scene communications
● Messages detailing important navigational warnings
● Messages concerning the Safety of Life At Sea (SOLAS)

206. What does AC stands for?


● Atlantic code
● Alternating Current
● Amplitude Current
● Automatic Code

207. Explain what will cause an individual battery cell to reverse polarity?
● When discharging the battery string if a cell becomes weaker than the remaining cells,
the discharge current will effectively charge the weaker cell in reverse polarity
● Insufficient charging which does not bring all of the cells up to full charge
● The charging circuits are connected in the correct polarity but all of the cells are equally charged
● High discharge rates without allowing for a cool down period

208. What does TX stands for?


● Telex
● Transmitter
● Transformer
● Telephony

209. Which of the following book aims to provide information on all aspects of Maritime Radio
Communications, helping bridge crews to manage communications and comply with all reporting regulations
throughout a voyage?
● IAMSAR manual
● MERSAR manual
● ITU List of coast station
● ALRS manual

210. Which of the following 3 letter signal means “Apply hot compress and renew every 4 hours?
● MRQ 4
● MRP 4
● MRR 4
● MRS 4

211. What is false regarding the Inmarsat L-band EPIRB?


● It will be activated when the ship sinks
● Minimum battery capacity is 48 hours
● Instant alerting
● Global coverage

212. What is the major advantage of the NAVTEX system when compared to other system?
● A low frequency band is used for long distance transmission
● The information can be received on an ordinary FM radio
● Broadcast are at scheduled times
● Warnings are printed out for reading when convenient

213. When maintaining lead acid batteries, how would you show your understanding the importance of
protective clothing should be worn?
● Because a corrosive electrolyte is use
● Because there is risk of electric shock
● Because a volatile gasses may be emitted
● Because the battery compartment may be in a dangerous location

214. What does PSDN stands for?


● Personal Service Data Network
● Pacific Services Data Network
● Public Search Data Network
● Public Switched Data Network

215. What is the primary equipment for receiving MSI?


● SART
● EPIRB
● NAVTEX
● INMARSAT-A
216. What is the meaning of the acronym MSI?
● Maritime Service Information
● Maritime Safety Identification
● Maritime Services Identification
● Maritime Safety information

217. How would you describe DSC equipment operation on VHF Ch.70 should be tested?
● Daily by means of a test call with a coast station
● Weekly by means of an Internal test
● Weekly by means of a test call with a coast station
● Daily by means of an internal test

218. Reference to Radio Regulation Article 5 Section 1 Region and Areas. What can you say about
allocation of frequencies the world has been divided into how many regions?
● Three (3)
● Five (5)
● Two (2)
● Four (4)
219. How many pieces of portable GMDSS VHF transceivers must be provided for a ship of 450 gross
tonnage?
● 4
● 3
● 2
● 5

220. Which of the following statements concerning satellite EPIRBs is True?


● Once activated these EPIRBs transmit a signal for use in identifying the vessel and for
determining the position of the beacon
● If the GMDSS Radio Operator does not program the EPIRB, it will transmit default information
such as follow-on communication frequency and mode
● The coded signal identifies the nature of the Distress situation
The coded signal only identifies the vessel’s name and port of registry

221. What does MCC stand for?


● Mission Conduct Center
● Mission Control Center
● Medium Control Center
● Mobile Control Center

222. In chapter 3 of the International Code of Signals, Section 2: Request for Medical Assistance, which
code should your message means “ I need a Doctor, I have radiation casualties”?
● BT 2
● BR 3
● AN 2
● AN 1

223. Which of the following stand for SDR stand?


● Search Data Region
● Special Data Rights
● Special Drawing Rights
● Ship Data Rights

224. Which of the following is designated as the International distress safety and calling frequency for radio
telephony for stations of the maritime mobile service when using frequencies in the authorized bands
between 156 and 174 MHz?
● 168.5 Mhz
● 156.8 Mhz
● 158.6 Mhz
● 165.8 Mhz

225. Which of the following situations would normally use the safety priority
● Man overboard
● A scenario concerning an important navigational warning
● Engine failure in congest water
● Steering gear failure in approaching harbour

226. The SART operates on 9 GHZ. What radar band do frequency belong to?
● Z-band
● B-band
● X-band
● A-band

227. How would you demonstrate an on-load test of the radio batteries while operating the transmitter on full
power and with the battery charger?
● on while observing the discharge current
● off while observing discharge current
● on while observing the battery voltmeter
● off while observing the battery voltmeter

228. What can you say about the GMDSS function performed by a NAVTEX receiver?
● NAVTEX transmit MSI
● NAVTEX received position
● Navtex received MSI
● Navtex give position report

229. What can you say about the GMDSSS function performed by a NAVTEX receiver?
● NAVTEX received position
● Navtex received MSI
● Navtex transmit MSI
● Navtex give position report

230. What is the principal advantage of NAVTEX radio warnings?


● They cover broad spectrum of radio band allowing reception on almost any type of receiver
● They can be used by mariners who do not know Morse code
● FM radio is necessary to receive these warnings
● Information on a given topic is only broadcast at specified times

231. How would you compare the frequency range for Medium Frequency?
● From 10-30 kHz
● From 1000-10000 MHz
● From 300-3000 kHz
● From 30-300 kHz

232. The AIS-SART should be capable of transmitting message of a unit in distress except:
● Static information
● Position
● Type of Ship
● Safety information
233. In which IMO Convention does the guidelines and criteria for Ship Reporting System are associated
with?
● ISM Code Chapter 9
● SOLAS regulation V/II
● Marpol Annex 3
● STCW Table A-IV/2

234. In observing the secrecy of communication which of the following statement is not correct?
● The operator could not divulge the contents of the messages received to unathorized people
● The operator could divulge the contents of the messages received to unathorized people
● The operator could divulge the contents of the messages received to chief cook
● The operator could divulge the contents of the messages received to the master of the vessel

235. How can rescue personnel detect that a SART is transmitting in the immediate vicinity?
● The DSC unit will react to the SART signal and respond with the two-tone auto alarm
● The SART signal appears as a target which comes and goes the effect of heavy swells on a
SART
● The SART blips on the PPI will begin arcing and eventually become concentric Circles
● The SART can provide an approximate location to within a two nautical mile radius per IMO
Standards

236. In programming the NAVTEX receiver, what are the types of messages which are always received and
cannot be rejected?
● A,B,D
● A,C,D
● A,B,E
● A,B,C

237. Which of the following GMDSS equipment can be utilized in locating homing signals?
● Loran C
● SART
● EPIRB
● GPS Signal

238. What sort of equipment or device to be used as a homing signal by the search and rescue vessels in
the immediate vicinity of the ships in distress?
● 406 MHz signal from a satellite EPIRB
● A 121.5 MHz emergency transmitter in a satellite EPIRB
● Strobe light
● Flare Gun

239. What should a vessel trading in a A3 area fitted with MF/HF DSC must keep continuous watch on?
● On VHF Ch.70, 8414.5 kHz plus one other HF distress frequency only
● On VHF Ch.70 2187.5kHz, 8414.5 kHz plus one other HF distress frequency
● On VHF Ch.70 and 2187.5kHz only
● On VHF Ch.70 2187.5 kHz and 2174.5 kHz

240. What is specific gravity reading to considered fully discharge a lead acid battery?
● Between 1250 and 1150
● 1150 or lower
● Between 1250 and 1280
● 1150 or higher

241. Distance from earth of the F layer of the ionosphere?


● 160-400 kms
● 50-95 kms
● 95-130 kms
● None of the above

242. At anchorage your vessel is on fire what flag signal you should hoist to alert other ship?
● “M”
● “J”
● “K”
● “F”

243. What is the term refers to supply of electrical energy required to supply radio installations on every ship
for the purpose of conducting distress and safety radio communications in the event of failure of the ship’s
main and emergency sources of electrical power?
● Ship’s standby generator
● Reserve Source of power
● Ship\’s emergency diesel generator
● Emergency power

244. What should a master do if a technical failure prevents a ship from reporting in a Mandatory Ship
Reporting System?
● As long as you send rectification report when leaving the area covered by the system
● Obtain permission form charterer and note the failure in ship’s log
● Enter the fact and reasons for not reporting in the ship’s log
● Enter the fact and reasons for not reporting in the ship’s oil record book

245. To whom should the Master has to liaise with where the Radio Medical Advice suggests emergency
evacuation of the patient to shore medical facility?
● Designated Person ashore
● CIRM
● WHO International Medical Guide for ships
● Nearest Coastal state MRCC

246. What should you include in the call when making RT test signals for transmitter adjustment?
● The position of the station making the test
● An indication of the frequency to be used
● The identification of the station making the test
● An indication of the mode of emission being used

247. For MF-HF voice frequencies, what is the channel spacing?


● On a 3.0 kHz
● On a 500 Hz
● On a 3.5 kHz
● On a 2.8 kHz

248. What is meant by the term antenna bandwidth?


● The frequency range over which and antenna can be expected to perform well
● The angle between the half-power radiation points
● Antenna length divided by the number of elements
● The angle formed between two imaginary line drawn through the ends of the elements

249. What is the code that you can used for special inquiries such as ship location, authorization, etc?
● 31
● 33
● 32
● 36

250. Where NAVTEX cannot be feasibly established, what system can be implemented to provide an
automated service in coastal waters to receive MSI?
● General Information
● FleetNet
● Coastal Radio Information
● Safety Net

251. A GMDSS equipped ship has to send a distress call. The call sequence to be followed is
● Send DSC distress alert-Wait for acknowledgement-send the distress message
● Send the distress message alert
● Send DSC distress alert
● Wait for acknowledgement-send DSC distress alert-Send the distress message

252. Which from the following Service code would you choose when sending an OBS (weather report) with
Inmarsat?
● KNMI in Holland
● MET office Washington, this office will take care of further dispatch of the weather reports
● The meteorological station connected with the CES use
● Swedish meteorological station

253. What does IOR stands for?


● Indian Ocean Region
● Indian Organization Rules
● International Organization Rules
● International Ocean Region

254. What does FSK stands for?


● Frequency Service key
● Forward Service keying
● Frequency Shift Keying
● Fast Service Keying

255. Who shall refer proposals for the adoption of ship reporting system to the Organization?
● Marine reporting system
● Contracting Governments
● AMVER
● USCG

256. In chapter 2 of the international code of signals, section 1: Distress-Emergency, what is the meaning of
the code HM?
● Survivors are in bad condition. Medical Emergency is urgently required
● You should indicate position of survivors by throwing pyrotechnic signals
● Survivors are in good condition
● Have you located survivors? If so, in what position?

257. What is the communication necessary for the coordination of ships and aircraft participating in a search
and rescue resulting from a distress incident?
● On-Scene communication
● Rescue coordinating center
● Coordinator surface search
● SAR coordinating communications

258. In which frequency band does a search and rescue transponder operate?
● 9 GHz
● 406 MHz
● S-band
● 3 GHz

259. Reference SOLAS IV, Regulation 1, Every ship shall be equipped with NAVTEX and EPIRB not later
than?
● February 1, 1992
● August 1, 1993
● July 8, 1993
● November 9, 1991

260. In a lifeboat or liferaft, what is a method of maximizing the effectiveness of a SART?


● Hold or mount the unit as high as possible
● Move the switch to test position
● Switched ON the SART
● Throw the unit to the sea
261. What is the primary frequency range for terrestrial communications using skywave propagation?
● 10-30 MHz
● 300-3,000 MHz
● 3-30 MHz
● 30-300 MHz

262. Choose from the following service or maintenance functions may NOT be performed by the holder of a
GMDSS Radio operator License?
● Replacement of combustible items such as paper, ribbons, etc.
● Any adjustments or maintenance that may affect the proper operation of the station
● Routine battery maintenance if used as part of the GMDSS station
● Reset tripped circuit breakers or replace defective fuses

263. What would not, in itself, be considered as a failure to comply with the rules of a mandatory ship
reporting system?
● Not included in the Master’s night order
● Not included in ship’s SMS manual
● Failure of Ship’s main engine
● Failure of ship’s radiocommunication equipment

264. VHF-DSC radiotelephone installation must be tested at what minimum intervals at sea?
● Annually by the class
● Daily
● Monthly
● At the annual SOLAS inspection

265. What is a series of nine digits which are sent in digital form over a radio frequency channel in order to
uniquely identify ship stations, ship eart stations, coast stations, coast earth stations, and group calls?
● Maritime Mobile Service Identity (MMSI)
● IMO number
● Official Number
● Maritime Identification Digit (MID)

266. Choose from the following ITU list of service publication that contains the following I. International call
sign series II. Signals characterizing the emissions of radio beacon stations and list of call signs, used in the
maritime mobile service III. List of call signs and numerical identities assigned to pre-determined groups of
stations and list of former present call signs of ship stations IV. Numerical identities used in the maritime
mobile and maritime mobile-satellite services?
● ITU LIST VI List of Radiodetermination and Special Service Stations
● ITU LIST V List of Ship Stations and Maritime Mobile Service Identity Assignments
● ITU LIST VIIA LIST OF Call Signs and Numerical Identities
● ITU LIST IV Coast Stations/ List of Radiodeterminations and Special Services\

267. How would you describe the main purpose of transmissions on 2174.5 kHz?
● It is for NAVTEX English language broadcast
● It is for MF telex distress, urgency and safety messages
● It is for routine ship-to-ship telex messages on HF
● It is for ship-to-ship RT communications for the safety of navigation

268. With reference to Radio Regulation Chapter 1 Article Section IV 1.69. Which of the following is a
station in e Mobile service not intended to be used while in motion?
● Land station
● Base station
● Fixed station
● Coastal station

269. Which channel is designated for GMDSS digital selective calling?


● Channel 6
● 2182.0 KHZ
● Channel 70 (156.525 MHZ)
● 406 Mhz

270. The kind emission classes which can be used on 2182 KHZ is any of the following “except”?
● R3E
● G3E
● A3E
● J3E

271. Concerning the GMDSS radio operator requirements. Which from the following statement is FALSE?
● While at sea, adjustments to and the maintaining of GMDSS equipment may be performed by
the GMDSS radio operator as long as the work is supervised by an onboard license gmdss radio
operator as long as the work is supervised by an on board licensed GMDSS radio maintainer
● Each compulsory vessel must carry at least two licensed GMDSS radio operators at all times
while at sea and may elect to carry a GMDSS radio maintainer as well
● Each compulsory vessel must carry at least two licensed GMDSS radio operators at all times
while at sea.
● All communications involving safety of life at sea must be logged as long as the
compulsory vessel was not involved in such communications
272. Which of the following system below is least susceptible to fading or static interference?
● HF sitor
● Inmarsat
● VHF ARQ
● MF-HF DSC controller

273. How do VHF radio waves propagate through the atmosphere?


● Absorbed by the ionosphere
● Reflected by the ionosphere
● Diffracted like light
● Refracted from the ground

274. What is the mark or position at which a vessel is required to report to the local VTS station to establish
its position?
● Reference line
● Reporting point
● Cardinal Point
● Separation zone/line

275. What are the designated ambient temperatures for an AIS-SART to be able to operate?
● -20℃ to +55℉
● -5℃ to +65℃
● -20℃ to +55℃
● -40℉ to +1055℉

276. What should be the temperature range for an AIS-SART can be stowed throughout without damaging?
● -50℃ to +90℃
● -10℃ to +90℃
● -30℃ to +70℃
● -40℃ to +70℃

277. How long must the radio log be retained on board before sending it to the shoreside license?
● at least one year after the last entry
● at least 30 days after the last entry
● at least 90 days after the last entry
● at least two years after the last entry

278. What is the display you can observe on the radar screen when an activated SART is detected on a
distant range?
● 12 blips
● 9 blips
● 6 blips
● 36 blips
279. What does “PC” stands for?
● Personal control
● Public Computer
● Public Centre
● Personal computer

280. What is usually the FIRST step for a GMDSS radio operator's action when initiating a distress priority
message via inmarsat?
● by dialing the correct code on the telephone remote unit
● by pressing a “distress button” or “distress hot key(s)” on the equipment
● by contacting the CES operator using the radiotelephone distress procedure “mayday” etc.
● by contacting the CES operator and announcing a distress condition is in existence

281. In Navtex message editing, what is the code of the start of a message?
● ZCZC
● +?
● BRK?
● NNNN

282. Which of the following references should the GMDSS radio operator consult for information on the
proper operation of equipment?
● Information is available through Safety NET channels
● ITU list of equipment operations
● The manufacturers operator manuals
● 47 CFR part 80

283. What do you call the area in which a missing craft and/or survivor are most likely to be found taking into
account possible errors in the navigation of the missing craft and of the search craft?
● Possibility area
● Probability area
● search and rescue region
● search and rescue case

284. On whose authority should a DSC distress alert be sent?


● Master or person responsible for the vessel
● Radio operator
● Officer of the watch
● Person responsible for radio communications in an emergency

285. What single letter signal is used for the man overboard?
● S
● D
● O
● R

286. Which statement is NOT true regarding the COSPAS-SARSAT system?


● Doppler shift is used to locate the beacons
● EPIRBs are satellite beacons used as alerting/locating devices
● Maybe used to transmit public correspondence
● Locate distress beacons transmitting on 406mhz

287. What does AM stands for?


● amplitude mass
● amplitude modulation
● alternating modulation
● automatic modulation

288. With reference SOLAS IV, Regulation2, terms and definitions this is the coordinated broadcast
and automatic reception on 518kHz of maritime safety information by means of narrow band direct
printing telegraphy using english language. which from the following conform with the term and
definition of the regulation?
● NAVTEX
● EPIRB
● DSC
● WEATHER FAX

289. What does DNID stand for?


● Direct network identification code
● Data reporting network identification code
● Data reporting network identification code development
● Data network identification code

290. A NAVTEX message has been received on board with preamble ZCZCPB25. This message
contains a?
● navigational warning
● search and rescue information
● meteorological warning
● sar information

291. Which of the following stands for TDM?


● Time direct multiplex
● Transmitter division multiplex
● Time division multiplex
● Transport division multiplex

292. Reference SOLAS IV, Regulation 2, what is meant by an area excluding sea area A1, within the
radiotelephone coverage of at least one MF coast station in which continuous DSC alerting is
available?
● sea area A1
● sea area A2
● sea area A3
● sea area A4

293. In the VHF DSC, what is the channel used for both distress alerting, and calling for public
correspondence?
● channel 70
● channel 50
● channel 60
● channel 20

294. What is the most common type of antenna for GMDSS MF-HF?
● horizontally polarized whip antenna
● satellite radome with vertical polarization
● Horizontally polarized vertical whip antenna
● vertical polarized whip antenna

295. How would you describe a vertical quarter wave antenna with a good ground connection?
● It would not function due to being grounded
● It will radiate directionally due to being grounded
● It will only be used in satellite communications
● it will radiate omni-directionally

296. Which of the frequency is allocated as the GMDSS digital selective calling frequency?
● 2182 KHz
● 156.525 MHz
● 156.525 KHz
● 156.300 KHz

297. How would you describe the primary function of an MRCC within GMDSS?
● to provide a network coordination role for all LES/’s within the INMARSAT system
● to provide a public correspondence services to ships
● to provide a coordination role for search and rescue operation
● to provide a communication control function within the COSPAS-SARSAT system

298. Trading at sea area A1, how would you best describe as being an area within?
● it is within vhf RT range of at least one shore station providing continuous VHF DSC alerting
● it is within approximately 100-150 miles from the coast
● it is within MF RT range of at least one shore station providing continuous MF DSC alerting
● it is within MF RT range of an MF shore station

299. Choose from the following is a nonfunctional requirements for GMDSS compatible ships?
● transmit ship-to-ship distress alerting
● transmitting and receiving on-scene communications
● transmit locating signals
● transmit and receive on-scene communications,including appropriate SAR coordinating
communications

300. How would you explain the meaning of “reserve source of energy”?
● power to operate the radio installation and conduct distress and safety communications in the
event of failure of the ships main and emergency source of electrical power
● high caloric value items for lifeboat, per SOLAS regulations
● the diesel fueled emergency generator that supplies AC to the vessel’s emergency power bus
● diesel fuel stored for the purpose of operating the powered survival craft for a period equal to or
exceeding the SOLAS requirements

301. Your ship received a distress relay from a coast station on DSC frequency 2187.5 KHz. How
would the watch officer demonstrate his action?
● he should retransmit the DSC call on 2187.5 kHz to other vessels in the vicinity to assist in SAR
operations
● he should transmit a voice “mayday relay” call on 2182.0 Khz to other vessels in the vicinity
● he should transmit a voice “mayday relay” call on 2187.5 KhZ to other vessels in the vicinity
● he should monitor 2182.0 KHz to determine if there are any genuine distress communications

302. Your ship received a distress relay from a coast station on DSC VHF channel 70. How would the
watch officer demonstrate his action?
● he should monitor Ch.06 to determine if there are any genuine distress communication
● he should transmit a voice “mayday relay” call on Ch.70 to other vessels in the vicinity
● he should transmit a voice “mayday relay” call on Ch.13
● he should monitor Ch.16 to determine if there are any genuine distress communications

303. Ship reporting system should be considered for adoption by the organization only if supported by
a demonstrated need to address one or more of the following EXCEPT?
● To increase the protection of the marine environment
● The safety and efficiency of navigation
● The improvement of the safety of life at sea
● To promote cooperation among search and rescue

304. Which system provides maximum communication range?


● VHF ARQ
● Inmarsat
● MF SITOR
● DSC Calling on 8414.5 KhZ

305. How would you transmit the prowords which mean “I HAVE NOT RECEIVED AND
UNDERSTOOD YOUR MESSAGE AND HAVE NOTHING MORE TO TRANSMIT” and reply is not
expected?
● Over and out
● Roger
● Roger, Out
● Over

306. How do you describe the broadcast transmission of NAVTEX?


● NAVTEX is transmitted by commercial coast radio stations following their traffic lists
● using FEC techniques
● No more often than every two hours and should immediately follow the radio telephone silent periods
● NAVTEX is transmitted only when an Urgency or Distress broadcast is warranted

307. Which of the following best describes a safety transmission?


● a man falls overboard
● all messages relative to the immediate assistance required by a ship, aircraft of other vehicle in
imminent danger
● a communication transmissions which indicates that a station is preparing to transmit an
important navigation or weather warning
● when giving position to AMVER

308. In the VHF band, what is the correct channel for making routine DSC alerts?
● Ch. 70
● Ch. 67
● Ch. 16
● Ch. 06

309. Choose from the following statement a compulsory vessel is NOT allowed to leave port?
● when the vessel is in an over carriage condition
● when the vessel is carrying only two licensed GMDSS radio operators and is capable of performing all
required functions
● when the vessel has replaced a required piece of GMDSS related equipment but its performance
has not been verified or logged
● when the vessel has arranged for both duplication of equipment and shore-based maintenance

310. What is the precautionary measures for the vessel regarding DSC relays of distress alerts?
● should be transmitted to all ships involved in distress traffic
● should be avoided, unless an acknowledgement has not been heard
● are the best means to provide for a re transmission of distress communications
● should be done for all distress alerts received aboard the ship

311. What are the highest priority communications from ships at sea?
● all critical message traffic authorized by the ship’s master’s
● navigation and meteorological warnings
● distress calls and communications preceded by the international urgency and safety signals
● authorized government communications for which priority right has been claimed
312. Identify what sea area if a vessel is engaged in local trade and at no points in its voyage travels
outside the range of a VHF shore station with continuous DSC alerting?
● sea area A1 and A2
● Inland and Coastal waters
● Coastal and International zones
● sea area A1

313. What is the responsibility of a compulsory GMDSS vessels?


● vessel must transmit a DSC distress relay upon receipt of a DSC distress alert
● vessels must immediately acknowledge all DSC distress alerts
● every vessels closest to a distress incident must render assistance
● only the vessels closest to a Distress incident must render assistance

314. What is the responsibility of vessels under GMDSS?


● vessels operating under gmdss, outside of areas effectively serviced by shoreside authorities, operating
in sea area A2, and A4 may be required to render assistance in distress situations
● vessels over 300 gross tonnage may be required to render assistance if such assistance does not
adversely affect their port schedule
● every ship is able to perform those communication functions which are essential for the safety
of the ship itself and of other ships
● only the vessels closest to a Distress incident must render assistance

315. Which of the following channels is designated as the VHF follow-on communications channel
and is required in all portable survival craft equipment?
● Channel 70
● Channel 16
● 8291 SSB
● 2182 KHz

316. Which of the following is the primary frequency that is used exclusively for NAVTEX broadcast
internationally?
● 518 khz
● VHF Channel 16 when a vessel is sailing in sea area A1, and a one and 2187.5 khz when in sea area
A2
● 2187. khz
● 4209.5 khz

317. The international telecommunication union facilitates the assignment of mids to countries. What
is the allocated MID number for the philippines?
● 533
● 574
● 548
● 567

318. How would you classify the mmsi number 0218740100?


● it is a coast station
● it is a ship earth station
● it is ship station
● it is group of ship stations

319. Need an antenna height of 15 meters, what is the approximate distance of the SART Operates
correctly when interrogated by the navigation radars?
● At least 1 nautical miles
● At least 5 nautical miles
● At least 2 nautical miles
● At least 3 nautical miles

320. How does a garbled character is printed in NAVTEX?


● Comma
● Colon
● Asterisk
● Period

321. What is the term that refers to a land station in a maritime mobile service?
● Maritime station
● Ship earth station
● Cost earth station
● Coast station

322. Choose from the following characters that are sent by the ship station when opening hf telex, to
indicate a desire to send a message to amver?
● MSG+
● AMV+
● AMV
● MSG

323. What is the frequency range for very high frequencies?


● 300-3000 khz
● 30-300 mhz
● 10-30 mhz
● 3-30 mhz

324. Which radio frequency/channels are reserved for emergency communication?


● 2168 khz vhf channel 8
● 2182 khz vhf channel 6
● 2128 khz vhf channel 16
● 2182 khz vhf channel 16
325. Your antenna tuner becomes totally inoperative. What would you do to obtain operation on 2 hf
bands?
● It is impossible to obtain operation on two different hf bands, Without an operating antenna tuner
● Bypass the antenna tuner use a straight whip or wire antenna approximately 30 feet long
● Without an operating antenna tuner transmission is impossible
● Bypass the antenna tuner and shorten the whip to 15 feet

326. What action will you carry out to regain operation on MF-HF frequencies if your MF-HF whip
antenna breaks off and is blown away in a storm?
● Rig a long wire antenna at least 60 meters long
● Rig a horizontal, center-fed dipole antenna 12 meters long
● Rig a wire antenna approximately 10-12 meters long as per the equipment instruction manual
● Rig a wire antenna 5-10 meters long from the antenna tuner to the highest vertical support

327. Where does the authority to make “standards for the operation of radio stations in the amateur
radio service” is derived from?
● The itu radio regulation
● The radio communication regulations
● The radio communication act
● The standards for the operation of radio station in the amateur radio service

328. Which action should the gmdss radio operator take in a distress situation when embarking in a
survival craft?
● Notify rcc rescue coordination center through vhf dsc with portable equipment
● Communicate via inmarsat c from the survival craft
● Switch on epirb and sart manually prior to launching and remain aboard vessel in distress
● Switch on epirb and sart immediately and leave on

329. Which of the following has been designated for on-scene communications in gmdss?
● 2187.5 khz and mf
● vhf tron air
● Channel 16 on vhf radio telephone
● Channel 70 dsc

330. Choose from the following statement is false?


● Key letters or abbreviations may be used in gmdss radio log books if their meaning is explain
● Distress communications heard do not require entries in the vessel did not participate in SAR
activity
● Urgency communications may need to be entered in the gmdss radio logbook
● Log entries of vhf safety broadcasts are not required

331. What advantage does a vertical we have over a long wire?


● Is radiates directionally for better propagation
● Is radiates more signal fore and aft
● It radiates equally well in all direction
● It radiates a strong signal vertically

332. Which of the following watches should a vessel compulsory maintain at sea area a1?
● a continuous dc watch on channel 70
● a continuous dsc watch on 2187.5 khz
● a continuous dsc watch and channel 16
● Continuous dsc watch and 8414.5 khz plus one other hf dsc frequency

333. Which maintenance functions can a gmdss radio operator perform?


● operator is responsible for ensuring that antennas are free of build-up soot and clear of
obstacles
● all levels of maintenance must be performed by a licensed gmdss radio maintainer
● The operator can make fine internal adjustments to the transmitter as long as the output power does
not change by more than 1%
● install an eprom in order to ensure the equipment continuous to operate within legal constraints

334. Upon receipt of the distress alert transmitted by use of digital selective calling dsc techniques,
what is the proper procedure to be followed?
● set watch under radiotelephone distress and safety frequency associated with the distress and
safety calling frequency on which the distress alert was received
● set a continuous watch on vhf fm channel 13,16 and dsc on channel 70
● set watch frequency in the band of frequencies the alert was received
● Shipstation equipped with narrow band direct printing equipments should respond to the distress alert
as soon as practicable by this means

335. Select from the following is the appropriate safety convention certificate?
● Itu list IV and list V appropriate safety convention certificate and itu manual for maritime mobile
stations
● appropriate safety convention certificate itu master plan of gmdss stations and itu manual for maritime
mobile stations
● itu master plan of gmdss coast stations, Itu manual for maritime mobile station, itu list IV and list V
● itu list IV and list V appropriate safety convention certificate and IMO manual for maritime mobile
stations

336. What kind of information can be achieved from a navtex?


● navigational warning
● satellite positions
● course and speed of own ship
● routes for sailing

337. Which of the following steps should be taken ,if possible, when the vessel must be abandoned
because of a distress situation?
● place the sart and epirb in the “on” position and secure them to the survival craft
● program the sart and epirb to transmit the vessels location and situation
● alert the coast guard by using the survival crafts portable inmarsat unit
● no additional steps are needed as the sart and epirb will both automatically float free and operate
properly

338. When did full equipment carriage under gmdss become mandatory?
● mandatory after february 1 1995
● mandatory after february 1 1992
● mandatory after february 10 1995
● mandatory after february 1 1996

339. A dsc distress call is received by your vessel in your transceiver frequency displays reads:
transmit= 4207.5 khz and received= 4207.5 khz with j3e emission - what information can you infer from
this?
● the dsc call requested voice on 4207.5 khz simplex but the requested alternate frequency is
improper
● the dsc call came in on 4 mhz dsc. you should set up your transmitter and respond on the appropriate
voice follow on frequency of 4177.5 khz
● The dsc controller decoded the requested voice frequency as 4207.5 khz simplex and your dsc
controller has automatically set up your transceiver and therefore the requested alternate frequency is
correct
● the dsc call came in on 4 mhz dsc. you should set up your transmitter and respond on the appropriate
voice follow on frequency of 4207.5 khz

340. What can you say about the following statement is true?
● that there are two tones used in j2b mode
● that there is only a single tone used in j2b mode
● that there is a carrier and two sidebands in h3e mode
● that there is an rf carrier is always required to carry the information

341. If medical evacuations are being considered, what should be waived for the benefits to both the
person needing assistance and to the rescued personnel?
● Against the inherent dangers of such operations
● The claim for cargo damages that may be held against the ship owner
● To maintain a high state of readiness against the possibility of damage in subsequent flooding
● Against partial loss of the subject injured, caused by apparel insured

342. What does gmdss stand for?


● Global maritime distress and safety system
● Global maritime disaster and safety services
● Global maritime distress and safety services
● General maritime distress and safety system

343. When would you check the battery voltage of the gmdss radio installation?
● everyday
● every week
● every Month
● every two weeks
344. Which from the following features is a non component of a 406 mhz satellite epirb?
● Aural locator signal
● float free release bracket
● emergency transmission on 406.025 mhz
● 121.5 mhz emergency home in transmitter

345. How many frequencies are available under gmdss for dsc distress related calls?
● 7
● 4
● 5
● 6

346. What is the joint international satellite aided sar system?


● cospas sarsat
● marad
● imo
● inmarsat

347. Which of the following schedules should be carried out for lead acid batteries to insure a full
specific gravity?
● Monthly
● once for international voyage
● weekly
● daily

348. What is the main function of transmissions on 406 mhz?


● it is for a epirb satellite position
● it is for mf telex distress,urgency and safety message
● it is for navtext english language broadcast
● it is for two-way rt communication with life raft during sar operations

349. Reference to radio regulation article 1 section 3 radio service. What is meant by a maritime
mobile service in or near a port, Between coast stations and ship stations, or between ship stations, In
which messages are restricted to those relating to the operational handling the movement and the
safety of ships and in emergency to the safety of persons?
● port operations service
● maritime radiation service
● broadcasting service
● ship movement

350. What is a valid mmsi number for a dsc call to a specific group of vessels?
● 030327931
● 334862941
● 003023791
● 384629410

351. On receipt of a dsc distress alert how would you demonstrate your action?
● Silence the alarm read the display screen and/or print out and listen for any follow on
transmissions
● Silence the alarm read the display screen and/or print out only call the master if the alert is within 500
nautical miles
● Read the display screen and/or print out silence the alarm always called the master to confirm the alert
is real
● Listen for any follow on voice/telex transmission on the appropriate dsc frequency

352. What is the code to obtain assistance to connect to a subscriber in a country where the service
provider is, in any country which does not have an international operator?
● 13
● 11
● 12
● 00

353. What is the code used for directly transmitting a message originated from mes to a selected
telegraph office for delivery by mail or other appropriate means inmarsat c only?
● 24
● 22
● 21
● 17

354. To ensure receipt of all relevant msi and text receiver should be turned on at least how many
hours prior to departure from port?
● 8
● 4
● 2
● 6

355. Which of the following log keeping statements is false?


● Entries of all company communications using gmdss satellite equipment are required
● Entries relating to pre voyage, pre-departure and daily tests are required
● Entries related to failures of compulsory equipment are required
● A summary of all distress communications heard and urgency communications affecting the stations
own ship

356. What does msc stands for?


● maritime safety committee
● maritime service committee
● maritime search committee
● mobile service committee
357. Ship reporting systems should be considered for adoption by the organization only if supported
by a demonstrated need to address one or more of the following except?
● to ensure protection of the seafarers right
● the improvement of safety of life at sea
● to increase the protection of the maritime environment
● the Safety and efficiency of navigation

358. What are the mandatory dsc watchkeeping bands/channels?


● 8 mhz hf dsc, 1 other hf dsc, 2 mhz mf, dsc and vhf channel 70
● Vhf channel 70, 2 mhz mf dsc, 6 mhz dsc and 1 other hf dsc
● Vhf channel 70, 2 mhz nf dsc, 4 mhz dsc and 8 mhz dsc
● 2 mhz mf dsc, 8 mhz dsc, vhf channel 16 and 1 other hf dsc

359. Which term describes the source of energy required to supply the gmdss console with power if
the ship source of main or emergency energy fails?
● Reserve source of energy
● Ships emergency diesel generator
● Ships standby generator
● Emergency power

360. What is the maritime radio system consisting of a series of coastation transmitting coastal
warnings?
● Navtex
● Safesea
● NavArea
● hydrant/hydropack

361. How many gmdss radio maintainers must be carried aboard a compulsory vessel if the at-sea
maintenance method is used?
● one regardless of sea area of operation
● two in sea area a3 and a4
● two in sea area 1 and a4
● two in sea area a1

362. Fire occurred on board your ship and you need assistance in sending a dsc message. How will
you categorize a message?
● distress
● safety
● routine
● precaution
363. What is the only international body for developing guidelines, criteria and regulations on an
international level for ship reporting systems?
● IMO
● BIMCO
● Classification societies
● The contracting government

364. What is the exchange of medical information and recommended treatment for sick or injured
persons where treatment cannot be administered directly by prescribing medical personnel?
● MEDICO
● medevac
● mass rescue operation
● telemedical assistance services

365. When the gmdss radio operator on watch hears securite spoken three times what information
can he expect to be received?
● Message concerning the safety of navigation or important meteorological warnings
● Message concerning maritime assistant
● Message concerning urgency involving vessels at sea
● Message concerning circumstances that threaten the lives and property at sea

366. How would you use the radiotelephone safety signal?


● securite repeated three times
● pan-pan repeated three times
● calling all stations repeated three times
● mayday repeated three times

367. In flag signaling what does a hoist or signal is said to be at the close up when it is hoisted?
● to the full extent of the halyards
● halfway to the mast
● both a and b
● about half of the full extent of the halyards

368. What is the mechanism that allows a float free epirb?


● hydrostatic release mechanism
● spring release mechanism
● air release mechanism
● hydraulic release mechanism

369. How many HF frequencies are available for DSC distress related costs?
● five
● four
● two
● one
370. What sea area is defined as being within range of a shore based MF station that provides for
continuous dsc alerting?
● sea area a2
● sea area a3
● sea area a1
● coastal waters

371. Select from the following frequency range for very high frequency ?
● 30-300 Mhz
● 300-3000 Khz
● 3-30 Mhz
● 10-30 Mhz

372. What type of station would be assigned the call sign YBRV2?
● Passenger ship
● Bulk carrier
● Navy landing craft
● Container ship

373. Which of the following statement support that a dipole antenna should be mounted as high as
possible?
● To improve reception of Navtex broadcasts
● To reduce damage from funnel emissions
● To increase the range of VHF transmissions
● To improve satellite synchronization

374. You observed a floating wreck of a small fishing boat, what priority of DSC call are you going to
send?
● Safety
● Distress
● Routine
● Urgency

375. How would you classify the MMSi number 259289000?


● It is group of ship stations
● It is a ship station
● It is ship earth station
● It is coast station

376. In Chapter 3 of the International Code of Signals, Section 2: Request for Medical Assistance,
which one should your message describing that the patient has been ill for 3 days?
● MAM 3
● MAK 3
● MAJ 3
● MAN 3

377. The print out of a Navtex self test shows a signal test pass but a noise test fail. What immediate
action should be taken to?
● Consult the fault finding guide in the operation manual
● Check receiver is set up to receiver station for your area
● Check the amplifier fuses
● Check that the receiver is set up to receive the correct message types

378. What is most common type of antenna for GMDSS MF-HF?


● Wire antenna
● Vertical whip
● TV antenna
● Coaxial cable antenna

379. What is an example of locating signal?


● Ship to shore transmission
● SSB phone traffic
● A float-free EPIRB
● Loran C

380. Identify which of the following MMSi numbers indicates a Philippine flag ship station?
● 548623300
● 430326890
● 257326819
● 33609991

381. At what point does a SART begin transmitting?


● It has been placed in the “on” position, it will respond when it has been interrogated by a 9 Ghz
radar signal
● It immediately begins radiating when place in the “on” position
● When it is held horizontally as high as possible
● It must be manually activated

382. What is the required GMDSS equipment which is the PRIMARY source of transmitting locating
signals?
● a SART transponds on 9GHz
● survival craft transceiver
● an epirb transmitting on 406mhz
● radio direction finder (RDF)

383. Where was the functional requirements for GMDSS compatible vessels (vessels to which the
SOLAS does not apply) are contained?
● In Schedule 3 of Marine Order 29 (Safety of Navigation and Radio equipment) 2016
● In Schedule 3 of Marine Order 28 (Safety of Navigation and Radio equipment) 2016
● In Schedule 3 of Marine Order 27 (Safety of Navigation and Radio equipment) 2016
● In Schedule 3 of Marine Order 26 (Safety of Navigation and Radio equipment) 2016

384. Where was the functional requirements for GMDSS compatible vessels (ships to which the
SOLAS apply) are contained?
● In chapter IV, regulation 2 of the solas convention
● In chapter IV, regulation 3 of the solas convention
● In chapter IV, regulation 4 of the solas convention
● In chapter IV, regulation 5 of the solas convention

385. How would you describe the signal transmitted in J2B mode?
● It is a full carrier and one sideband
● It has an upper sideband of 2 alternating tones
● It is a full carrier and two sidebands
● It has an upper sideband of a single tone switched on and off

386. Regarding the GMDSS requirement for ship sources of energy, which of the following statement
is FALSE?
● The reserve sources of energy and need to supply independent MF and HF radio installations at
the same time
● If a vessel’s position is constantly required for the proper performance of a GMDSS station, provisions
must be made to ensure position information is uninterrupted if the ship’s source of main emergency
energy fails
● At all times while the vessel is at sea, a sufficient supply of electrical energy to operate the radio
installations and charge any batteries which may be part of the reserve source of energy is required
● An uninterruptible power supply or other means of ensuring a continuous supply of electrical power to
all GMDSS equipment that could be affected by normal variations and interruption of ship’s power is
required

387. How would you describe which of the following defines “ITU Channel 1216”?
● It is Ch-1216 in the MF band
● It is Ch-12 in the 16 Mhz band
● It is Ch-12 in the 16 Khz band
● It is Ch-16 in the 12 Mhz band

388. What is the operational communication other than distress conducted by radio?
● General Communication
● Operational traffic communication
● Security related communication
● Public correspondence communication

389. Choose from the following service of maintenance functions that a holder of a GMDSS Radio
Operator License is prohibited to depart from any port?
● From any reset tripped circuit breakers or replace defective fuses.
● From any routine battery maintenance if used as part of the GMDSS station
● From any replacement of consumable items such as paper, ribbon, etc.
● From any adjustments or maintenance that may affect the proper operation of the station
390. The frequency of channel 70 which is exclusively used for DSC calling is?
● 156.526 Mhz
● 156.8 Mhz
● 156.550 Mhz
● 156.525 Mhz

391. Which of the following channel an operator may use, when conducting on-board VHF
communications?
● Ch.16 or 6
● Ch. 13 or 6
● Ch. 15 or 17
● Ch.12 or 13

392. What does ADE stands for?


● Above Deck Equipment
● Authority Development Equipment
● Above Development Equipment
● Automatic Deck Equipment

393. Distress traffic consists of the following EXCEPT for?


● nature of distress
● position
● time
● medical transport messages

394. Which of the following systems is not used in radio detection and ranging?
● frequency and modulation
● amplitude modulation
● pulse radar
● frequency shift

395. Who provides round the clock free radio-medical assistance to patients onboard ships flying all
over the world?
● CIRM
● CES operators
● MRCC
● WHO

396. How is NAVTEX receiver programmed to reject certain messages?


● By entering the Selcall of the transmitting station
● By pressing \”00\” in the transmitter’s ID block
● By selecting a message category’s Single fetter (A-Z) identifier
● The transmittings stations two digits stations, two digit identification can be entered to deselect
reception of its broadcasts
397. Which of the following statements describes the carrier?
● The carrier is a Radio Frequency (RF) signal that is modulated to carry intelligence
● There are always sidebands on either side of the carrier
● the carrier consists of at least 3 separate but closely spaced frequencies
● The carrier is used to modulate the information signal

398. To make a national ship-to-shore DSC alert, what will be your first choice of MF frequencies?
● Ship transmits on 2187.5 KHZ and receives on 2187.5 KHZ
● Ship transmits on 2177 KHZ and receives on 2177 KHZ
● Ship transmits on 2189.5 KHZ and receives on 2177 KHZ
● Ship transmits on 2182 KHZ and receives on 2187.5 KHZ

399. The sorting of MID assignments in numerical order reveals a regional structure, what is the first
digit assigned to Europe?
● 2
● 7
● 4
● 5

400. Which of the following statements concerning MMSI is true?


● All EPIRBs units that are used as alerting devices has an MMSI for easy identification
● These are satellite identification to an Earth-side Local User Terminal (LUT)
● All coast station MMSI number must be 9 digits and begin with 2 zeros followed by the MID
● It is the doppler frequency identification for measurement concept used to determine the EPIRB
location

401. NAVTEX transmission has a designed range of up to?


● 400 nautical miles
● 300 nautical miles
● 100 nautical miles
● 200 nautical miles

402. What does MES stands for?


● Mobile Emergency Station
● Mission Enhanced Services
● Mobile Earth Station
● Mobile Earth Services

403. What does NCS stands for?


● Network Coordinating Station
● Network Coordinating System
● Network Computer System
● Network Committee Services

404. With what system is a NAVTEX message transmitted?


● PLB
● SELFEC
● FEC
● ARQ

405. What is the name of the international convention that regulates GMDSS?
● SOLAS
● ITU
● STCW
● IMO

406. At mid-day, what would be the best choice in attempting to communicate with the shorestation
15 miles (24 km) distant?
● VHF band
● DSC
● MF
● HF

407. How does mutual interference among NAVTEX stations be avoided?


● Transmitter power is limited to that necessarily for coverage of assigned area
● Transmitter power are schedules to night time and daytime operation only
● Transmission by stations in each NAVAREA are arranged in a time-sharing basis
● Transmissions by stations are schedules to daytime and nighttime operation only

408. Reference SOLAS IV, Regulation 2, what term describe automated telegraphy techniques which
comply with the relevant recommendations of the international radio consultative committee?
● Direct Printing Telegraphy
● Direct Telephony
● DSC
● Direct Telephone and Telegraphy

409. Which EPIRB transmits a distress alert that is received and relayed by an INMARSAT satellite?
● L-Band EPIRBS
● Category I EPIRBS
● Class A EPIRBS
● Class B EPIRBS

410. If you are operating off the Atlantic coast of the United States and synched to the AOR-W
satellite. Which Inmarsat Earth Stations would your vessel utilize for INMARSAT-C Traffic?
● Southburry (USA), Burum (Netherland), Eik (Norway)
● Southburry (USA), Nakhoda (Russia), or Beijing (PRC)
● Southburry (USA), Santa Paula (USA), or Yamaguchi (Japan)
● Santa Paula (USA), Aussaguel (France), or Eik (Norway)

411. Which of the following is an operating method that the transmission is made alternately in each
direction of a telecommunication channel?
● Semi-duplex operation
● Simplex-operation
● Half-duplex operation
● Duplex-operation

412. Via a coast-station radio medical advice is asked using NBDP, what command has to be given
after ga+?
● med+
● 38+
● 41+
● pan+

413. Trading at GMDSS Sea Area A3, how would you best describe as being an area within?
● It is within 50 to 600 miles from the coast
● It is within coverage range of the Inmarsat geostationary satellite system
● It is within VHF RT range of at least one shore station providing continuous VHF DSC alerting
● It is within MF RT range of at least one shore station providing continuous MF DSC alerting

414. What does GFR stands for?


● Gold Franc
● Global Frequency Range
● Global Frequency Service
● Global FRequency Rate

415. Which from the following statement from a test should be carried out to check at least once per
month?
● that each printer has an adequate supply of paper
● each SART for signs of damage
● the reserve source of energy, when it is not on battery

416. In the international NAVTEX System, what NAVAREA covered Philippine waters?
● XI
● VI
● V
● IX

417. Choose from the following ITU List of Service Publication which were merged into a new Service
Publication entitled List of Ship Stations and Maritime Mobile Service Identity Assignments?
● ITU List VIIA
● ITU List V
● ITU List VI
● ITU List IV

418. What is this service publication prepared and issued, once a year by the international
telecommunication union (ITU), in accordance with the provision no.20.8 of the radio regulations (RR)?
● ITU List VIIA list of callsigns and numerical identities
● ITU List V of ship stations and maritime mobile service identity assignments
● ITU List VI list of radiodetermination and special service stations
● ITU List IV coast stations/ list of radio determinations and special service

419. Reference to Radio Regulation Article 1 Section VI- Characteristics of emissions and radio
equipment, what is meant by a single sideband emission in which the degree of carrier suppression
enables the carrier to be reconstituted and to be used for demodulation?
● full carrier single-sideband emission
● suppressed carrier single-sideband emission
● reduced carrier single-sideband emission
● single-sideband emission

420. How would you describe the features on GMDSS satellites EPIRB units?
● A strobe light, distres homing transmission on 406 MHz float-free release bracket
● A hydrostatic release, distress alert transmission on 121.5 MHz, a strobe light
● An emergency transmission on 406 MHz, hydrostatic release, AIS, homing frequency
● A float-free release bracket, strobe light & distress alert transmission on 406 MHz

421. What information is transmitted by a 406 MHz EPIRB alert?


● Nature of distress
● A unique Hexadecimal I.D. number
● Vessel position
● Vessel name and identification

422. A crewmember suddenly becomes blind in both eyes, which code should your message contain?
● MNM
● MNJ
● MNI
● MNO
423. Who provides a ready fleet of vessels to provide assistance, in place and on call, in remote
areas of the world areas?
● AMVER
● MRCC
● IAMSAR
● USCG

424. As stipulated in Appendix 16 to the RR, which of the following ITU List of Service Publication
shall be provided to all ships stations for a Global Maritime Distress and Safety System (GMDSS)
installation is required by International Agreement?
● ITU LIST VI AND VIIA
● ITU LIST VIIA
● ITU LIST VI
● ITU LIST IV AND V

425. What does KBPS stands for?


● Kilobyte per second
● Kilobyte per second - 1000 bits/second
● Kilohertz
● Kilobits per second - 1000 bits/second

426. Which of the following emission mode occupies the most bandwidth?
● the f1B
● the J3E
● the F3E
● the J2B

427. Choose from the following the required to maintain a watch on a vessel certified for service in
Sea Area A3?
● MF frequency 2187.5 kHz, on 8414.5 kHz and other HF DSC frequency, HF on 4125.0
● VHF channel 70, MF frequency 2182.0 kHz, HF on 8414.5 kHz and other HF DSC frequency
● VHF channel 16, VHF channel 70, MF frequency 218.5 kHz, HF on 8414.5 kHz and other HF 4177.5
MHz
● VHF channel 70, MF frequency 2187.5 kHz and other HF DSC frequency

428. Which of the following EPIRBs is most likely to be used to transmit a distress alert signal?
● Class A EPIRBs
● S-band EPIRBs
● 406 MHz EPIRBs
● 121.51.243 MHz EPIRBs
429. How would you explain why MF-HF transmission is required by antenna tuner?
● Because the antenna tuner calculates the proper spectrum band for the operator to use
● Because the antenna tuner indicates whether the ionosphere is ready to reflect a transmission properly
● Because it ensure the transmissions are restricted to legal marine frequencies
● Because the length of the physically-fixed antenna must be electrically matched to the intended
frequency of operation

430. Your vessel is expected to proceed to a port with a daylight saving time, which of the following
ALRS Volume you have to consult?
● Volume 1 (NP281)
● Volume 2 (NP282)
● Volume 4 (NP284)
● Volume 3 (NP283)

431. What are the applications of paired frequencies in general communications?


● Normally used between private coast stations and ship stations
● Normally used with private coast stations
● Normally used with public stations
● Normally used between ship stations

432. Your ship received a distress relay DSC freq. 2187.5 kHz. You would acknowledge by radio
telephony on what frequency?
● 2182.0 kHz
● 8290.0 kHz
● 4207.5 kHz
● 6312.0 kHz

433. A ship makes a Routine manual DSC ship-to-shore DSC Alert but receives no response. When
should be the alert repeated?
● At 2 minutes interval
● After 2 minutes with the next alert 3 minutes later
● At 5 minutes interval
● After 5 minutes with the next alert 15 minutes later

434. What action should you take after sending a false distress alert on MF?
● make a voice announcement to cancel the alert on 2187.5 kHz
● make a voice announcement to cancel the alert on 8291 kHz
● make a voice announcement to cancel the alert on CH 16
● make a voice announcement to cancel the alert on 2182 kHz

435. Which equipment on the bridge is used for locating a SART?


● COSPAS-SARSAT EPIRB
● VHF AIS
● Portable VHF
● 3cm (9Ghz) radar

436. If a vessel engaged in local trade, and at no point in its voyage travels outside 25 miles from the
Florida Coast, in which GMDSS zones would this vessel is operating?
● SEA AREA A4
● SEA AREA A3
● SEA AREA A2
● SEA AREA A1

437. What is the time interval for a distress alert from an Inmarsat-EPIRB is received in the coverage
area of a satellite by ground station?
● Within two minutes
● 30 to 60 minutes
● 60 to 90 minutes
● Within ten minutes

438. What is the VHF Channel used for communication between ships and aircraft for coordinated
SAR operation?
● Channel 1
● Channel 70
● Channel 13
● Channel 6

439. What is the responsibility of vessels under GMDSS?


● Vessel over 300 gross tonnage may be required to render assistance if such assistance does not
adversely affect their port schedule
● Every ship is able to perform those communication functions which are essential for the safety
of the ship itself and of other ships
● vessels operating under GMDSS, outside of areas effectively serviced by shoresided authorities,
operating in sea areas A2, and A4 may be required to render assistance in distress situations
● Only that vessel, regardless of size, closest to a vessel in distress, is required to render assistance
440. Where are the operational details of the stations transmitting maritime safety information to be
found?
● ITU list call sign and special numerical identities
● ITU list of ship station
● ITU list of radio determination and special service stations
● ITU list of coast station

441. Choose from the following the required to maintain watch on a vessel certified for service in Sea
Area A2?
● 2174.5 kHz
● 2738.0 kHz
● 2182.0 kHz
● 2187.5 kHz

442. What statement best describes demodulation?


● extracting intelligence from the radio carrier signal
● removing atmospheric noise from the signal
● detuning the receiver to remove interfering signals
● separating the TELEX signals from the voice signals

443. Can you describe modulation from the statement below?


● Charging mark-space 1 and 0
● Converting the carrier from a low frequency to a higher frequency
● Imposing intelligence onto a radio carrier signal
● Adjusting the frequency to the optimum band for long distance communications

444. Which of the following ALRS Volume that is split across eight publication
● Volume 3 (NP283)
● Volume 6 (NP286)
● Volume 4 (NP284)
● Volume 5 (NP285)

445. In Ship Reporting System, if practicable, what should shore-based authorities considers to
reduce ship reporting burden?
● Navigational assistance service to assist onboard navigational decision-making and to monitor its effect
● Automated electronic operated means of the ship reporting recognized by the organization
● Manually operated means of ship reporting recognized by the organization
● A 4-digit group latitudes in degrees and minutes and a 5-digit group giving longitudes in degrees and
minutes
446. When in transpond mode, the SAR battery should last up to?
● 8 hrs
● 48 hrs
● 24 hrs
● 96 hrs

447. What does the message 4 in the DSC distress call contains?
● Nature of distress
● Time indication (UTC) when the coordinates were valid
● Single character to indicate the type of communication which is preferred by the station in
distress for subsequent exchange of distress traffic or SUBSEQUENT COMMUNICATION
● Distress coordinated message

448. How would you classify from the following is commonly part of 406 MHZ satellite EPIRBs?
● Automatic hydrostatic release (ARM), 1-watt 121.5 MHz alerting beacon, strobe light
● A strobe light, automatic float-free bracket, 1 watt 406 MHZ alert beacon
● Automatic float-free bracket, 5-watt 121.5 MHz homing beacon, strobe light
● A 5 watt 406 MHz alert beacon, automatic hydrostatic release (ARM), strobe light

449. Choose from the following control selections may result in limited receiving range?
● The setting of the channel selection which midway between channels 6 and 16
● The setting of the squelch control to its minimum level
● The setting of the power switch is set to the \”high”\ output position, resulting in receiver overloading
● The setting of the squelch control to its maximum level

450. What does a NAVTEX receiver do when it runs out paper?


● The system will automatically change from receiving MSI by NAVTEX to receiving it by SafetyNETTM
so that no message will be lost
● Listen to appropriate VHF weather channel for NAVTEX messages that was not printed
● It will give either an audible and/or visual alarm
● The unit cannot operate, and all subsequent MSI broadcast are missed until paper is replaced

451. What does NAVTEX stands for?


● Narrow Telex
● Navigational Telephony
● Navigational Telex
● Network Telex
452. What is meant by the term antenna efficiency?
● Efficiency= radiation resistance / total resistance x 100%
● Efficiency= radiation resistance / transmission resistance
● Efficiency= total resistance / radiation resistance x 100%
● Efficiency= effective radiated power / transmitter output x 100 %

453. Some LESOs have direct connections with local hospitals for used with this code. What is this
code to obtain medical advice?
● 33
● 37
● 32
● 38

454. Which from the following address would you select if you wish to transmit a weather report with
an Inmarsat-C Terminal?
● OBS+
● 32
● 41
● Meteorological center

455. What does DC stands for?


● Data current
● Direct current
● Data code
● data circuit

456. How would you best describe the following the GMDSS Sea Area A4?
● An area within 70 degrees South
● A continuous global coverage of the COSPAS-SARSAT system
● An MF RT range of at least one shore station having continuous MF DSC
● An area within 70 degrees - 90 degrees North and 70 degrees - 90 degrees South

457. When an external MF DSC test is carried out, a fault condition is displayed on the transmitter.
What should be the immediate action to be taken?
● Consult the fault finding guide in the operation manual
● Carry out an internal DSC test as per the operational manual
● Check the location of MF transmitter fuses in the operational manual
● Isolate the antenna and retest

458. What is the standard phrase to indicate that you have completed your transmission and you are
ready to receive?
● roger, out
● over
● roger
● over and out

459. What is the first three digit refers to when ship has an MMSI 25789000?
● the nationality of the ship
● company name of the ship
● the route of the ship
● the name of the ship

460. Who may be contacted for the ship reporting systems information on the availability of ships with
medical officer?
● Aircraft co-ordinator (ACO)
● IAMSAR
● Coast Radio Station (CRS)
● Rescue Coordination Center (RCC)

461. How would you describe the first symbol in the designation of radio emission under the ITU
rules?
● It is a bandwidth
● It is a nature of signals modulating the main carrier
● It is a type of information to be transmitted
● It is a type of modulation of the main carrier

462. What is the fundamental principle for imposing radio silence?


● To ensure that interference to proprietary communications is minimized
● To ensure that only voice communications can be effected on the distress frequency channel
● To ensure that a distressed vessel will have a “window” twice each hour for transmitting routine
messages
● To mitigate the risk of interference on a frequency or channel being used for emergency
communications

463. How do you call the communication between ship stations and shore-based communication
network which concern the management and operation of the ship may have an impact on its safety,
E.G. orders for pilot and tug services, charts replacements, repairs, etc.?
● Navigational Communications
● On-Scene Communication
● General Radio Communications
● Short Range Communication
464. Who shall comply with the requirements of adopted reporting systems and report to the
authority all information required in accordance with the provisions of each such system?
● VTIS
● Governments or governments concerned
● The master of a ship
● IMO

465. When a vessel rendering medical assistance to another distressed vessel or aircraft, the
following equipment must be ready for possible use except?
● rescue litter
● clothing
● medical supplies and medicines
● shelter

466. How would you show from the following frequencies would the J3E emission be used for gmdss
distress communications?
● on 4125.0 kHz, 16420.0 kHz, or 4177.5 kHz
● on 8414.5 kHz, 8291.0 kHz or 8376.5 kHz
● on 2182 kHz, 2187.5 kHz, or 8414.5 kHz
● on 2182.0 kHz,16420.0 kHz or 8291.0 kHz

467. What is the term for a modulation in which no signal is present between pulses?
● pulse modulation
● FSK
● QAM
● PSK

468. What are an ITU channels?


● International calling frequencies
● An international designation of specific frequencies
● International Telecommunication Union
● International Ships assigned working frequencies

469. If a vessel is operating 100 nautical miles from shore within range of a shore based MF station,
the vessel is operating within what area?
● Coastal waters
● Navigational Waters
● Sea Area A4
● Sea Area A2

470. Which statement is not true regarding the SART?


● transmits a distinctive code or easy recognition
● responds to interrogations by a vessels x-band radar
● transmits on the 9 Ghz band reserved for navigational radar
● Operates in conjunction with a vessels S-band radar

471. What is the required GMDSS equipment which is the primary source of transmitting locating
signals?
● An EPIRB transmitting on 406 MHz
● Radio Direction Finder (RDF)
● A SART transponders on 9 Ghz
● Survival Craft Transceiver

472. What is the term for listing of the date and time events, programs, equipment, test malfunctions
and correction in communication system?
● Reporting
● Log
● File
● Documentation

473. What does the message 2 in the DSC distress call contains?
● Nature of distress
● Time indication (UTC) when the coordinates were valid
● Single character to indicate the type of communication which is preferred by the station in distress for
subsequent exchange of distress traffic
● Distress coordinate message

474. How would you describe the importance of ventilating a ship’s battery locker?
● to allow oxygen into the locker to assist charging
● to reduce the risk of explosion
● to keep the cell tops dry
● to prevent sulphation on the terminals

475. Which of the following address would you select if you wish to transmit a weather report with an
Inmarsat-C terminal?
● Meteorological center
● 41
● OBS+
● 32

476. What is the medical service permanently staffed by doctors experienced in conducting remote
consultations and aware of the particular nature of treatment on board ships?
● Medivac
● Interco
● Medico
● TMAS

477. How would you describe the electrolyte of a lead acid battery?
● It is a concentrated sulphuric acid
● It is a dilute hydrochloric acid
● It is a dilute sulphuric acid
● It is a concentrated hydrochloric acid

478. What would be an indication of a malfunction on a GMDSS station with a 24 VDC battery
system?
● After testing the station on battery power, a voltmeter reading of 30 volts for a brief period followed by a
steady 26 volt reading
● A constant 30 volt reading on the GMDSS console voltmeter
● All of these symptoms would indicate a potential battery charger malfunction
● After testing the station on battery power, the ammeter reading indicates a high rate of charge that then
declines

479. Which of the following statements is describing a fixed RADIOTELEPHONE?


● A fixed unit that could received meteorological and navigational warning
● A units operate in the routine mode which is capable of telegraphic transmission
● Fixed units operate in the routine mode with RF power level no greater than 1 watt
● Fixed units that could send and received radio bearing signals operating in greater wattage

480. Which of the following type of equipments does not involve satellite detection?
● Class A EPIRB
● 121.5/406 MHz EPIRB
● SART
● L-Band EPIRB

481. What does ARQ stands for?


● Automatic Retransmission Request
● Automatic Region Request
● Automatic Request Retransmission
● Atlantic Region Request

482. Once the vessel is berthed and will stay for several weeks, choose from the following action to
be taken?
● The Inmarsat-C system can be powered down without taking additional steps once the GMDSS Radio
Operator has ensured that all incoming Safety NETTM messages have been received and stored
● The GMDSS Radio Operator may log out of the Inmarsat-C system and turn the power off
(unless the vessel decided to leave the unit on during the port stay)
● The GMDSS radio Operator must transmit an all-ships alert, to notify vessels within the satellites
footprint that the vessel will be off-line
● The GMDSS radio Operator must notify the NCS that the vessel will be off-line, and wait for the NCS to
acknowledge with a confirmation number that must be logged

483. In accordance with relevant recommendations, which international body is responsible for the
technical characteristics of AIS-SART?
● ITU
● ICAO
● SOLAS
● IMO

484. Which of the following ALRS volume includes radio details for: I.Listing of VHF Radio Direction-
Finding Stations II. Radar Beacons (Racons and Ramarks) III.Known Operational Automatic
Identification System (AIS) IV.Aids to Navigation (AtoN) V. radio beacons transmitting DGPS
corrections VI.International Standard and Daylight Saving Times and Dates VII.International Radio
Time Signal Broadcast Details
● Volume 3 (NP283)
● Volume 2 (NP281)
● Volume 5 (NP285)
● Volume 2 (NP282)

485. What is the most appropriate action for a GMDSS Radio operator to take in a distress situation
where immediate help is needed, but the vessel is not sinking nor needs to be abandoned?
● Notify the RCC through VHF FM on Ch 13
● Transmit distress call by HF/MF/VHF DSC or Inmarsat
● transmit distress call by activating the radiotelegraph automatic alarm signal
● Switch off EPIRB and SART manually

486. A NAVTEX message has the preamble ZCZTD00. What type of message is this?
● Meteorological Warning
● Search and Rescue information
● Navigational warning
● SAR information

487. What is the IMO resolution adopted on 16 June 2017 that concerns with the guidelines and
criteria for Ship Reporting System?
● Resolution MSC.333(98)
● Resolution MSC.435(98)
● Resolution MSC.433(98)
● Resolution MSC.329(98)

488. How long should reserve power source must be able to power all radio equipment plus an
emergency light system?
● 24 hrs
● 6 hrs
● 12 hrs
● 8 hrs

489. Reference to Radio Regulation Article 1 Section IV Radio stations and system. What is meant by
a station located either on the earth's surface or within the major portion of the earth’s atmosphere and
intended for communication?
● land earth station
● space station
● terrestrial station
● earth station

490. Once activated, how is the signal of 406 MHz COSPAS-SARSAT is detected?
● Detected within half hour by the satellites concerned
● Always immediately detected by the satellites concerned
● Only detected and directly passed on when an LUT is within the reach of the satellite
● Detected within one and a half hours by the satellites concerned

491. Identify from the following frequency range for Super High Frequency?
● 300-3000 MHz
● 3-30 GHz
● 30-300 MHz
● 30-300 GHz
492. If your vessel is in the Southern Pacific Ocean near the Dateline, which of the following LES
should you select to update a distress alert message?
● Yamaguchi (Japan) or Nakhoda (Russia) would be the best choice depending on SAR jurisdiction
● Auckland (New Zealand) would be the best choice depending on SAR jurisdiction
● Beijing (PRC) would be the best choice depending on SAR jurisdiction
● Kumsan (South Korea) or Sentosa (Singapore) would be the best choice depending on SAR jurisdiction

493. Your ship is participating in the AMVER reporting system, which of the following report your
arrival at your destination?
● AMVER/AR//
● AMVER/DR//
● AMVER/PR//
● AMVER/FR//
494. Which Inmarsat Systems Special Access Code (SAC) can be used to obtain medical advice or
medical assistance?
● SAC 41
● SAC 43
● SAC 32
● SAC 39

495. What does UTC stands for?


● Universal Transport Committee
● Universal Time Committee
● User Time Coordinated
● Universal Time Coordinated

496. If not acknowledge automatically the repetition of a DSC Distress call, how would you expect the
extend of the delay?
● 10 - 15 minutes
● 1 - 2 minutes
● 2 - 5 minutes
● 3.5 - 4.5 minutes

497. How would you describe if a DSC Distress alert is not relayed?
● No distress traffic has been heard and the DSC alert is unacknowledge via DSC
● If the mobile unit in Distress is incapable of further Distress alert communications
● A coast station DSC acknowledgement of the original Distress alert was received by your
vessel.
● If no Coast Station/Mobile Unit acknowledgement of the alert is observed.

498. Vessel operating in which sea area(s) are required to carry either Inmarsat or HF equipment or a
combination thereof under GMDSS?
● Sea Area A1
● Sea Area A2
● Sea Area A3
● Sea Area A4
499. What is the code normally used by a LESO to allow automatic access to its information retrieval\
database
● 92
● 6(x)
● 91
● 70

500. What would be done on a DSC Relays of DSC Distress alerts received from other ships?
● Preferably by MF/HF voice or TELEX directly to the RCC conveying the follow-on Distress traffic that
has been heard
● Only by Inmarsat-C TELEX with Distress priority conveying the follow-on distress traffic that has been
heard.
● Only by Inmarsat-B voice or TELEX with Distress priority if no follow-on Distress traffic has been heard.
● Only when the original DSC call is not acknowledge and no follow-on Distress traffic has been
heard.

501. What is the approximate distance of E layer of the ionosphere from earth?
● None of the above
● 160-400 kms
● 95-130 kms
● 50-95 kms

502. What type of satellite system should be fitted on any ship intending to navigate in areas outside
of reliable coverage by a terrestrial hyperbolic system?
● GPS or GLONASS equivalent
● Mobile Satellite System
● Satellite Hyperbolic System
● Global Maritime Distress and Safety System

503. Which of the following is not a purpose of bridge-to-bridge communication?


● AMVER Reporting
● Security Communication
● Navigational Communications
● Safety of Navigation

504. What should a GMDSS Radio operator do if a NAVTEX warning message is received but it
contains too many errors to be usable?
● Press RP button and NAVEX messages will be reprinted instantaneously
● Contact the NAVAREA coordinator and request a repeat broadcast
● Listen to appropriate VHF weather channel for repeat warnings
● Do nothing. Vital NAVTEX messages will be repeated on the next scheduled broadcast

505. What entry would NOT be shown on the V line of an AMVER report?
● MD
● NONE
● NURSE
● MED TECH

506. VHF communication is sometimes referred to as?


● Ground wave propagation
● Short range communication
● Bridge-bridge communication
● Line of sight communication

507. How would you describe the precautionary measures for the vessel regarding DSC relays of
Distress alerts?
● Are the best means to provide for retransmission of Distress communications
● It should be avoided unless an acknowledgement has been heard
● It should be done for all Distress alerts received aboard the ship
● It should be transmitted to all ships involved in Distress traffic

508. What does MID stand for?


● Maritime Identification Digits
● Marine Indemnity Directory
● Mobile Identification Number
● Mobile Interference Digits

509. This code should be used if the condition of an ill or injured person aboard the vessel requires
urgent evacuation a shore or the services of a doctor aboard the vessel. What is this code that will
ensures the call is routed to the appropriate agency or authority ashore to deal with the situation?
● 38
● 37
● 36
● 33
510. The DSC controller is programmed to send out a distress alert. Which of the following is its other
usage?
● I can be used to communicate with private coast station
● It can be used to call individual station
● It should not be used for other purposes except for distress alerting
● It can be used to send messages to the owner and charterers
511. Which from the following statement a test should be carried out to check at least once per
month?
● the correct functioning of the NAVTEX receive
● each SART for signs of damage
● the reserve source of energy, when it is not on battery
● that each printer has an adequate supply of paper

512. Which of the following is the primarily used of DSC?


● Report ships position to Search and rescue authorities via satellite
● Transmit and receive distress and safety alerts to and from other Ships and Shore Stations via
radio
● Provide routine communications with the ship owner
● Receive weather warnings, navigational notices and other marine safety information

513. In which of the following situations would normally use urgent priority?
● Steering gear failure in open waters
● Engine failure in the middle of an ocean
● a serious medical situations involving crewmember
● Sighting of floating wreckage

514. What does IA5 stands for?


● International Alphabet(ASCII) 16-bit codes
● International Alphabet(ASCII) 7-bit codes
● International Alphabet(ASCII) 8-bit codes
● International Alphabet(ASCII) 36-bit codes

515. What does IA5 stands for?


● International Alphabet(ASCII) 16-bit codes
● International Alphabet(ASCII) 7-bit codes
● International Alphabet(ASCII) 8-bit codes
● International Alphabet(ASCII) 36-bit codes

516. What does ITA2 stands for?


● International Telegraph Alphabet 8-bit codes
● International Telegraph Alphabet 25-bit codes
● International Telegraph Alphabet 16-bit codes
● International Telegraph Alphabet 7-bit codes

517. Concerning COSPAS-SARSAT, which of the following statements is INCORRECT?


● EPIRB\’s, ELT\’s and PLB\s use the system primarily for Distress alerting
● This satellites monitor 406 MHz for EPIRB signals
● After initiating a call request and selecting the LES, these satellites may be used for commercial
messages
● These satellites use doppler shift measurement to determine the location of the beacons
518. What does USB stands for?
● Universal Services Band
● User Services Band
● Upper Side band
● Unity Side band

519. What does BPS stands for?


● Bits per hour
● Bits per services
● Bytes per services
● Bits per second

520. Reference SOLAS IV, Regulation 2, this means maritime global services identify the ship’s call
sign, Inmarsat identities and serial number identity which may be transmitted by the ship’s equipment
and used to identify the ship:
● NAVTEX
● AIS
● EPIRB
● Global Maritime Distress and Safety System (GMDSS)

521. How could the search vessel’s GMDSS radio operator interrogate a survival craft SART?
● The SART automatically responded when it is detected by the search crafts or other vessel’s on
X-band radar signal
● By switching ON the search vessels SART it responded automatically detecting other vessels SART
signals
● The SART signal is interrogated by the vessel AIS equipment and the bearing and the position can be
determine.
● By activating the emergency position-indicating radio beacon it will point to the direction of the SART.

522. Which of the following 3-letter signal means ‘’Do not give laxative or enema ‘’?
● MRU
● MRY
● MRP
● MRS

523. What does VHF stands for?


● Vast heavy Frequency
● Very hard Frequency
● Very Heavy Frequency
● Very High Frequency
524. Choose from the following communications functions should a GMDSS-equipped ships be
obliged to perform?
● Bridge-to-Bridge and general radio communications functions,RDF of EPIRB homing signals Distress
alerting and MSI.
● Distress alerting,MSI,SAR and on-scene communications and receipt of satellite alerts from other
vessels.
● SAR and on-scene communications,Bridge-to-Bridge and General radio communications ,MSI and
relay of satellite alerts from other vessels.
● Transmit distress alerts, SAR and on scene communications. MSI, Bridge-to-Bridge and general
radio communications.

525. What does AORE stands for?


● Atlantic Operator Region East
● Above Ocean Regions East
● Atlantic Operators Radio Emergency
● Atlantic Ocean Region East

526. Which is not a function of satellite under COSPAS-SARSAT using satellite EPIRB?
● Receiving signals from SART
● Information received from EPIRBS is time-tagged and transmitted to any Local User Terminal in the
Satellites view.
● Vessel information recovered from the digital encoded message provided by the Satellite EPIRB
● Doppler shift of EPIRB signal is measured

527. How would you compose your message if you wish to send an e-mail using Inmarsat -C
installation?
● As a FAX
● As a SMS
● As a TELEX
● In ASCII

528. What was the GMDSS- equipped vessel must carry if operating within Ocean area A1 and
outside of NAVTEX coverage?
● A GPS receiver
● Equipment capable of maintaining a continuous DSC watch on 2187.5 khz
● An Inmarsat -B terminal
● Equipment capable of reception of Maritime Safety Information by the Inmarsat enhanced group
call system,or HF SITOR(NBDP)

529. Which of the following is an Inmarsat terminals antenna?


● Whip antenna
● Directional antenna
● Omnidirectional antenna
● Vertical antenna
530. How many types are there in an AMVER reports?
● 8
● 3
● 4
● 6

531. What is the two letter signal which means “I AM ON FIRE”?


● “CI”
● “IT”
● “GB”
● “NC”

532. How do you cancel a false EPIRB distress alert?


● Transmit a DSC distress alert cancellation
● Make a radiotelephony “distress cancellation” transmission on 2182 kHz
● Notify the Coast Guard or Rescue Coordination Centre at once
● Transmit a broadcast message to “all stations” cancelling the distress message

533. What does AAIC stands for?


● Authority Accounting Identification Code
● Automatic Authority Identification Code
● Accounting Authority Identification Code
● Accounting Authority International Code

534. What immediate action to be taken when a fault condition is displayed on the transmitter during
MF DSC external test?
● Isolate the antenna and retest
● Check the location of MF transmitter fuses in the operational manual
● Carry out an Internal DSC test as per operational manual
● Consult the fault finding guide in the operational manual

535. Which of the following is the Radiotelephone Safety signal?:


● \”Securite\’ repeated 3 times
● \”Safety SafetySafety\”
● \”SecuriteSecurite\” repeated 3 times
● \”Pan Pan\” repeated 3 times

536. What indication is given to the personnel of survival craft of the approach of another vessel?
● The VHF portable will provide a visual indication
● The VHF portable radio will emit an audible alarm signal on 156.525 MHz
● The SART will provide a visual or audible indication of interrogation by a 3 CM radar
● The Satellite EPIRB will emit an audible signal
537. Mandatory installation of NAVTEX receiver and satellite EPIRB as additional Navigational AIds
Equipment of Ship Radio Stations engaged in coastwise trade, Pursuant to the provisions of (SOLAS
IV) and its amendments and relevant Philippine Radiocommunications Laws of the National
Communications Commission (NTC). When was the mandatory date of compliance?
● On or before 30 Feb 2002 for EPIRB and 30 June 2002 for NAVTEX
● On or before 30 Jan2002 for EPIRB and 30 Dec2002 for NAVTEX
● On or before 30 June 2002 for EPIRB and 30 Sept 2002 for NAVTEX
● On or before 30 July 2002 for EPIRB and 30 Sept 2002 for NAVTEX

538. For clarity and convenience of Admiralty List of Radio Signals (ALRS) with contents ranging from
Maritime Radio Station listings to Maritime Safety Information Services worldwide. How many volumes
it is split across?
● Seven(7)
● Five(5)
● Eight(8)
● Six(6)

539. What does “PSTN” stands for?


● Personal Services Telephone Network
● Public Switched Transport Network
● Public Switched Telephone Network
● Public Services Telegraph Network

540. Select from the following statement regarding the additional equipment carriage requirement
imposed for the survival craft of vessels over 500 gross tons is TRUE?
● A second radar transponder is required
● Four additional portable VHF radios are required
● Additional carriage of two radio equipped lifeboats aft
● The ability to communicate in all mode with any shore station

541. Which of the following maintenance functions is not the responsibility of the GMDSS Radio
Operator?
● visual inspection of equipment including the antenna and associated components
● Perform on-the-air verification checks
● Perform scheduled testing of the battery’s charged condition
● Aligning the power output

542. Reference SOLAS IV, Regulation 2, this means an area, outside Sea Area A1, A2, and A3.
Which from the following Sea Area conform with the term and definition of the regulation?
● Sea Area A4
● Sea Area A2
● Sea Area A1
● Sea Area A3

543. What is this code that will connect the caller to the radio-telegram service position for the
transmission of radio-telegrams originated via telex?
● 13
● 15
● 14
● 11

544. If the NAVTEX cannot be feasibly established. What system can be implemented to provide an
automated service in coastal waters to receive MSI?
● ARQ SITOR
● Safety/NET
● VHF DSC
● AMVER

545. What is the specific gravity reading to consider a fully charged lead acid battery in good
condition?
● 1080
● between 1100 and 1210
● between 1250 and 1280
● 1150

546. The establishment and operation of a ship reporting system should take the following into
account EXCEPT:
● Safety
● Pollution purposes
● Distress
● Urgency

547. Select from the following ALRS Volume that includes radio detail for: A. Detailed Pilot
information, contact details and procedures B. Vessel traffic Service Information, contact details and
procedures C. National and International Ship Reporting Systems D. Port information, contact details
and procedure
● Volume 6 (NP286)
● Volume 4 (NP284)
● Volume 2(NP282)
● Volume 5 (NP285)
548. What is the possible reason for an individual battery cell to reverse polarity?
● The discharge current will effectively charge the weeker cell
● The charging circuits are connected in the correct polarity but all of the cells are equally charge
● Insufficient charging which does no bring all of the cells up to full charge
● High discharge rates without allowing for a cool down period

549. What is valid MMSI number for a DSC call to a specific group of vessels?:
● 338462941
● 003664512
● 3036483
● 030327931
550. VHF Channel 6 is exclusively for what kind of communication?
● Radio direction finding
● Radio checks and ime checks
● Working with helicopters
● Inter-vessel safety and search and rescue

551. What does the abbreviation MASTREP means?


● Modernised Aruba Ship Tracking and Reporting System
● Mid-Argeninian Ship Tracking and Reporting System
● Modernised American Ship Tracking and Reporting System
● Modernised Australian Ship Tracking and Reporting System

552. The ITU that facilitates the assigned MID numbers to country, how many digits are the Maritime
Identification Digits(MID)?
● 5
● 6
● 7
● 3

553. What is the communications between the ship in distress and assisting units which relate to the
provision of assistance to the ship or rescue of survivors?
● On-Scene Communications
● Navigational Communications
● Short range Communication
● General Radio Communications

554. How would you best describe from the following the GMDSS Sea Area A4?
● An area within 70 North- 70* South
● An MF RT range of at least one shore station having continuous MF DSC
● A continuous global coverage of the COSPAT-SARSAT system
● An area within 70*-90* North and 70*-90* South

555. What does DCE stands for?


● Data Circuit Termination Equipment
● Direct Circuit Termination Equipment
● Direct Circuit Equipment
● Direct Circuit Equipment

556. choose from the following ALRS Volume that includes information for: A. Worldwide
communication requirements for distress,search and rescue B. Extract from SOLAS and ITU
Regulations C. Distress and SAR (incorporating MRCC and MRSC contacts) D. Worldwide NAVTEX
and Maritime Safety Information
● Volume 4 (NP284)
● Volume 3(NP293)
● Volume 5(NP285)
● Volume 6 (NP286)
557. Which of the following systems is most susceptible to fading or static interference?
● VHF ARQ
● HF SITOR (NBDP)
● Inmarsat
● DSC on channel 70

558. What is the IAMSAR procedural word that indicates,’’ I must pause for few seconds,standby for
further transmissions’’?
● Negative
● Wait
● Affirmative
● Break

559. What does the abbreviation of MID mean?


● Mobile Interference Digits
● Marine Indemnity Directory
● Maritime Identification Digits
● Mobile Identification Number

560. In Ship Reporting System, where language difficulties exist and in particular where requested by
the Master or the shore based authority, the following should be used to send detailed information
EXCEPT:
● AIS text messaging
● Where possible the Standard Marine Communication Phrases
● The languages used should be English
● Alternatively,the international Code of Signals

561. Which frequency must be used by a coast station to transmit a DSC international Routine shore-
to-ship alert?
● 2177 KHz
● 2182 KHz
● 2189.5 KHz
● 2187.5 KHz

562. Which statement concerning radio log archival by the station licensee is false?
● retain for three years if there are Distress entries
● Logs related to an investigation may not be destroyed without specific authorization
● Retain for one year unless there are Distress or Urgency entries
● Retain for two years if there are no distres entries

563. Choose from the following is nonfunctional requirements for GMDSS compliant ships?
● transmitting and receiving ship-to-ship distress alerts
● perform ship-to-shore distress alerting by two independent means
● transmitting ship-to-shore distress alerts by at least two separate and independent means, each using a
different radiocommunication service
● transmitting and receiving search and rescue coordinating communications

564. What is the Inmarsat special access code to be used to report a navigation hazard?
● 43
● 42
● 41
● 38
565. What is distress communication?
● An internationally recognized communication indicating that the sender is threatened by grave
and imminent danger and requests immediate assistance
● Communications indicating that the calling station has a very urgent message concerning safety
● An official radio communications notification of approaching navigational or meteorological hazards
● Radio communications which, if delayed, will adversely affect the safety of life or property

566. Which of the following is not consider a part of the radiotelephone distress message?
● Nature of distress and kind of assistance desired
● Particulars of its position, latitude and longitude, and other information which might facilitate rescue,
such as length, color and type of vessel, number of people on board
● Securities spoken three times, call sign and name of the vessel
● MAYDAY spoken three times, call sign and name of vessel in distress

567. What does HF stands for?


● High Frequency
● Hand Frequency
● High Fidelity
● Hard Frequency

568. What element of a DSC call describes the hour of the position and is given in UTC with the
aid of 4 digits?
● Message 4 of distress calls
● Message 3 of distress calls
● Message 1 of distress calls
● Message 2 of distress calls

569. In an instance, where a seafarer is contracted with a dreaded disease onboard, where a
Second Officer is acting as doctor, and he does not know what specific medicine to be given. What
category of urgency message will the Master send ashore?
● Radio Medico
● Urgency message
● Medical Assistance
● Medical advice message

570. What method to determine the condition of charge of a lead-acid battery?


● measuring the voltage off load
● measuring the temperature of the electrolyte
● measuring maximum current under load
● measuring the specific gravity of the electrolyte

571. Select from the following references should be consulted to identify the name of a vessel based
on its Maritime Mobile Service Identity?
● ITU List of Coast Stations
● ITU List of Ship Stations and Maritime Mobile Service Identity Assignments
● ITU Master Plan of Shore Based Facilities
● ITU List of Radio determination and Ship stations

572. Which of the following conditions that would be a symptoms of malfunction in a 2182 kHz
radiotelephone system?
● If no indication of power output when speaking into the microphone
● A much lower noise level observed during daytime operation
● When testing a radiotelephone alarm on 2182 kHz into an artificial antenna, the Distress frequency
watch receiver become unmuted an improper testing procedure
● Failure to contact a shore station 600 nautical miles distance during daytime operation

573. What is meant a maritime global services that identify the ship’s call sign, Inmarsat identifies
and serial number which may be transmitted by the ship’s equipment according to SOLAS IV
Regulation 2
● Global Maritime Distress and Safety System (GMDSS)
● EPIRB
● AIS
● NAVTEX

574. Choose from the following system has the least effective radius of operation?
● NAVTEX
● VHF DSC
● MF SITOR (NBDP)
● HF SITOR(NBDP)

575. The MMSI is used in all of the following DSC calls ‘’EXCEPT’’?
● distress calls and all ships call
● Safety alert and ships call
● Routine alert and ships call
● Urgency alert and all ships call

576. Which equipment on the bridge is used for locating a sart?


● VHF AIS
● Portable VHF
● COSPAT-SARSAT EPIRB
● 3 cm (9Ghz) radar
577. Your vessel ‘’JERINEB’’ is being called, but you are unsure of the identity of the station.What
action you must proceed?
● wait one minute, then reply using ,all stations who is calling JERINEB
● wait two minutes , then reply using /’’Station who is calling JERINEB,repeat you call/’’.
● reply immediately using /’’ Station calling JERINEB,repeat your call/’’
● wait for the call to be repeated

578. What do you call the time it takes for radio wave to move from zero to 360 degrees?
● It is called lambda
● It is called peak
● it is called comma
● it is called period

579. What is the main purpose of VTS?


● Provide information
● Increase safety for all ships in the area
● Increase safety for persons participating in the area
● Reduce port operation costs

580. Which of the following message Categories can be disabled by the GMDSS Radio Operator?
● Search and Rescue information
● Meteorological warnings
● Navigational Warnings
● Ship to ship correspondence

581. If a ship sinks, what device is designed to float free of the mother ship, is turned on automatically
and transmits a distress signal?
● Auto alarm keyer on any frequency
● EPIRB on 121.5 MHz/243 MHz or 406.025 MHz
● Bridge-to Bridge transmitter on 2182 kHz
● EPIRB on 212 kHz and 405.025 kHz

582. The SART is required to have sufficient battery capacity to operate in the stand-by mode for
what period of time?
● Three days
● Eight hours
● Forty-eight hours
● Four days

583. What is the normal schedule for checking and testing of EPIRB?
● monthly
● daily
● weekly
● annually
584. Which of the following ALRS Volume that includes radio details for: I. Global Maritime
Communications II. Satellite Communication Services III. Coastguard Communications IV. Maritime
Telemedical Assistance Service(MTAS) V. Radio Quarantine and Pollution reports VI. Anti-Piracy
Contact Table
● Volume 4 (NP284)
● Volume 3 (NP283)
● Volume 1 (NP281)
● Volume 5 (NP285)

585. What is defined as the area within the radiotelephone Coverage area with a continuous DSC
alerting available as defined by the IMO regulation for GMDSS?
● Sea Area A1
● Sea Area A3
● sea Area A2
● Sea Area A4

586. In the international Code of Signal, Chapter 2, Table M-3, List of Medicaments, which injection is
to be used only on Medical Advice by radio except in case of Anaphylactic shock due to penicillin
injection?
● Tetracycline Injection(100 mg per ampule)
● Streptomycin Injection (1000 mg per amplule)
● Adrenalin Injection (1mg in Ampins)
● Morphine injection (15 mg per amples)

587. What does LES stands for?


● Land Earth Services
● Low Earth Station
● Land Earth Station
● Local Earth Station

588. Choose from the following statement is TRUE regarding an MF/HF DSC Distress call?
● Follow on communications should be presumed to take place on the voice frequency
associated with the specific DSC frequency used.
● Both the nature of Distress and the alternate emission and frequency must be specified foe follow-up
communications in the original Distress alert.
● An alternate emission and frequency may be specified for follow-up communications by the vessel in
the original Distress alert
● Follow on communications should be presumed to take placed on the SITOR(NBDP) frequency
associated with the specific DSC frequency used.

589. How would you explain the typical caused of much longer than normal VHF communications
distances?
● By skywave reflections from the D layer
● By changing power from 1W to 25W
● By ionospheric activity in layers F1/F2
● By atmospheric ducting or tropospheric propagation
590. GMDSS equipment is required to be powered from the following source EXCEPT:
● A dedicated radio battery supply.
● Ship’s emergency alternators/generators
● Ship’s normal alternators/generators.
● An emergency solar powered battery supply

591. How many VHF frequencies are available under GMDSS for DSC distress related Calls?
● Five
● One
● Four
● Two

592. What is the requirements for emergency and reserve power in GMDSS radio installations?
● Compulsory ships must have emergency and reserve power sources for radio communications.
● A reserve power source is not required for radio communications
● Only one of the above is required if a vessel is equipped with second 406 epirb as a back up means of
sending a Distress alert.
● An emergency power source for radio communications is not required if a vessel has proper reserve
power (batteries)

593. What part of a satellite EPIRB may function as a visual aid to rescue vessels?
● Loud beeping tone emitted by the unit once activated
● 406 MHz signal from satellite EPIRB
● Strobe light
● A 121.5 MHz emergency transmitter in a satellite EPIRB

594. Which of the following statements concerning EPIRB alerts is FALSE?


● The COSPAS SARSAT system may take a full hour or more to provide an alert.
● The INMARSAT system provides worldwide coverage for Distress alerts
● 406 Mhz EPIRB units may be equipped with GPS receiver.
● The GOES weather satellites are in a geostationary orbit.

595. In radiowave propagation, the distance traveled by a radio wave from one wave top to another is
called?
● frequency
● wavelength
● wave band
● period

596. What is the wavelength of a radar signal that responds only to an activated SART?
● 4cm
● 3cm
● 1cm
● 2cm
597. What does RX stand for?
● Radio Telex
● Medical Prescription
● Receiver
● Rescue

598. What does DNIC stands for?


● Direct Network Identification Code
● Data Network Identification Circuit
● Data Network International Code
● Data Network Identification Code

599. If a distress alert is received on the 12 MHz DSC frequency, how would you demonstrate your
action?
● Use DSC to acknowledge/relay the alert using the 12 MHz DSC frequency
● Set the transceiver to 12520.0 kHz simplex F1B/J2B emission
● Do nothing. Ship is too far away to render assistance
● Set the transceiver to 12290.0 kHz simplex J3E emission

600. How would you describe the radiation pattern of a vertical(whip) antenna?
● an ellipse
● a cardioid
● a figure of eight
● a circle

601. In which ADMIRALTY LIST OF RADIO SIGNAL (ALRS) publications can you find the
frequencies and times of traffic list by coast stations?
● Vol.1
● Vol.4
● Vol.3
● Vol.2

602. In radiotelephony “SEELONCE FEENE” means?


● Restricted may not be resumed
● Distress has ended and normal working may be resumed
● Distress has not ended and normal working may not be resumed
● Search and rescue operation is still ongoing

603. Which statement is correct regarding a vessel equipped with GMDSS equipment that will remain
in Sea Area A1 at all times?
● VHF DSC alerting may be the sole means of Distress alerting
● The vessel must be provided with a radio installation capable of initiating the transmission of
ship-to-shore distress alerting from the position from which the ship is normally navigated
● HF SSB with 2182 kHz automatic alarm generator may satisfy the equipment requirement
● HF or MF DSC may satisfy the equipment requirement
604. You are in Sea Area A1 and require medical assistance from Aberdeen Coastguard. What you
should initially transmit on Ch.70?
● A DSC Urgency call addressed to Aberdeen Coastguard MMSI
● A DSC Routine call to all stations
● A DSC Distress Alert addressed to Aberdeen Coastguards MMSI
● A DSC Safety Call to all stations

605. What element of a DSC call describes the emergency position with aid of 10 digits?
● Message 4 of distress calls
● Message 1 of distress calls
● Message 3 of distress calls
● Message 2 of distress calls

606. What element of a DSC call indicates the frequency desired for the subsequent
communication?
● Message 4 of distress calls
● Message 2 of routine calls
● Message 3 of distress calls
● Message 1 of distress calls

607. Can routine DSC alerts be made on 2187.5 KHz?


● At No time
● Between 1700 and 1900 UTC
● Between 0900 and 1700 UTC
● At Any time

608. Choose from the following statement about the term duplex “ITU Channel”?
● It refers to a vessel’s SELCAL number
● It is a series of frequency pairings used for ship-ship communications
● It is a standardized series of frequency pairings for common use
● It refers to VHF channels 128 and 60-88

609. Which of the following is the purpose of the SART’s audible tone monitor?
● It informs A survivor when the battery’s charge Condition has weakened
● It informs Survivors that assistance may be nearby
● It informs survivors that a nearby vesel is signaling on DSC
● It informs survivors when the SART switches to the “standby” mode

610. How would you classify the MMSI number 002570700?


● It is ship earth station
● It is a coast station
● It is aircraft station
● It is a ship station

611. What is an urgency transmission?


● A communications alert that important personal messages must be transmitted
● A communications transmission concerning the safety of a ship, aircraft, or other vehicle or of
some person on board or within sight
● A radio distress transmission affecting the security affecting the humans or property
● Health and welfare traffic which impacts the protection of on-board personnel\

612. In the preamble of the NAVTEX message, what is the technical code of the identity of the
transmitting station?
● B3
● B2
● B1
● B4

613. What does PLB stands for?


● Personal Life Band
● Public List Beacon
● Personal Locator Beacon
● Public Locator Beacon

614. Your ship is participating in the AMVER reporting system, which of the following report is sailing
at your departure?
● AMVER/PR//
● AMVER/SP//
● AMVER/DP//
● AMVER/DR//

615. With reference to Radio Regulation Chapter 1 Article 1 Section IV 1.70. Which of the following is
an earth station in the fixed-satellite service or in some cases in the mobile-satellite service, located at
a specified fixed point or within a specified area on land to provide a feeder link for the mobile-satellite
service?
● Base earth station
● Land mobile earth station
● Mobile earth station
● Land earth station

616. Which of the following 3-letter signal means "Give enema"


● MRW
● MRS
● MRX
● MRU

617. What is class “A” EPIRB?


● A satellite-based maritime distress and safety alerting system
● An automatic, battery-operated emergency position indicating radio beacon that floats free of a
sinking ship
● A high efficiency audio amplifier
● An alerting device notifying mariners of imminent danger
618. Reference to Radio Regulation Article 1 Section III Radio Service. What is meant by a mobile-
satellite service in which mobile earth stations are located on board ships; survival raft stations and
emergency position-indicating radiobeacon stations may also participate in this service?
● land mobile service
● maritime mobile service
● land mobile-satellite
● maritime mobile-satellite service

619. Which is true concerning a required watch on VHF Ch-16?


● When a vessel is in an A1 sea area and subject on the Bridge-toBridge act and in a VTS system, a
watch is required on Ch-16 in addition to boh Ch-13 and he VTS channel
● It is not always compulsory in sea areas A2,A3 and A4
● When a vessel in an A1 sea area and subject to the Bridge-to-Bridge act and in a VTS system, a
watch is not required on Ch-16 provided the vessel monitors both Ch-13 and the VTS channel
● It is not compulsory at all times while at sea until further notice, unless he vessel is in a VTS system

620. What does SSC stands for?


● Surface Service Committee
● Search and Services Committee
● Surface Search Council
● Surface Search Coordinator

621. Which from the following list the mandatory DSC watchkeeping bands/channels?
● VHF Ch.70, 2MHz MF DSC, 4 MHz DSC and 8 MHz DSC
● 8 MHz HF DSC, 1 other HF DSC, 2 Hz F DSC and VHF Ch.70
● VHF Ch.70, 2MHz MF DSC, 6 MHz DSC and 1 other HF DSC
● 2MHz MF DSC, 8 MHz DSC, VHF Ch.16 and 1 other HF DSC

622. Frequency modulation is best describes from which of the following statement?
● Both the amplitude and frequency are changed by the modulating signal
● High level mixing of the final amplifier signal and the information signal
● Frequency modulation is subject to interference by atmospheric noise
● The information signal changes the radio carrier frequency but the amplitude remains constant

623. Which of the following 3-letter signal means "You should follow treatment in your own medical
guide”?
● MQT
● MQP
● MRJ
● MQO

624. In the international code of signals, what does a group of three letter indicates?
● The vessel national identity signal
● A group from general signal code
● Urgency or an emergency
● A group from the medical signal code

625. As stated by the convention, what is the requirement for ships which do not carry a medical
doctor on board?
● A medical secretary whose responsibility is to guide the Master in handling medical care
● At least one seafarer who is in charge of medical care and administering medicine as part of
their regular duties or at least one seafarer competent to provide first aid
● A qualified therapist who is responsible for therapy of possible patient
● At least one nurse who is in charge of medical care and administering medicine as part of their regular
duties or at least one seafarer competent to provide first aid

626. How long should the battery of the SART last when in Transpond mode?
● 5 hours
● 8 hours
● 4 hours
● 7 hours

627. If the Inmarsat-c terminal is inoperative but the vessel remains within NAVTEX coverage, which
of the following message categories should not be disabled by the GMDSS Radio Operator?
● Search and Rescue information, navigational warnings and other electronic navaid messages
● Search and Rescue information, meteorological warnings and ice reports
● Navigational warnings, meteorological warnings and meteorological forecast
● meteorological warnings, search and rescue information and navigational warnings

628. How does a SART signal appears on the RADAR screen looks like?
● It will appear on a radar unit’s PPI as a line of dots radiating outward with the outermost dot indicating
the SART’s position
● None of the above
● It will appear on a radar unit PPI as a line of dots radiating outward with the innermost dot
indicating the SART position
● It transmits “SOS” and the vessel’s name and position in slow speed Morse Code

629. Reference to Radio Regulation Article 1 section III Radio Service. What is meant by a maritime
mobile service in or near a port, between coast stations and ship stations, or between ship stations, in
which messages are restricted to those relating to the operational handling the movement and the
safety of ships and in emergency to the safety persons?
● broadcasting service
● port operations service
● maritime radionavigation service
● ship movement service

630. What is the signal transmitted in J2B mode?


● An upper sideband of a single tone switched on and off
● An upper sideband of 2 alternating tones
● A full carrier and one sideband
● A full carrier and two sidebands
631. What is the primary purpose of the GMDSS
● Allow more effective control of SAR situations by vessels
● Provide additional shipboard systems for more effective company communications
● Automate and improve emergency communications for the world’s shipping industry
● Effective and inexpensive communications

632. What is the code used to gain access to sore-and-forward unit(Sfu) for national calls?
● 21
● 24
● 22
● 31

633. Which of the following 3-letter signal means "To induce sleep, give two sedative tablets” ?
● MSK
● MSL
● MSJ
● MSM

634. Which of the following book aims to provide guidance to those who, during emergencies at sea
may require assistance or may be able to render assistance?
● ITU List of Coast Station
● MERSAR manual
● Admiralty List of Radio Signals
● IAMSAR manual

635. Reference SOLAS IV, Regulation2, what is meant by an area within the radiotelephone
coverage of at least one VHF coastal station in which continuousDSC alerting is available?
● Sea Area A4
● Sea Area A3
● Sea Area A1
● Sea Area A2

636. What is the transmitting range of most NAVTEX stations?


● Typically upwards of 1000 nautical miles (1800 km) during daytime
● Typically 50-100 nautical miles (90-180km) from Shore
● It is limited to line of sight or about 30 nautical miles (54km)
● Typically 200-400 nautical miles (360-720km)

637. Which of the following would be best be used for visual detection of a distressed vessel?
● A 121.5 MHz EPIRB beacon
● A SART two-tone alarm
● An EPIRB strobe light
● A 9-GHz SARTs beacon

638. How would you explain the effect of a dirty battery terminals?
● May cause a excessive gassing
● may cause an increase in charging voltage
● may cause a decrease in the discharge current when off load
● may cause a decrease in the voltage of the battery when on load

639. What would be the best choice in attempting to communicate with a shore station 15 miles
(24km) distant?
● VHF Channel 70
● VHF Channel 06
● VHF Channel 16
● VHF Channel 12

640. What do NAVTEX broadcasts typically achieve maximum transmitting range?


● Middle of the night
● Sunset
● Local Noontime
● Post sunrise

641. When was full equipment carriage under GMDSS become mandatory?
● mandatory after February 10, 1995
● mandatory after February 1, 1996
● mandatory after February 1, 1992
● mandatory after February 1, 1995

642. Which of the following is a functional or carriage requirement for compulsory vessels?
● A compulsory vessel must be able to transmit and respond to Distress lers and carry only one (1) FCC
licensed GMDSS Radio Operator in sea areas A1 & A2
● A compulsory vessel must satisfy certain equipment carriage requirements based on the
intended sea area of operation
● None of these answer is correct
● A compulsory vessel must carry at least two(2) FCC licensed GMDSS Radio Operators in all sea areas
as well s a GMDSS Maintainer in sea areas A3 &A4

643. What should be the prefix to a MEDICO message so that communication personnels know how
to handle them?
● MEDRAD
● XXX RADIOMEDICAL
● DHMEDICO
● PANPAN MEDICO

644. What is the correct DSC Alert Priority to be used for sending a Distress Alert on behalf of
another vessel?
● Mayday relay
● Distress relay
● Urgency
● Distress
645. What is the proper procedure to be followed upon receipt of a distress alert transmitted by use of
Digital Selective Calling Techniques?
● Ship stations equipped with narrow band direct printing equipment should respond to the distress alert
as soon as practicable by this means
● Set a continuous watch on VHF-F channel 13,16 and DSC on Channel 70
● Set watch on the radiotelephone distress and safety frequency associated with the distress and
safety calling frequency on which the distress alert was received
● Set watch on the DSC alerting frequency in the band of frequencies the alert was received

646. Which of the following are personnel, functional, or equipment FCC requirements of the
GMDSS?
● Distress alerting and response, two STCW GMDSS watchstandard, equipment carriage based on
intended sea area of operation
● Equipment carriage based on intended sea area of operations, distress alerting nd response
and two FCC licensed GMDSS radio operators
● Equipment carriage reduced for sea areas A3 & A4, Distress alerting and response and two FCC
licensed GMDSS radio operators
● One FCC licensed GMDSS radio operator in sea are A1 & A2, two FCC licensed GMDSS radio
operaors in sea areas A3 & A4 and equipment carriage based on intended ea area of operations

647. Identify which of the following numbers indicates a ship station MMSI?
● 3372694
● 338472
● 623944326
● 030356328

648. What might be used as homing signal by the search and rescue vessels in the immediate
vicinity of the ship in distress?
● A 121.5 MHz emergency transmitter in a Safelite EPIRB
● Strobe light
● 406 MHz signal from satellite EPIRB
● Flare gun

649. What does ELT stands for?


● Emergency Locator Transmitter(Airborne)
● Emergency Load Transmitter
● Emergency Locator Transport
● Enhanced Locator Transmitter

650. Which of the following equipment is NOT a mandatory carriage requirement for survival craft
under GMDSS?
● At least one rada transponder must be carried on every cargo ship of 300-500 gross tons and two
transponders (one for each side) of every passenger ship and every Cargo ship of 500 gross tons and
upward
● At least two approved two-way VHF radiotelephones on every cargo ship between 300-500 gross tons
● At least an AIS equipment is provided in a survival craft under GMDSS
● At least three approved two-way VHF radiotelephones on every passenger ship and cargo ships of 500
gross tons and upwards

651. Which of the following 3-letter signal means "Commence artificial respiration immediately”?
● MRK
● MRI
● MRL
● MRJ

652. Which of the following 3-letter signal means “Give rectal saline slowly to replace fluid loss”?
● MSD
● MSA
● MSB
● MSC

653. Select from the following are required GMDSS functions?


● Bridge-to-Bridge communications, general communications, SAR communications
● Reception of weather map facsimile broadcasts, receiving company email. On-scene communications
● Bridge-to bridge communications, reception of weather map facsimile broadcasts, SAR
communications
● Reception of weather map facsimile broadcasts, receiving company email, On-scene communications

654. What does AORW stands for?


● Atlantic Operators Radio Watch
● Atlantic Ocean Region West
● Atlantic Operators Region West
● Above Ocean Region West

655. What is the acronym for AUSREP?


● Australian vessel rescue reporting system
● Australian vessel reporting system
● Australian Radio Reporting System
● Australian Ship Reporting System

656. How would you describe the equipment carriage requirement for survival raft under GMDSS?
● At least two radar transponders must be carried on every cargo ship of 300-500 gross tons and two
radar transponders (one for each side) of every passenger ship regardless of tonnage
● At least three (3) Survival Craft Transponder units and two(2) SART’s on every passenger ship
and cargo ships of 500 gross tons and upwards
● All cargo vessels above 300 gross tons and every passenger ship regardless of tonnage must carry
three(3) Survival Craft Transponder units and two(2) SART’s
● At least three (3) Survival Craft Transponder units and two(2) SART’s on every cargo ship between
300-500 gross tons and the same on all passenger ships regardless of tonnage

657. Which does ASCII stand for?


● American Standard Code for International Interchange
● American Standard Code for Information Interchange
● American Standard COde for Information Interchange
● American Standard Code for Interchange Information

658. What is the ionosphere layer that allows high frequency long distance transmissions due to its
refractive capabilities for frequencies up to 30 mhz, and also due to the long skip distance provided by
refraction from such high elevation?
● E layer
● D layer
● C layer
● F layer

659. What information should be contained in a designated Distress alert?


● ID, position and number of crew onboard
● ID, position and vessel’s call sign
● ID, position and the nature of distress
● ID, position and name of vessel in distress

660. Concerning the testing of a compulsory radiotelephone station, which of the following statement
is FALSE?
● The most effective method by calling the Coast Guard on VHF Ch.16 or 2182.0 kHz
● The test may not interfere with communications in progress and must wait or be suspended if a request
to do is made
● Test may be accomplished by using the equipment for normal business
● A daily test is necessary unless the equipment was used for routine traffic

661. Regarding the maintenance of GMDSS equipment at sea, select from the following statement is
INCORRECT?
● Ships must carry at least one person who qualifies as a GMDSS maintainer for maintenance and repair
of equipment if the at-sea maintenance option is selected
● The GMDSS maintainer may be the person responsible for ensuring that the watches are properly
maintained and the proper guard channels and the vessel’s position are entered into the DSC
equipment
● The GMDSS maintainer may not be the person designated to have primary responsibility for
radio communications during Distress incidents even if licensed as an operator
● All at-sea maintenance and repairs must be performed by or under the supervision of a person holding
a GMDSS maintainer license

662. Which from the following statement best describes amplitude modulation?
● The amplitude in the sideband(s) changes but the radio carrier frequency remains constant
● The frequency is varied in synchronization with the modulating signal
● The character data from the terminal is changed to audio tones
● The amplitude of the carrier is changed but there is still only a single frequency being transmitted

663. What does FEC Stands for?


● Forward Error Connection
● Forward Error Correction
● Fas Error Correction
● Fast Error Connection

664. What is distress traffic?


● All message relative to the immediate assistance required by a ship, aircraft or other vehicle in
imminent danger
● In radiotelegraphy, SOS sent as a single character, in radiotelephony, the speaker of the word
“Mayday”
● Internationally recognized communications relating to emergency situations
● Health and welfare messages concerning the immediate protection of property and safety of human life

665. What is the code used for the retrieval of meteorological forecast?
● 42
● 43
● 52
● 51

666. How are the normal working conditions restored after radio silence has been imposed?
● The rescue coordination Center (RCC) that imposed the radio silence must transmit a voice message
on the distress frequency stating /’’ Mayday’’
● The rescue coordination center (RCC) that imposed the radio silence must transmit a voice message
on the distress frequency stating/’’Mayday Relay’’
● The rescue coordination center (RCC) that imposed the radio silence must transmit a voice message
on the distress frequency stating/’’Prudonce’’
● The rescue coordination center (RCC) that imposed the radio silence must transmit a voice
message on the distress frequency stating/’’Seelonce Feenee’’

667. For MF-HF voice signals, how would you describe the emission designation?
● It is a J3E
● It is a J2B
● It is a F1B
● It is a F3E

668. After received the full details as specified by Doctor instructions, which of the following 3 letters
signal progress report sent by the Master that the ‘’ Patient is improving’’?
● MPE
● MPF
● MPG
● MPK

669. Which of the following ALRS Volumes is split across two publications and includes radio details
for:A.Maritime Weather services B. Safety Information broadcast C. World wide NAVTEX and
safetyNET information D.Submarine and Gunnery Warning details (Subfacts and Gunfacts) E. Radio-
Facsimile Station,frequencies and weather map areas.
● Volume 3(NP283)
● Volume 4(NP284)
● Volume 1 (NP281)
● Volume 2 (NP282)

670. What does SCDA stand for?


● Super Control on Data Authority
● Special Control and Data Authority
● Supervisory Control and Data Acquisition
● Supervisory Center and Data Acquisition

671. ‘’PAN PAN’’ spoken 3 times is heard by the watch officer over the Radiotelephone from another
station,what does thid mean?
● The mobile unit is in need of immediate assistance.
● The station is preparing to transmit safety message regarding vessel traffic movements in the area.
● The station will transmit a navigation or important meteorological warning message to follow.
● The station will transmit an URGENT message possibly concerning the safety of a mobile unit
or person

672. What is the function of squelch in VHF receiver?


● To eliminate audio noise where there is no traffic
● To silence the interference
● To minimize statistics
● To increase the volume

673. How does type “D” or SAR information messages in the international NAVTEX system are
numbered?
● “00”
● “01”
● “02”
● “03”

674. Choose from the following statement regarding a GMDSS equipped ship is FALSE?
● Ships must have the required equipment inspected at least once every five years
● The regulations apply to all passenger ships regardless of size and cargo ships of 300 gross ons and
upwards
● Ships must carry at least two persons holding GMDSS Radio Operator’s License for distress and
Safety radio communications purposes
● A conditional or partial exemption may be granted, in exceptional circumstances, for a single voyage
outside the sea area for which the vessel is equipped

675. What is the action that a GMDSS radio operator should take when the DSC distress alert is
received?
● The operator should immediately set continuous watch on the radio telephone frequency that is
associated with frequency band on which the distress alert is received
● Transmit distress ebay alert immediately
● Acknowledge the alert at once on the radio telephone frequency that is associated with frequency band
on which the distress alert is received.
● Switch OFF the DSC receiver to silence the alarm

676. What is the two-letter signals which means “I AM DISTRESS AND REQUIRE IMMEDIATE
ASSISTANCE”?
● “NC”
● “GB”
● “CI”
● “NM”

677. What is the reporting system that covers the area within the Continental Shelf or Exclusive
Economic Zone off the coast of Greenland?
● Off Greenland
● GREENREP
● GREENPOS
● Greenland Traffic

678. Which of the following statements concerning GMDSS Radio Operator requirements is FALSE?
● Each compulsory vessel must carry at least two licensed GMDSS Operator at all times at sea
● While at sea, adjustments to, and the maintaining of, GMDSS equipment may be performed by the
GMDSS Radio operator as long as the work is supervised by an on board licensed GMDSS Radio
Maintainer
● All communications involving safety of life at sea must be logged as long as the compulsory
vessel was not involved in such communications
● Each compulsory vessel must carry at least two licensed GMDSS Radio Operators at all times while at
sea and may elect to carry a GMDSS Radio Maintainer as well

679. What does the message 1 in the DSC distress call contains?
● Nature of Distress
● Time indication (UTC) when the coordinates were valid
● Single character to indicate the type of communication which is preferred by the station in distress for
subsequent exchange of distress traffic
● Distress coordinate message

680. How would you demonstrate the power method MF DSC equipment should be tested?
● weekly, by means of an internal test
● monthly, by contacting a coast station
● monthly, by means provided by the equipment
● weekly, by contacting a coast station

681. A GMDSS equipped vessel must carry equipment capable of Medium Frequency (MF) DSC
transmission and reception
● In all operating ocean areas
● When operating outside of VHF-DSC range of shore
● When operating in areas of no INMARSAT coverage
● Only if not equipped with the Enhanced Group Call System

682. How would you describe modulation?


● Converting the carrier from a low frequency to a higher frequency
● Adjusting the frequency to the optimum band for long distance communications
● Imposing intelligence onto a radio carrier signal
● Changing mark-space to 1 and 0

683. Over water, what should be the detectable range of an AIS-SART?


● 12nm
● 8nm
● 10nm
● 5nm

684. What is safety transmission?


● Radiotelephony warnings preceded by the words “PAN”
● A radiotelephony lert preceded by the letters “XXX” sent three times
● A communications transmission which indicated that a station is preparing to transmit an
important navigation or weather warning
● Health and welfare traffic concerning the protection of human life

685. Which of the following ALRS Volume that includes information for: A. Worldwide communication
requirements for distress, search and rescue B. Extracts from SOLAS and ITU Regulations C. Distress
and SAR (Incorporating MRCC and MSC contacts) D. Worldwide NAVTEX and Maritime Safety
Information
● Volume 6 (NP286)
● Volume 4 (NP284)
● Volume 5 (NP285)
● Volume 3 (NP283)

686. Which GMDSS equipment is used in or near the survival craft?


● Walkie-Talkie
● ELT
● EPIRB
● PLB

687. In Chapter 3 of the International Code of Signals, Section 2: Request for medical assistance,
which code should your message describing that the patient has had no previous illness?
● MBC
● MBE
● MBF
● MBD

688. You have received the 9 digit MMSI of a ship in distress. In which ITU publication would you find
the name of the ship?
● ITU list of coast stations
● List of call signs and numerical identities
● Admiralty list of radio signals
● IT list of ship station

689. What is an example of a locating signal?


● Ship to shore transmission
● Loran C
● SSB phone traffic
● A SART in transpond mode

690. When performing battery maintenance, what is the purpose of petroleum jelly?
● to protect exposed terminals and connectors
● to protect hands from acid splashes
● to protect the battery casing from corrosion
● to protect against acid leakage through cell top caps

691. What is the code use to make automatic telex calls using the international telex country?
● 11
● 01
● 10
● 00

692. When positive information is received that a ship or a person on board is in grave and imminent
and in need of immediate assistance, what is the emergency phase to be declared?
● SAR phase
● Critical phase
● Distress phase
● Uncertainty phase

693. What is the transmission speed of a DSC call on the VHF band?
● 1350 BAUDS
● 1250 BAUDS
● 1300 BAUDS
● 1200 BAUDS

694. In chapter 3, section 1 of the International Code of Signals, what is the meaning of the code MFE
N?
● Bleeding is not severe
● Bleeding has been stopped by tourniquet
● Bleeding is slight
● Bleeding can be stopped

695. What element of the DSC call indicates if the call requires an acknowledgement of receipt and if
the current message is a reply to another call?
● Awaiting acknowledgement
● SSb connected
● End of sequence
● DSC call in progress

696. How would you demonstrate your action after sending a false distress alert on MF?
● Make a voice announcement to cancel the alert on 2174.5 kHz
● Make a voice announcement to cancel the alert on 2182.0 kHz
● Send another DSC alert and follow on with voice on 2182.0 khz
● Make voice announcement to cancel the alert on 2187.5 khz
697. You are onboard a GMDSS equipped vessel as GMDSS operator what is your responsibility
under GMDSS?
● You should be able to perform communications functions that are essential for the safety of
your ship itself and of the other ship.
● If you are close to a vessel in distress regardless of your vessel size you much render assistance
● If your ship is over 300GT you can be require to render assistance if such assistance will not effect your
port schedule
● As GMDSS vessel if outside service areas A2 and A3 you may be require to render assistance in
distress situation.
698. What should you do after receiving DSC Distress Alert Relay from a MRCC on MF?
● Listen for Mayday Relay message on 2187.5 khZ.
● Listen for Mayday Relay message on 2182 khz.
● Immediately acknowledge by R/T on 2182 khz.
● Immediately acknowledge by R/T on 2187.5 khz
699. What is the responsibility of compulsory GMDSS vessels?
● Only the vessels closest to a Distress incident must render assistance.
● Every vessel must be able to perform communications functions essential for its own safety of
other vessels.
● Vessel must transmit a DSC distress relay upon receipt of a DSC distress alert.
● Vessel must immediately acknowledge all DSC distress alerts.
700. Which of the following stand for RCC?
● Rescue Coordinating Council
● Rescue Center Cooperative
● Rescue Cooperative council
● Rescue Coordinating Center
701. What is the emergency frequency on MF(Medium frequency) radio?
● 1616
● 1718
● 2182
● 2617
702. How would you describe the transmission of a distress alert by a station on behalf of another
vessel actually in distress should not occur?
● When the responsible person at Coast Station determines further help is necessary.
● when the mobile unit actually in distress is not itself in a position to transmit the distress alert.
● When the master or responsible person on the mobile unit not in Distress so decides
● When communications between the Distress vessel and a Coast stations are already in
progress.
703. What is the worldwide system operated exclusively to support Search and Rescue (SAR) and
make available to all Rescue Coordination Centers (RCCs)?
● IAMSAR
● INSPIRES
● AMVER
● SARSAT
704. Which device provides the main means in the GMDSS for locating ships in Distress, or their
survival craft?
● Satellite EPIRBs
● MF/HF DSCs
● VHF homing devices
● Radio direction finders
705. How the vessel’s operational capacity in detecting the operation of a SART in the vicinity?
● A unique radar signal consisting of a blip code radiating outward from a SART’s position along
its line of bearing
● A Unique two tone alarm signal heard upon the automatic unmuting of the 2182 khz radiotelephone
automatic watch receiver
● The SART signal appears as a target which comes and goes,the effect off heavy swells on a SART
● A unique two tone ‘’warbling’’ signal heard on VHF-FM Channel 70
706. Which of the following is not a CORPAS-SARSAT beacons?
● EPIRB
● ELT
● SART
● PLB
707. Which of the following indicates a Coastal station?
● 041329746
● 003572946
● 325700142
● 454813265
708. Choose the best answer that would indicate an Inmarsat-C terminal?
● A 9 digit number beginning with the MID
● A 9 digit number always starting with 4
● A 9 digit number always starting with 3
● A 7 digit number
709. What should you do after receiving VHF DSC Distress Alert Relay from a MRCC?
● Immediately acknowledged by R/T on Ch 16.
● Listen for a May day relay on Ch 70.
● Listen for a Mayday relay on VHF Ch 16.
● Immediately acknowledged by DSC on Ch 70.
710. In what conditions would you relay a DSC distress alert?
● If the mobile unit in distress request to relay the Distress Alert communications
● If coast station request to relay the distress alert communication
● If the mobile unit in distress is incapable of further Distress Alert communications
● You should never relay such alert the Coast station & RCC will do that
711. Which of the following terms is defined as a backup power source that provides power to radio
installations for the purpose of conducting Distress and Safety communications when the vessel’s main
and emergency generators cannot?
● Reserved Source of Energy(RSE)
● Emergency Back-up Generator (EBG)
● Reserve Source of Diesel Power (RSDP)
● Emergency Diesel Generator (EDG)

712. What should you do after receiving VHF DSC Distress Alert Relay from a MRCC?
● Immediately acknowledged by R/T on Ch 16.
● Listen for Mayday relay message on VHF CH 16
● Immediately acknowledge by DSC on Ch 70
● Listen for a Mayday Relay on Ch 70

713. Your ship has a restricted maneuverability, you want to send a message to all ships in the
vicinity requesting to wide berth. The priority for such message is:
● Distress
● Routine
● Safety
● Urgency

714. What is an ‘’Adopted Ship Reporting System’’?


● Established by a government/s after it has been accepted by the Organization as complying
with all requirements of SOLAS regulation V/II
● Designated by a contracting Governments with the responsibility for the management and coordination
of a system,the interaction with participating ships,and the safe and effective operation of a system
● Implemented by a competent authority designed to improve the safety and efficiency of vessel traffic
and protect the environment
● Interchange of data between ships participating in a system and shore based authority aimed at
enhancing maritime safety or protection of marine environment
715. Where can additional information on operators of ship reporting systems may be obtained?
● Rescue coordination Center(RCC)
● On scene co-ordinator (OSC)
● Rescue sub-center (RSC)
● Aircraft co-ordinator (ACO)

716. How can SART’s effective range be maximized?


● It must be switched On position
● The sart into the sea as soon as possible to begin transmitting
● It must be mounted 1 meter above the sea surface while on board the survival craft
● It must be switched to Test position

717. In GMDSS, What is the VHF channel reserved for the bridge-to-bridge communication?
● Channel 70
● Channel 1
● Channel 13
● Channel 2

718. How many pieces of SART must be provided for a ship of 500 gross tonnage & above?
● 2
● 4
● 1
● 3

719. How would you describe the main purpose of RT transmission on VHF Ch.13?
● It is for SAR aircraft communications to home onto an EPIRB
● It is for ship-to-ship safety of Navigation communications
● It is for canceling a false DSC Distress Alert sent on CH. 70
● It is for ship-to-ship routine communications

720. What statement is TRUE regarding the additional equipment carriage requirement imposed for
the survival craft of vessel over 500 gross tons?
● Additional carriage of one adio equipped lifeboats
● Additional carriage of four radio equipped lifeboats
● Additional carriage of two radio equipped lifeboats
● Additional carriage of three radio equipped lifeboats

721. Which of the following MMSI belongs to a Philippines registered vessel?


● 854123400
● 541823400
● 584123400
● 548123400

722. Which of the following statements concerning CORPAS-SARSAT is not true?


● EPIRBs are units that are used as alerting devices
● The doppler frequency measurement concept is used to determine the EPIRB’s location
● These are satellites detecting signals from SART
● thee are satellites in a low-earth polar orbit that detect EPIRB

723. What should you do when you received a DSC alert?


● You should immediately set continuous watch on the radiotelephone frequency that is
associated with the frequency band which the distress alert was received
● You should immediately set continuous watch on VHF channel 70
● No action is necessary as the DSC unit will automatically switch to the NBDP on follow on
communication frequency
● You should immediately set a continuous watch on the NBDP frequency that is associated with the
frequency band which the distress alert was received

724. What is the standard reference designed for use by all crew members incharge with providing
medical care onboard?
● The International Medical Guide for Ships
● CIRM
● The international first aid guide for ships
● radio medio

725. What does MRCC stands for?


● Maritime Receiver Center Council
● Maritime Rescue Coordinating Centre
● Maximum Rescue Coordinating Centre
● Maritime Rescue C0uncil Coordinator

726. Which of the following stand for RSC?


● Rescue Sub Centre
● Rescue Services COmmittee
● Escue Special Cooperative
● Rescue Services Center

727. Who should be the sole authority to order the use of distress signals or distress alerts?
● Only the ship owner
● Only the master
● The first person to discover the distress situation
● The person designated to maintain communication during distress situations

728. Which of the following ALRS Volumes includes radio details for I. Global Maritime
Communications II. Satellite Communications Services III. Coastguard Communications IV. Maritime
TeleMedical Assistance Services (MTAS) V. Radio Quarantine and Pollution reports VI. Anti-Piracy
Contact Table?
● Volume 3 (NP283)
● Volume 1 (NP281)
● Volume 4 (NP284)
● Volume 5 (NP285)

729. What is the primary advantage of single-sideband over FM for voice transmissions?
● All of these choices are correct
● SSB signals have narrower bandwith
● SSB signals are less susceptible to interference
● SSB signals are easier to tune

730. In the EGC safety net service, what type of navtex warning is letter “A” represent?
● Navigational Warning information
● Search and rescue warning information
● Meteorological warning information
● Sat Nav messages

731. Who should establish VTS where traffic volume or risks justify it?
● Port Captains
● IMO
● Classification Societies
● Contracting Government

732. Which Equipment is required for all vessels over 300 gross tons and all passenger ships after
Aug 1, 1993?
● VHF Tron air
● SART
● Navtex and Satellite EPIRB
● VHF transceivers

733. For safety reasons, radar reflectors fitted should produce reliable detection in practical conditions
and perform in what type of radar?
● 3 GHz (S-Band)
● 9 GHz (X-band)
● Both 3 GHz (S-Band) and 9 GHz (X-band)
● Both 9 GHz (S-Band) and 3 GHz (X-band)

734. A shore-to-ship distress alerts relay initiated by a station of a rescue coordination centre which
receives a distress alert shall be addressed to any of the following, EXCEPT?
● Navigation Lanes
● Coast Stations
● Coastal and inland water
● Inland and Coastal waters

735. When measured directly above given point, solar radiation in the ionosphere is lowest or lesser
to minimum during what time?
● Sunrise or night
● At noon
● At night
● At sunlight

736. What is the approximate bandwidth of a single sideband voice signal?


● 15 kHz
● 1 kHz
● 3 kHz
● 6 kHz

737. What is the preferred frequency / channel for two-way communication with survival craft during
SAR operations?
● 406 kHz
● 9 GHz
● 2182 kHz
● VHF Ch. 16

738. At midday in the summer, what would be the best choice in attempting to communicate, using
NBDP with shore station some 1800 miles distant?
● VHF
● Lower HF bands
● MF
● Higher HF bands

739. Select from the following frequencies does channel 70 operates as digital selective calling for
distress, safety, and calling frequencies?
● 158.526 MHz
● 159.526 MHz
● 156.526 MHz
● 157.526 MHz

740. Following the stand-by period, how long for the SART batteries provide transponder
transmission when being continuously interrogated by a radar with pulse repetition frequency of 1 kHz?
● 24 hours
● 48 hours
● 8 Hours
● 12 Hours

741. Who should comply with the requirements of an adopted mandatory reporting systems?
● Masters
● Flag Administrations
● Ships
● Ship owners

742. Can you choose which one of the following number is a ship station SELCAL?
● 1104
● 230364973
● 1502352
● 11243

743. What equipment is programmed to initiate transmission of distress alerts and calls to individual
stations?
● NAVTEX
● DSC Controller
● GPS
● Facsimile

744. Which is the message type with the highest priority?


● Request for berthing instructions
● Telephone call to owners
● Gale Warnings
● Radio Telex Letter (RTL)

745. What is the basic information to estimate ship’s position, and it should be sent at the time when a
ship participates in ship reporting system?
● Sailing Plan
● Monitoring Plan
● Passage Plan
● Deviation Plan

746. What is the first three digits refers to when a ship has an MMSI pf 257289000?
● The route of the ship
● The name of the ship
● The nationality of the ship
● Company name of the ship

747. What is the vessel reporting system that is operated by AMSA as part of the services offered by
the joint rescue coordination center (JRCC Australia) to track location of vessels?
● MASTREP
● MAREP
● MEDREP
● MANCHEREP

748. When measured directly above a given point, solar radiation in the ionosphere is minimum
during what time?
● At sunlight
● At night
● At sunset
● At noon

749. What system provides accurate vessel position information to the GMDSS equipment?
● INMARSAT C
● EPIRB
● NAVTEX
● GPS

750. How many digits for the ship station selective call number shall be assigned by the responsible
administrations as per Radio Regulation Article 19 Section V?
● six (6) digits
● three (3) digits
● five (5) digits
● four (4) digits

751. What type of modulation is most commonly used for VHF packet radio transmission?
● SSB
● AM
● FM
● CW

752. When using Inmarsat-C, select from the following modes of communications are available?
● FAX Reception
● Fax and Voice
● TELEX and e-mails
● Voice transmission

753. What is the term that describes a transistors ability to amplify signal?
● Tune
● Forward voltage drop
● Forward resistance
● Gain

754. The initial report required from a ship entering a ship reporting system should generally be
limited to the following EXCEPT: ?
● Port of Destination
● Position
● Ships name
● IMO identification number

755. Who should encourage ships so that VTS be in use?


● Administrations
● IMO
● Port authorities
● AMVER

756. Which of the following is a form of amplitude modulation?


● Packet Radio
● Phase shift keying
● Single sideband
● Spread-spectrum

757. What is the time allowed for the AIS-SART to transmit messages after the activation?
● within 30 seconds
● within 1 minute
● within 2 minutes
● within 10 seconds

758. What does the abbreviation “LED” stands for?


● liquid ejecting diode
● liquid emitting diode
● light emitting diode
● lighter emission diode
759. If you receive a TELEX with the senders ID of 433863491. What type of terminal sen this
message to your vessel?
● Land TELEX Terminal
● Inmarsat-B
● Inmarsat-M
● Inmarsat-C

760. If one requires medical advice by means of an Inmarsat-C terminal, how should you use the
following address?
● 32
● MED+
● Medico
● Radiomedical

761. Which of the following circuits demodulates FM signals?


● Phase inverter
● Discriminator
● Product detector
● Limiter

762. In most GMDSS equipment, Which of the following is commonly used as visual indicator?
● LED
● Zener Diode
● FET
● Dimmer

763. Under specified conditions of immersion, what is the temperature for an AIS-SART to maintain
water tightness when subjected to thermal shock?
● 100 C
● 45 C
● 75 C
● 60 C

764. The initial report required from a ship entering a ship reporting system should generally be
limited to the following EXCEPT: ?
● IMO identification number
● Call sign
● MMSI number
● Ships Name

765. After transmitting a VHF DSC Distress Alert in sea are A1, which of the following RT channel you
should expect a subsequent correspondence?
● on ch. 13
● on ch. 16
● on ch. 06
● on ch. 70

766. For how long should a SART be in a watertight condition at a depth of 10 meters?
● At least 15 minutes
● At least 5 minutes
● At least 3 minutes
● At least 10 minutes

767. Identify from the following statement the transmitter frequency of MF/HF console DSC Distress
alert would be interpreted?
● Will always go out first on 2187.5 kHz
● Depends upon operator DSC Call set up entries or manufacturer’s defaults
● Will go out on 8MHz and 2 MHz and one other DSC Distress frequency
● Will always go out first on 8414.5 kHz to ensure receipt by a coast station

768. Where can be VTS be only mandatory?


● Freedom of the seas
● EEZ
● Contiguous zone
● Territorial seas

769. Which sideband is normally used for 10-meters HF, VHF and UHF single-sideband
communication?
● inverted sideband
● Lower sideband
● Suppressed sideband
● Upper sideband

770. Which of the following is NOT an example of a failure to log-out properly?


● Turning off the power prior logging out with the NCS
● Sailing out the footprints of the satellites before logging out
● A message on the screen or printer from the NCS
● Sailing the vessel into a shadowing or local RF interference situation before logging out

771. Which term describes the ability of a GMDSS transceiver to discriminates between multiple
signals?
● Selectivity
● Clarity
● Sensitivity
● Linearity

772. Which of the following equipment should be tested at least once a month and details entered in a
radio log?
● Each printer for adequate paper.
● NAVTEX
● SART, EPIRB, handheld VHF sets
● Reserve source if not a battery

773. What sea area is define as being within range of shore-based MF station that provides for
continuous DSC alerting?
● Coastal waters
● Sea Area A2
● Sea Area A3
● Sea Area A1

774. In INMARSAT operation, what is the two-digit code which corresponds to medical assistance?
● 32
● 38
● 41
● 42

775. What is the function of automatic gain control or AGC of digital communications receiver?
● To keep received audio relatively constant
● To protect an antenna from lightning
● To eliminate RF on the station cabling
● Am asymmetriconlometer control used for antenna matching

776. What is the emission or mode control of an MF/HF transceiver?


● Allows fine tuning of the receiver
● Reduce the receiver gain automatically
● Increases the range of the transmitter
● Allows the type of modulation to be selected

777. To indicate correct operation of the AIS-SART be equipped with the following means EXCEPT:
● Both audible and visual
● AUTOMATIC
● Visual
● Audible

778. What is the proper term used to describe a coast radio station that primary handles chargeable
ship-to-shore message traffic of a routine nature?
● Accounting Authority
● Network Coordination Station
● Mobile Radio Service
● Public Correspondence station

779. When measured directly above given point, solar radiation in the ionosphere is the greater or
maximum during what time?
● At sunset
● At night
● At sunlight
● At noon
780. What is the reporting interval capability of AIS-SART in transmitting messages?
● 2 minutes or less
● 10 minutes or less
● 5 minutes or less
● 1 minute or less

781. Which of the following is non operating DSC watch frequency?


● 12577.0 kHz
● 2187.5 kHz
● 6312.0 kHz
● 2182.0 kHz

782. Your vessel is in GMDSS Sea Area A1, drifting ashore and you required a tow, what do you initially
transmit by Ch.70 DSC?
● A distress Alert relay
● An all stations Safety call
● An all stations Station call
● An all stations Urgency call

783. Which of the following indicates a ship station MMSI?


● 623945326
● 003572946
● 548123400
● 623944326

784. For interaction with ships participating in the ship reporting system, what should be the equipment
of the shore-based authority compatible with the requirements of SOLAS Chapter IV?
● Provide with a booklet which shall be written in a language mutually agreed with ship/’s officer.
● Radiocommunications and other radio equipment appropriate to accomplish the objective of the
system.
● Standard marine communications phrases under development by the organization.
● Equipped with emergency starting devices approved by the International Maritime Organization.

785. What was the component of DSC Distress Alert contains?


● ID position and nature of distress
● ID position and name of vessel in distress
● ID position and number of crew onboard
● ID position and vessel’s call sign

786. Why should the transmitted power on VHF is to be reduced?


● To give maximum range
● To minimize interference
● To use more battery energy
● To increase the volume of the received signal
787. From the height can an AIS-SART be capable of withstanding without damage when dropped into the
water?
● 20 meters
● 25 meters
● 15 meters
● 30 meters

788. After using DSC to set up Routine RT communications between a ship and a coast station. who will be
the control station?
● Any coast guard station
● The nearest ship in the vicinity
● The coast station

● The nearest MRCC
799.What is meant by the term ‘’PTT’’?
● Precise tone transmission used to limit repeater access only certain signals
● A primary transformer tuner use to match antennas
● Pre-transmission tuning to reduce transmitter harmonic emission
● The push to talk function which switches between receive and transmit

800.Which of the following is correct when a paper model NAVTEX receiver runs out of paper?
● The unit is unable to print messages and all subsequent MSI broadcasts may be missed until the paper
is replaced.
● MSI messages may be missed because the unit cannot print them out.
● It may give off either an audible and/or visual alarm.
● The system will automatically change from receiving MSI by NAVTEX to receiving it by
safetyNETTM so that no messages will be lost.

801. The establishment and operation of a Ship Reporting System,the following should be taken into account
EXCEPT:
● The cost to the ship operators and responsible authorities.
● International as well as national responsibilities and requirements.
● Existing and proposed aids to safety.
● Duties and responsibilities of every crew on board

802. How would you explain the relay of DSC Distress alerts?
● Can quickly overburden the GMDSS system in the vicinity with improperly transmitted or
inappropriately relayed DSC calls.
● Was not originally an intended function of the GMDSS system but now is the preferred method to notify
an RCC.
● Remains the preferred method for passing Distress message traffic to an RCC or COAST station.
● Should always be done immediately to ensure a coast station receives the DSC Distress alert.

803.Which of the following is NOT an advantage of a 406 MHz satellite EPIRB?


● They can be used by mariners who do not know the Morse Code
● It is compatible with the COSPAS-SARSAT satellites and Global Maritime Distress Safety System
(GMDSS) regulations
● Provides a fast,accurate method for the Coast Guard to locating and rescuing person in distress.
● Includes a digitally encoded message containing the ship’s identity and nationality

804.Which statement support on the mutual interference on 518 KHz among NAVTEX stations?
● Transmissions scheduled on a time-sharing basis,power limited and station assignment codes
are geographically separated.
● transmitter power is limited,station assignment codeshare not shared by other NAVAREAS and stations
alternate between daytime and night time operations?
● Stations codes are not shared by other NAVAREAS, transmissions scheduled on a time sharing basis
and power limited.
● All stations transmit at the same time but stations are limited to daytime operation only to reduce te
radius of propagation

805.What is the aim of interchange of data between ships participating in a ship reporting system and a shore-
based authority?
● To check the navigational equipment working onboard.
● Increase safety for ships participating in a system.
● Enhancing maritime safety or the protection of marine environment.
● To poll the DSC unit of the vessel in distress on download the necessary information.

806. What data is at least requested in a DSC distress alert?


● The identification of the vessel in distress and the number of people on board
● The identification of the vessel in distress and her position
● The identification of the vessel in distress, her position and the nature of distress
● The identification of the vessel in distress, her position and port of destination

807. What is the purpose of the squelch control on a GMDSS transceiver?


● To set the transmitter power level
● To adjust the automatic gain control when no signal is being received
● To set the highest level of volume desired
● To mute receiver output noise when no signal is being received

808. Pick from the following license meets the requirements to perform or supervise the performance of at-sea
adjustments, servicing or maintenance which may affect the proper operation of the GMDSS station?
● GMDSS Radio Operators\’s License or Marine Radio Operators\’s Permit
● Marine Radio Operators\’s Permit or GMDSS Maintainer\’s License
● GMDSS Operators Maintainers License of GMDSS Maintainers License
● General Radiotelephone Operators License with Shipboard RADAR endorsement

809. What are the categories of ships required to participate in mandatory ship reporting system?
● Only ships not under command, restricted in their ability to maneuver or having defective navigational
aids
● Only ships with gross tonnage of 5,000 and above
● Any warship, naval auxilliaries or other ships owned or operated by a State only
● Any ship regardless of tonnage, flag or type as far as she is within the service area of the
system

810. In the EGC Safety net service, what type of NAVTEX warning is letter “A” represent?
● Meteorological Warning Information
● SatNav messages
● Navigational Warning Information
● Search and Rescue information

811.What is the correct format for ongoing “on-board” communications between substation and bridge?
● SEAQUEST BRIDGE(3 times) from SEAQUEST AFT (3 times)
● BRAVO this is SEAQUEST CONTROL
● SEAQUEST BRIDGE this is SEAQUEST TWO
● SEAQUEST CONTROL (3 times) this is SEAQUEST BRAVO (3 times)

812. Which of these would be vital to a GMDSS SAR situation in polar regions?
● COSPAS/SARSAT satellite to receive Distress Alerts and HF voice for follow-on and SAR
activity
● GOES satellites to receive Distress Alerts and Inmarsat voice for follow-on and SAR activity
● Inmarsat satellites to receive Distress Alerts and HF voice follow-on and SAR activity
● GOES satellites to receive Distress Alerts and HF voice for follow-on and SAR Activity

813. How would you describe on the following statement concerning Inmarsat-B and Inmarsat-C terminal is
CORRECT?
● Inmarsat-B units are incapable of data communications, but inmarsat-C units are capable of data
communications
● Both Inmarsat-B and Inmarsat-C units are capable of fax and voice communications
● Inmarsat-B units are capable of data communications, but Inmarsat-C units are incapable of data
communications
● Both Inmarsat-B and Inmarsat-C units can send data as well as send messages to fax machines

814. How would you describe GMDSS is primarily a system appertaining to?
● The linking of search and rescue authorities ashore with shipping in the immediate vicinity of a
ship in Distress or in need of assistance
● Ship-to-ship Distress communications using MF or HF radiotelephony
● Distress, Urgency and Safety communications carried out by the use of narrow-band direct printing
telegraphy
● VHF digital selective calling from ship to shore
815. How would you demonstrate your action after sending a false distress alert on 12577.0 kHz?
● Send a message to the nearest RCC via Inmarsat canceling the alert
● No action is necessary
● Make an “ALL STATIONS” call on all 5 HF TELEX frequencies canceling the alert
● Make an “ALL STATIONS” call on the associated 12 MHz J3E frequency canceling the alert

816. How would you demonstrate your action after sending a false or accidental Distress alert on Inmarsat-C?
● Select “Transmit” or “SEND/REC” menu and send a cancellation message via the LES used for
the Distress alert
● Do nothing until the RCC contacts your vessel to determine if the Distress alert was genuine
● Select the cancel false Distress alert option in the Distress Setup and re-transmit the call
● Press the “Distress Hot Key” then press the “cancel” key

817. Which is the message type with the highest priority?


● Radio Telex Leter (RTL)
● Request for berthing instructions
● Telephone call to owners
● Gale warning

818. What would be your action after you have received an MF DSC Urgency call?
● Acknowledge by RT on 2182 KHz
● Listen for the message on 2177 KHz
● Listen for the message in 2182 KHz
● Acknowledge by DSC on 2187.5 KHz

819. How should communications be conducted on ship reporting, taking into account any other relevant
guidelines, criteria, regulations or instruments developed by IMO?
● In conformity with IMO resolution A 851 (20)
● In conformity with IMO resolution A 401(92)
● In conformity with IMO resolution A 433(20)
● In conformity with IMO resolution A 356(92)

820. Choose from the following the best description for a shipboard Inmarsat-C system?
● It is a satellite communications system that also provides continuous digital Selective Calling coverage
for all ocean regions
● It is a small, lightweight terminal capable of providing satellite store-and-forward message
communications
● It is a small, light weight terminal used to transmit messages over high frequency bands to
communicate through a satellite
● It is a satellite communications system that provides real-time connectivity

821. Which of the following maintenance functions is NOT the responsibility of the GMDSS Radio Operator?
● Perform scheduled testing of the battery’s charge condition
● Visual inspection of equipment, including the antenna and associated components
● Aligning the power output stage for maximum power
● Perform on-the-air verification checks

822. What is an “Adopted Ship Reporting System”?


● Established by a government/s after it has been accepted by the Organization as complying
with all requirements of SOLAS regulation VII
● Interchange of data between ships participating in a system and a shore-based authority, aimed at
enhancing maritime safety or protection of marine environment
● Designated by a Contracting Governments with the responsibility for the management and coordination
of a system, the interaction with participating ships, and the safe and effective operation of a system
● Implemented by a competent authority designed to improve the safety and efficiency of vessel traffic
and protect the environment

823. In a distress follow on voice transmission after a DSC Alert, what information should be included?
● Ship’s position Ship’s IMN, the nature of distress and assistance requested
● Ship's Name and Call sign, MMSI number and position, the nature of distress and assistance
requested.
● Ship’s Name and call sign, repeat IMN, provide any other information that might facilitate rescue.
● Ship’s Name and call sign, MMSI number, DSC frequency used and any other information and might
facilitate rescue.

824.If a NAVTEX warning message is received but it contains too many errors to be usable,what should a
GMDSS Radio Operator’s action?
● Listen to the appropriate VHF weather channel for repeat warnings.
● Contact the NAVAREA coordinator and request a repeat broadcast.
● Ignore it. Vital NAVTEX messages will be repeated on the next scheduled broadcast.
● Initiate a request for Category A,B,L and D messages

825.How would you explain’’ radio silence’’being imposed?


● By the Land earth Station (LES) controlling the Distress communications on frequency.
● By the On Scene Coordinator (OSC) or the RCC chosen by the SAR Mission Coordinator
● By the vessel first responding to the distress call
● By the nearest Public Correspondence Coast Station

826. Describe the main idea of transmissions on 406 MHz?


● NAVTEX English-language broadcasts
● EPIRB satellite position fixing
● Two-way RT communication with life raft during SAR operations
● MF telex distress, Urgency and Safety message

827. Which of the following is NOT normally found on EPIRBs that are detected by satellites?
● Radio Operator programs ID on EPIRB for immediate prior activation
● A bracket designed to allow EPIRB to automatically float free
● Transmit a unique hexadecimal identification number
● Emergency transmission on 406-MHz beacon

828. Your ship is a GMDSS ship, what is required on the capability of the VHF installed onboard the ship?
● The DSC channel 70 must be able to initiate transmission of distress alerts from position to
which the ship is navigated
● The VHF transmitter output must be 30 watts for better output signals
● The VHF must have a dedicated non scanning receiver capable of monitoring distress on DSC channel
on VHF 16
● A required channel for radio telephony (transmit and received) of distress on any VHF channel
frequency

829. Which is the best answer on the following requirements regarding radio telephone stations does not
conform to the present regulations?
● The radiotelephone station shall be located in the upper part of the ship, sheltered to the greatest
extent from noise impairing reception of messages/signals
● There shall be an efficient communication between the radio station and the bridge
● The instruction card should give a clear summary of the distress procedure and shall be displayed at
the operators position
● While at sea the installation shall be maintained in a satisfactory condition

830. What is the requirement for emergency and reserve power in GMDSS installations?
● All newly obstructed ships under GMDSS must have both emergency and reserve power
sources for radio communications
● A reserve power source is not required for radio communications
● An emergency power source for radio communications is not required if a vessel has proper reserve
power (batteries)
● Only one of the above is required if a vessel is equipped with a second 406 EPIRB as a backup means
of sending a Distress alert

831. What is the proper format for a Distress follow on voice transmission (3x is three times)?
● All ships 3x, this is Ship’s name/Call sign 3x, Ship’s position, nature of distress and assistance
requested
● All station 3x, this is Ship’s name/Call sign 3x, Ship’s position, nature of distress and assistance
requested
● Mayday 3x, this is Ship’s name/Call sign 3x, Ship’s position, nature of distress and assistance
requested
● Mayday 3x, this is Ship’s name/Call sign once, Ship’s position, nature of distress and assistance
requested

832. How would you explain the relay of DSC Distress alerts?
● Should always be one immediately to ensure a coast station receives the DSC Distress alert
● Remains the preferred method for passing Distress message to an RCC or Coast stations
● Was not originally an intended function of the GMDSS system but now is the preferred method to notify
an RCC
● Can quickly overburden the GMDSS systems in the vicinity with improperly transmitted or
inappropriately relayed DSC calls

833. How would you explain the meaning of “Reserve SOurce of Energy”?
● Diesel fuel stored for the purpose of operating the powered survival craft for a period equal to or
exceeding the SOLAS requirements
● Power to operate the radio installation and conduct Distress and Safety communications in the
event of failure of the ship’s main and emergency sources of electrical power
● The diesel fueled emergency generator that supplies AC to the vessel’s emergency power bus
● High caloric value for lifeboat, per SOLAS regulation

834. Which of the following is a non functioning satellite under COSPAS-SARSAT using satellite EPIRB?
● Doppler shift of EPIRB signal is measured and the EPIRB’s position is calculated.
● Relayed satellite message includes the EPIRB ID number which provides a reference for retrieval of
vessel information from the shore database.
● Information received from EPIRB is time-tagged and transmitted to any Local User Terminal in
satellite’s view.
● After the EPIRB’s position is calculated using the Doppler shift COSPAS-SARSAT satellite
provide follow-on SAR communications.

835. What would be the most appropriate HF bands for communication from San Francisco to Taiwan or the
Philippines?
● 16 or 22 MHz daylight at each end and MHz when dark at each end.
● 4 or 6 MHz during daylight hours and 8 MHz during darkness.
● 12 MHz during daylight at each end and 2 MHz when dark at each end .
● 8 MHz during daylight hours and 16 MHz during darkness.

836. Select from the following statement GMDSS communication function must all compulsory vessels be
capable of performing to meet International Maritime Organization requirements?
● Distress alerting, general communication and receipt or Maritime Safety Information in the
vessel’s intended area of operation
● Distress alerting and receipt of MSI in Sea Areas A1 A2 A3 and A4 regardless of the vessels intended
area of operation
● General communications via Inmarsat and receipt of Maritime Safety Information via Enhanced Group
calling in Sea Area A4
● Distress alerting and receipt of Maritime Safety Information via Inmarsat for all vessels Intending to
operate in Sea Area A4

837. What are the categories of ships required to participate in mandatory ship reporting system?
● Any warship, naval auxiliaries or other ships owned or operated by a State only
● Any ship regardless of tonnage, flag or type as far as she is within the service area of the
system
● Only ships not under command, restricted in their ability to maneuver or having defective navigational
aids
● Only ships with gross tonnage of 5,000 above

838. What action would you take if you are in voice communication on Ch-16 with a vessel in Distress that
advises you they are unable to contact a Coast Station?
● Send a DSC Distress Relay transmission on Ch-16
● Make an all ships call with Urgency priority
● Attempt to contact a coast Station using voice on Ch-16 with a Mayday Relay
● Call the Coast Station on CH-70 with Distress priority giving the other vessel’s position

839.Concerning the EPIRB system,which of the following statements is authentic?


● GOES weather satellites will provide alerting with complete world wide coverage.
● COSPAS-SARSAT satellites always provides an alert and position report within 10 minutes of reception
● The Inmarsat system will not provide alerts and position report for 406 MHz EPIRB equipped
with GPS receivers
● The GPS satellite system will relay an alert and position report within 20 minutes of reception.

840. What is one of the advantages of using J3E?


● Less current is drawn from the batteries and H3E
● The signal is less efficient than R3E
● Less fine tuning of the receiver frequency
● The receiver signal is clearer than H3E

841. With reference to SOLAS IV,Regulation 2, means a technique using digital codes which enables a radio
station to establish contact with and transfer information to another station or group of stations, complying
with the relevant recommendations of the International Radio Consultative Committee (CCIR). which from the
following conform with the term and definition of regulation?
● Direct Selective Calling
● Digital Selective Calling (DSC)
● Direct selection calling
● Digital Selection Calling

842. Which of the following statements concerning EPIRB alerts is FALSE?


● The COSPAS-SARSAT system may take a full hour or more to provide an alert.
● 406 MHz EPIRB units may be equipped with GPS receiver.
● The INMARSAT system provides worldwide coverage for Distress alerts
● The GOES weather satellites are in geostationary orbit.

843. According to International Regulations ,what is the other use of VHF Ch. 06?
● For ship to aircraft engaged in SAR operations
● For ship to shore communications
● for on board communications
● for routine DSC alerting

MISMO REVIEWER COMPETENCE 2


1. Your vessel suffered a fire and it is out of control. You were ordered by the Master to make a
distress call. How will you initiate a call by radiotelephony?
● MAYDAY (3x)
● SOS (3x)
● SECURITY (3x)
● PAN PAN (3x)
2. Your ship received a distress relay on DSC frequency 2187.5 kHz that a vessel is on fire. On
what frequency by radio telephony would you acknowledge?
● 4207.5kHz
● 2182.0 kHz
● 6312.00 kHz
● 8290.0 kHz

3. On a survival craft, you hear the SART’s Audible tone, what does it mean?
● It is a sign when the battery charge condition is weak
● It informs the survivors that a near by vessel is signaling on DSC
● It informs the survivors when the SART switches to the “standby mode”
● It is informing you that assistance may be nearby

4. Why must SART be mounted as high as possible in the survival craft?


● To be seen better on the radar
● Because it increases the power
● So that it will not get wet
● To increase the range

5. Your vessel losses Inmarsat SES operation after a large course change? could cause this?
● Misalignment of the shadow correction filter
● System loses gyro follow-up or a gyro system malfunction
● Shadowing of SES antenna by clouds or other weather formation
● Misalignment of the SES antenna and the LES

6. During abandon ship, which of the following types of EPIRBs is most likely to be used to transmit
a distress alert signal?
● S-band EPIRBs
● 121.5/ 124.3 MHz EPRIBs
● Class A EPRIBs
● 406 mhz EPRIBs

7. Under GMDSS , at what minimum intervals a compulsory VHF-DSC radio telephone installation
must be tested?
● Once a week
● Daily
● Monthly
● Weekly

8. Which of the following is “NOT” allocated alerting in the Global Maritime Distress and Safety System?
● 1626.5 kHZ- 1645.5 kHz (INMARSAT)
● 405MHz (EPIRB)
● DSC on 156.525 MHz (channel 70) and various MF/HF frequencies
● 415kHz and 526.5 kHz

9. Which channel is designated for GMDSS digital selective calling?


● Channel 70 (156.525 MHZ)
● 406 Mhz
● 2182.0 KHZ
● Channel 6

10. What should be the information content of GMDSS distress alerts such as a fire onboard?
● The distress alert message should contain the name and position of the distress vessel
● The distress message contain is automatically program when the distress key is press
● The distress message should contain information about flag and port of registry
● The distress message should contain the nature of the distress

11. What is your main function if your vessel is the designated-On Scene Co-ordination with respect
to Distress communication?
● Deploy a SART to assist in homing casualties
● Fixed the position of the casualty and advice all participating SAR facilities
● You should maintain communication with all Search and Recue (SAR) facilities
● Recover the EPIRB and switch it off so that it stop sending Distress signal

12. Which of the following indicate proper operation of a SSB transmitter rated at 60 watts PEP output
in J3E mode?
● A steady indication of transmitter energy on RF power meter with no fluctuations
when speaking into the microphone
● In SSB (J3E) voice mode, with the transmitter keyed but without speaking into
the microphone no power output is indicated
● In SSB (J3E) mode, speaking into microphone causes no power meter fluctuate on aboce
the 60 watts reading
● In communication the power meter can be seen fluctuating regularly from zero to the 60
watts relative output reading.

13. What is usually the first step of GMDSS radio operator to take when initiating a distress priority
message via INMARSAT?
● Edit distress message
● By pressing the security alert safety system (SSAS) button in the Master’s Cabin
● Access the File Menu
● By pressing a distress button or hot key(s) on the equipment

14. What action should you take after sending a false Distress Alert on 2187.5kHz indicating J3E?
● Reset the DSC controller and send a cancellation message to all station on 2182Khz
● Reset the DSC controller and no need to send cancellation message
● Switch off the equipment for 30 minutes and it will automatically reset
● Reset the DSC controller and message to all station on VHF Ch. 16

15. If you ship is on fire, what priority signal will you use in sending your message by FAX?
● SAFETY
● DISTRESS
● URGENT
● EMERGENCY

16. To ensure that all necessary MSI has been received, when should the NAVTEX receiver be
switched on?
● At least 4 hours before sailing time
● At least hours before sailing time
● At least 3 hours before sailing time
● At least 1 hour before sailing time

17. Which of the following units as per IAMSAR is responsible for promoting the efficient organization
of search and rescue?
● Rescue Coordination Center (RCC)
● Ship Earth Station (SES)
● Coordination Surface Search (CSS)
● On-Scene Commander (OSC)

18. If you abandon your ship and boarded a survival craft, which of the following step you should
prioritize?
● Nothing to do as EPIRB and SART activated automatically when remove form
securing bracket
● Throw the EPIRB and SART in the water to automatically activated
● Make ready the EPIRB and SART activate when another vessels as in sight
● Ensure that the EPIRB and SART is activated until help arrived

19. What is the substance produced during the battery


20. changing process which requires battery charging rooms to be well ventilated?
● Highly combustible oxygen
● Corrosive gases
● Highly poisonous gas
● Explosive gases

21. What should the GMDSS Radio Operator consult in the proper procedure to be followed in
distress situations under GMDSS?
● Part 80, Subpart of the FCC rules and Regulations
● Part 90 the FCC Rules and Regulations
● The IMO recommendation
● The manufacturer’s instruction manual

22. What is the signal you must transmit to locate your position using your mobile unit
● Alerting signal
● Radio beacon signal
● Radio direction signal
● Homing Signal

23. You are preparing to abandon your ship, what type of signal you must transmit to locate you r
position using your mobil unit?
● Homing signal
● Alerting Signal
● Radio Direction Signal
● Radio Beacon

A storms hits your ship, causing your MF-HF whip antenna to shatter. What would you do if
you needed to reclaim MF-HF frequencies?
● Rig a wire antenna 10-15 ft long from the antenna tuner to the highest vertical support
● Rig a horizontal dipole antenna 40 feet long
● Rig a wire antenna approximately 35-75 feet long per the equipment instruction manual
● Rig a long wire antenna at least 200 feet long
24. What is usually the first step for a GMDSS Radio Operator to take when initiating a distress
priority message such as fire onboard via INMARSAT?
● Contacting the CES operator using the radiotelephone distress procedure Mayday …etc
● Contacting the CES operator and announcing a distress condition is in existence
● Pressing a Distress Key on the equipment
● Dialing the correct code on the telephone remote unit

25. What happen if your vessel is on fire and you begin a DSC distress transmission but do not include
a message?
● The transmission aborted and alarm sound will indicated that will advice you to
provide information
● The transmission will be made “default” information provided automatically
● The transmission will not be initiated and “ERROR” will indicate on the display read out
● The receiving station will poll the DSC unit of your vessel to download the
necessary information

26. What is term which refers to the supply of electrical energy required to supply radio installations
on every ship for the purpose of conducting distress and safety radio Communications in the event
of failure of the ship’s main and emergency sources of electrical power?
● Ship\’s emergency diesel generator
● Emergency power
● Ship\’s standby generator
● Reserve source of energy

27. If an Inmarsat Distress Alert set up menu contains an incorrect or unavailable LES code, what will
happen to the distress transmission?
● The NCS wll update the LES code in the distress alert set up menu
● If the LES code is incorrect the distress alert will not be detected by any RCC
● The NCS will step in and route the Distress alert transmission to an RCC
● The NCS will send a distress alert set incorrect and the LES code alarm will activate

28. Which of the following maintenance function may not be performed by the holder of a GMDSS
Operator License?
● Any adjustments of maintenance that may affect the proper operation of the station
● Replacement of consumable items such as paper, ribbons, etc.
● Routine battery maintenance if used as part of GMDSS station
● Reset tripped circuit breakers or replace defective fuses

29. What protection should you wear when performing GMDSS equipment back up power battery
maintenance?
● You should wear face mask and gloves
● You should wear eye protection, gloves and apron
● You should wear cover all and gloves
● You should wear eye protection and cover all

30. When receiving signals using emission class J3E what tuning device is adjusted until the voice
is clearly audibly heard in the loudspeaker?
● Clarifier Control
● Squelch Control
● RF Control
● Volume Control

31. In the proper setting up and safe operation of radio equipment, what publication you should consult?
● Code of Federal regulations
● The manufacturer’s instruction manual
● FCC rules and regulation
● ITU Manual

32. When making a DSC public correspondence, how long would you wait before attempting to make
a second with a station which does not answer?
● 3 minutes
● 15 minutes
● 10 minutes
● 5 minutes

33. What is the NAVTEX message which ensure that the system is functioning normally?
● G
● Z
● A
● B

34. You are about to abandon your ship as GMDSS Radio Operator, what is the first step to take
in initiating a distress priority message via Inmarsat?
● By pressing a “Distress Button or Distress Hot key” on the equipment
● By contacting the CES operator using the radiotelephone distress procedure “Mayday” ..etc
● By contacting CES operator and announcing a distress condition is existence
● By dialing the correct code on the telephone remote unit

35. You are combating fire engine room and the Master order you to send a distress call. Why is it
important to transmit MAYDAY or PAN-PAN signals even if not in distress?
● To alert the consignee/s that their cargoes may not arrive on time
● To give warning to other ship to keep clear of your vessel
● It will alert shipping to a possible future distress call
● To advice other station that you require assistance

36. Which statements is correcting regarding the requirements for emergency and reserve power
in GMDSS radio installations?
● Only one of the above is required if a vessel is equipped with second 406 EPIRB as a
backup means of sending a distress alert
● An emergency power source for radio communication is not required if a vessel has
proper reserve power (batteries)
● A reserve power source is not required for radio communications
● All newly constructed ships under GMDSS must have both emergency and
reserve power Sources for radio communications
37. What steps should you take after abandoning a ship and boarding a survival craft?
● ensure that the epirb and sart are activated until help arrives
● throw the epirb and sart in the water to automatically activate
● nothing to do as epirb and sart is activated automatically when removed from securing bracket
● program the sart and epirb to transmit the vessels location and situation

38. What step should you take after abandoning your ship and boarding a survival craft?
● Program the SART and EPIRB to transmit the vessel’s location and situation
● Ensure that the EPIRB and SART are activated until help arrived
● Nothing to do as EPIRB and SART is activated automatically when remove from
securing bracket
● Throw the EPIRB and SART in the water to automatically activated
39. You had abandon your vessel of distress situation, which of the following should be taken on board
a survival craft?
● Place the SART and EPIRB in the “ON” position and secure them to the survival craft
● Nothing to worry as the SART and EPIRB will both float free and operate automatically
● Program the SART and EPIRB to transmit the vessel location and nature of distress
● Throw the SART and EPIRB in the water to activate automatically

40. What will you do when a received Distress Alert from a vessel on fire is requesting J3E?
● Immediately set a continuous watch on VHF Ch. 70
● No action is necessary as the DSC control unit will automatically switch to the SITOR
(NBDP) follow on frequency
● Immediately set continuous watch on the radio telephone frequency that is
associated with frequency band on which the Distress alert was received
● Immediately set a continuous watch on the SITOR (NBDP) frequency that is associated
with frequency band on which the Distress alert is received
41. Which of the following as per IAMSAR is responsible for promoting the efficient organizing Search
and Rescue Service?
● Rescue Coordination Center (RCC)
● On-Scene Commander (OCS)
● Coordination Surface Search (CSS)
● Ship Earth Station (SES)
42. For distress call such as fire onboard, how will you initiate a call on VHF?
● Initiate a call on VHF Channel 70
● Initiate a call on VHF Channel 13
● Initiate a call on VHF Channel 69
● Initiate a call on VHF Channel 16

43. If your vessel is on fire and requiring immediate assistance, what is the two letters signal you should
display?
● NC
● AL
● CD
● PM
44. What are the conditions, under GMDSS, whereby a ship is NOT allowed to depart from any port?
● The vessel has a temporary waiver of its radio license and safety certificate
● The vessel is not carrying a GMDSS radio maintainer, but has provided for shore
side maintenance
● The vessel is carrying more than the required number of qualified GMDSS radio operators
● The vessel is not capable of performing all required distress and safety functions
45. What must be the characteristics of the Reserve Source of Energy under GMDSS?
● It must be incorporated into the ship’s electrical system
● It must be independent of the propelling power of the ship
● It must be independent of the ship’s electrical system
● It must have power a supply independent to HF and MF installations at the same time

46. Your antenna tuner becomes totally inoperative. What would you do to obtain operation on 2
Hf bands?
● To bypass the antenna tuner. Use a straight whip or wire antenna approximately 30
ft long
● To bypass the antenna tuner and shorten the whip to 15 ft
● Without an operating antenna tune transmission is impossible
● It is impossible to obtain operation on 2 different HF bands, without an operating antenna tuner
47. You are approaching another vessel and you see that the signal flag “J” is hoisted, what should you
do?
● Proceed on present course and speed since the vessel is requesting a tug
● Attempt to call the vessel on VHF radiotelephone because she is requesting to communicate
● Keep well clear of the vessel because she is either on fire and has dangerous cargo
on board or she is leaking dangerous cargo
● Approach the vessel with caution because she is dragging her anchor

48. If your vessel is on fire and requiring immediate assistance, what frequency you should use?
● Use only the frequencies that are within your operating privileges
● Use only the frequencies used by police, fire or emergency medical services
● What ever frequency that has the best chance of communicating the distress message
● You should send on 2182kHz as it is a designated distress frequency
49. The SART is required to have the sufficient battery capacity to operate in the stand-by mode for what
period of time?
● Six days
● Four days
● Three days
● Five days
50. You are on the vicinity of the search area of an abandon ship. How can you detect that a SART
is transmitting in the immediate vicinity?
● The SART signal appears as a target which comes and goed in the effects of heavy swells
● The SART can provide an approximate position location within a two nautical per
IMO Standard
● The SART’s blips on the PPI will begin arcing and eventually become concentric circles
● A unique two tone signal heard upon the automatic un-muting of the 2182 khz receiver
51. Which statement concerning GMDSS distress alerts such as fire onboard is True?
● The information in distress alert is advising vessels in the vicinity of a distress situation to
leave the area without notifying the RCC that is overseeing the operation
● The information in distress alerts that is used to alert other vessels, including those in port
of existing weather warnings
● The information contained in a distress alert includes the name and position of
the distressed vessel and may include additional information such as nature of
the situation and what kind of assistance that may be required
● The information contained in a message that alerts other vessels and may include additional
information on existing navigational hazards in ports
52. Which statement is correct regarding a method that a vessel experiencing problems with
shadowing of the Inmarsat B SES antenna by an onboard obstruction could use to attempt reliable
communication?
● Change the Coast Station ID programming
● Change course to make the communication
● Change to other satellite region
● Switch from telex to voice mode with require lower signal strength for proper operation
53. Onboard a liferaft which piece which piece of required GMDSS equipment is the primary source
of transmitting locating signals?
● An EPIRB transmitting on 406 MHz
● A SART in transpond mode
● Radio Direction Finder (RDF)
● Survival Craft Transceiver

What piece of required GMDSS equipment is the primary source of transmitting locating signals in
the event of equipment failure?
 SART
 Handheld Transciever
 RDF
 EPIRB

54. In using a GMDSS communication equipment what is the most common practice that a radio
operator must learn?
● Send test transmission another ship using 2187.5 KHZ
● Transmit without identification
● Monitor the channel before transmitting
● Silence the alarm to prevent noise
55. When listening to the transmission of a very close station what step must be observed to prevent the
strong signal from “smothering” the receiver?
● Adjust volume control
● Switch off speaker and use headset
● Use headset and adjust the squelch
● Lower power output
56. When a DSC Distress Alert is received requesting J3E by the other vessel on fire, what should you
do as a GMDSS RADIO Operator?
● No action is necessary as the DSC control unit will automatically switch the NBDP on
a follow-on-communication frequency
● Immediately you should set a continuous watch on VHF channel 70
● Set a continuous watch on the radio telephone frequency corresponding with
the frequency band where the distress warning was received right away
● Set a continuous watch on the NBDP frequency that is associated with frequency band
on which distress alert was received right away

57. There are no signal or static heard in the receiver of the ssb transceiver although a loud noise
is heard if the volume control is advanced all the way up which of the following is not the
possible cause?
● The frequency is not properly set
● the antenna switch has been transferred to the test position
● a defective receiver
● the antenna of feed line has been damaged

58. There are no signal or static heard in the receiver of the ssb transceiver although a loud noise
is heard if the volume control is advanced all the way up which of the following is not the
possible cause?
● the frequency is not properly set
● the antenna switch has been transferred to the test position
● a defective receiver
● the antenna of feed line has been damaged

59. Your vessel is on fire and requires immediate assistance. How will you start your vhf call?
● mayday 3x
● security 3x
● pan-pan 3x
● sos 3x

60. When your vessel is on fire what is the rapid and successful reporting of a distress incident to a unit
which can provide coordinated assistance?
● Distress alerting
● Safety alerting
● Public alerting
● Urgency alerting

61. Which channel and mode should you use when initiating a distress alert transmission such as
fire onboard?
● Channel 70 dsc
● Channel 60
● Channel 6 radio telephony
● Channel 13 radio telephony and channel 16 dsc

62. What frequency should you use if your vessel is on fire and requires immediate assistance?
● Whatever frequency that has the best chance of communicating the distress message
● Use only the frequencies used by police, fire or emergency medical services
● You should send on 2182 khz as it is a designated distress frequency
● Use only the frequencies that are within your operating privileges

63. In proper setting up and safe operation of radio equipment what publication should you consult?
● The manufacturer's instruction manual
● Itu manual
● Fcc rules and regulation
● code of federal regulation

64. Why is it important to transmit a distress message when you have fire on board?
● The distress message is necessary to alert other vessel and authorities
● The distress message is not necessary because your situation is still on uncertainty phase
● The distress message is necessary so that other vessel can approach you and give assistance
● The distress message is not necessary if you don't need immediate assistance
65. A fire occurred onboard your ship and you need assistance in sending a dsc message, how will you
categorize your message?
● distress
● safety
● routine
● Precautions

66.If your antenna tunner becomes totally inoperative.what would you do to obtain operation on 2 HF Bands?
 It is impossible to obtain operations on 2 different HF bands without an operating antenna.
 Without operating antenna tuner,transmission is not possible.
 By pass the antenna tuner and shorten the whip to about 15 feet.
 By pass the antenna tuner.User a strait whip or wire antenna approximately 30 feet long

67.During charging process what does it produce that require battery charging room to be well ventillated?
 Explosive gasses
 Highly combustible
 Highly poisonous gas
 Corrosive gasses

68.In a fully charged lead-acid battery. What will be the electrolytes freezing point?
 Lower than a discharged battery.
 Higher than in a discharged battery
 The same as in a discharged battery
 Higher than in a discharged battery, but the specific gravity will be less

69.How would you measure the current drawn from a power supply by a receiver
 The current meter should be placed in series with one of the receiver power leads
 The current meter should be placed in parallel with one of the receiver power leads
 The current should be placed in parallel series with both poer receiver power supply leads
 The current meter should be placed in series with both receiver power leads.

70.You observed tha maximum usable frequencies varies during transmission,what mostlikely causes the
effect?
 The amount of radiation received from the sun,mainly ultraviolet.
 The type of weather just below the ionosphere.
 The speed of the winds in the upper atmosphere.
 The temperature of the atmosphere.

71.Which of the following maintenance function is not resposibility of the GMDSS Radio Operator?
 Visual inspection of equipment including antenna and associated components.
 Perform schedule testing of the battery charge condition
 Perform on the air varification checks.
 Aligning the power output stage for maximum power.

72.What should be the information content of GMDSS distress alerts,such as fire onboard?
 The distress message contain is automatically program when the distress key is press.
 The distress alert message should contain the name and position of the distress vessel.
 The distress message should contain information about flag and port of registry.
 The distress message should contain the nature of the distress.

73.You have a very loud low frequency hum appearing on your transmission. In what part of the transmitter
would you first look for trouble?
 The variable frequency oscilator
 The power supply
 The driver circuit
 The power amplifier circuit

74.What is the term which refers to the supply of electrical energy required to supply radio installations on
every ship for the purpose of conducting distress and safety radio communications in the event in the
event of failure of the ship’s main and emergency sources of electrical power?
 Emergency power
 Reserve source of energy
 Ship’s standby generator
 Ship’s emergency diesel generator

75.What is your responsibility under a GMDSS equip vessel in a distress situation?


 If your ship is over 300GT you can be required to render assistance if such assistance will not effect your
schedule.
 In distress situation you must be able to perform communications function that are essential for
the safety of your ship itself and of other ship.
 If outside of service areas you may be required to render assistance in Distress situation.
 If you are close to a vessel in distress regardless of your vessel size you must render assistance

76.What are the hazard associated with lead acid batteries that you must be aware?
 Risk of explosion and corrosion
 Risk of exposure to poisoning
 Risk in exposing to circuit
 Risk in exposing to chemical

77.Which of the following action should be taken once the vessel is berthed and will not leave port again for
several week?
 The GMDSS operator must transmit an all ship alert to notify vessel within the sattelite footprint that the
vessel will be offline.
 The GMDSS operator must notify the NCS that the vessel will be offline and wait for the NCSto
acknowledge with with confirmation number that must be logged.
 The GMDSS operator may log off the Inmarsat C system and turn the power off, unless the vessel
decides to leave the unit on during the port stay.
 The Inmarsat C system can be powered down without taking additional steps once the GMDSS operator
has ensured that all incoming safety net message have been received and stored.

78.How would yu describe a temperature effect on the specific gravity of lead acid batteries?
 Higher temperature results in a lower specific gravity reading
 Lower temperature results in a higher specific gravity reading
 Lower temperature results in a lower specific gravity reading
 Higher temperature results in a high specific gravity reading

79.Which of the following falures must be logged and notified to the Master when using 2182 khz
radiotelephone system?
 Failure to contact a shore station 600 nautical miles distance dueng daytime operation.
 Much higher noise level observed during daytime operation.
 When testing alarm to an artificial antenna the distress frequency watch receiver become unmuted.
 When talking to a microphone no power indicator output.

80.At present the MF/HF trasmitter often has an automatic aerial turning unit.Should this fail:
 The transmitter will automatically keep operating ON MF and HF distress frequencies
 The transmitter will automatically keep operating OFF the MF and HF distress frequencies
 It is not possible to put the turning unit I a fixed position ,so the MF distress frequencies can be used
 Its always possible to put the turning unit in a fixed position,so the MF distress frequencies can
still be used
81.You are onboard a 500GT,how many SART must be installed to your ship?
 None
 Two
 One
 As required by administrator

82. How can the satellite signal be immediately remedied by means of a shipboard obstruction?
 Raise the transmit power level
 Chage the ship’s course
 Increase the receiver again
 Relocate the mast or the obstruction

83. Under what condtion you should relay a DSC Distress alert?
 You should never relay a Distress alert, the Coast Station and RCC will do that
 If the mobile unit in Distress is incapable of sending further distress alert communication
 When the coast station determined fruther help is necessary
 You can relay anytime if you feel deem necessary

84. Who normally acknowledge a DSC distress lert mesage?


 Any station which heard it
 The first station that received it
 Rescue Coordination Center
 Coast Station

85. What is the mos likely reason for a message being erase from NAVTEX memory?
 The message are older than number 150
 The unlock message is not set on
 The lok message is set on
 The 66 hours passed from the moment received

86. What will you do when a received Distress Alert from a vessel on fire is requesting J3E?
 Immediately set a continuous watch on the SITOR (NBDP) frequency that is associated with frequency on
which the Distress alert is received
 No action is necessary as the DSC control unit will automatically switch to the SITOR (NDBP) follow on
frequency
 Immediately set a continuous watch on VHF Ch. 70
 Immediately set a continuous watch on the radio telephone frequency that is associated with
frequency band on which the Distress alert was received

87. Why is it important to transmit a Distress message when you have fire onboard?
 The distress message is not necessary because your situationis still on uncertainly phase.
 The distress message is not necessary if you dont need immediate assistance
 The distress message is necessary so that other vessel can approach you and give assistane
 The distress message is necessary to alert other vessel and authorities

88. What is the approximate lifetime of the marine leads batteries?


 2 years
 5 years
 4 years
 3 years

89. Your vessel is on fire and you require immediate assistance, how will you initaite your cll using the VHF?
 SECURITEE spoken three times followed by the nature of your message
 PAN-PAN spoken three times followed by your message
 Press the distress key on the equipment ad send your message
 MAYDAY spoken three times followed by the nature of your message

90. What is the reason why you should minimized function testing of a SART?
 Minimize power consumption and on test at sea to reduce potential interference and confusion
 Possibility of misinterpretation by other vessels as a distress situation
 Potential interference with safe navigation, possible misinterpretation of actual distress, and
minimizes draining of the battery
 Only test in port to prevent potential interference with a safe navigation at sea

91. When the fire onboard is already extinguished, what is the necessary message that you should send?
 A cancellation message of the previous alert message
 An advice message to update the alert message
 An allert message that fire is already extinguished
 Urgency message that fire onboard is already extinguished

92. What personal protective equipment you should use during maintenance of radio eqipment batteries?
 Eye protection and cover all
 Eye protection gloves and apron
 The normal working clothes
 Gloves and cover all to protect the hand

93. After the fire incident onboard your ship upon power-up, what controls are to be adjusted on an Inmarsat
C terminal?
 No control to be adjusted
 The Inmarsat terminal must be set to ocean region
 The receiver gain must be adjusted to maximum
 The antenna azimuth and elevation control

94. What is the usual effect of the ionoshere storms?


 It prevent communication by groundwave
 I produce extreme weather changes
 It increase the maximum usable frequency
 It causes a fade-out of skywaves signal

95. What is the cause of the LCD brilliance that cannot be raise?
 The life of the LCD has expired
 The fuse of the receiver blow
 The battery is dead
 The battery voltage is low

96. What protection should you wear when performing GMDSS equipment back up power battery
maintenance?
 You should wear eye protetion and cover all
 You should wear cover all and gloves
 You should wear face mask and gloves
 You should wear eye protection, gloves and apron

97. On a survival craft, you hear the SART’s Audible tone, what does it mean?
 It informs the survivors when the SART switches to ‘’standy mode’’
 It is is a sign when the battery charge condition is weak
 It informs the survivors thst a nearby vessel is signalling on DSC
 It is informing you that assitance may be nearby

98. You abandon your ship and boarded a survival craft. What should you do with your EPIRB and SART?
 Trow the EPIRB an SART in the water so that it will automaically activated
 Nothing o do EPIRB and SART is activated automatically when removed from te securing bracket
 Ensure that the EPIRB and SART is activated until help arrived
 Make reay the EPIRB and SART activate when another vessel is in sight

99. In proper setting up and safe operation radio equipment, what publication you should consult?
 FCC rules and regulation
 The manufacturer’s instruction manual
 Code of Federal Regulations
 ITU Manual

100. Your vessel losses Imarsat SES operation after a large course change. Which of the following could
cause this?
 Shadowing of SES antenna by clouds or other weather information
 Misalignment of the SES antenna and the LES
 Misalignment of the shadow corretion filter
 System loses gyro follow-up or a gyro system malfuction

101. What information should the message contain in GMDSS alert such fire onboard?
 The distress alert message should contain the nature of distress alert
 The distress alert message incude information about flag and port of registry
 The distress alert message should contain the name and position of the vessel
 Pressing the distress key on the equipment automatically send the information

102. What should you do if you test radio eqipment and it turns out to be defective?
 Consult the eqiupment manual and continue testing
 Stop testing and try testig next time on your duty
 Switch-off the equipment and re-start and test again
 The Master must be notified as soon as possible

103. If your vessel in on fire and requiring immediate assistance, what is the two letters signal you should
display?
 PM
 NC
 AL
 CD

104. As a storm hits your ship, causing you MF-HF whip antenna to shatter. What woul you do if you needed
to reclaim MF-HF frequencies?
 Rig a wire antenna approximately 35-75 feet long per the equipment instruction manual
 Rig a wire antenna 10-15 ft long from the antenna tuner to the highest vertical support
 Rig along wire antenna at least 200 feet long
 Rig a horizontal dipole antenna 40 feet long

105. If your ship is on fire, what priority signal will you use in sending your message by FAX?
 Distress
 Urgent
 Emergency
 Safety

106. Why must the SART be mounted as high as possible in the survival craft?
 To increase the range
 So that it will not get wet
 Because it increases the power
 To be seen better on the radar
107. What publication should GMDSS Operator reference for the correct GMDSS equipment set-up and
operation?
 The manufacturer’s instructon mannual
 NTC regulation Manual
 ITU publication Manual
 Part 90 of the FCC Rules and Regulation

108. When you are listening to the transmission of a very close station, what step you must observe to
prevent the strong signal from smothering the receiver?
 Switch off the speaker and use headset
 Adjust volume control
 Lower the power output
 Adjust the squelch

109. Which of the following is defined as back-up power source that provides power to the radio installations
for the purpose of conducting distress and safety communication when the vessel’s main emergency
generators failed?
 Reserve Source of Diesel Power (RSDP)
 Emergency Diesel Generator (EDG)
 Reserve Source of Energy (RSE)
 Emergency Back-up Generator (EBG)

110. Which statement is correct regarding the requirement for emergency and reserve power in GMDSS
radio installations?
 A reserve power source is not required for radio communications
 All newly constructed ships under GMDSS must have both emergency and reserve power sources
for radio communications
 Only one of the above is rrequied if a vessel is equipped with a second 406 EPIRB as a backup means of
sending a distress alert
 An emergecy poer source of radio communications is not required if a vessel ha proper reserve power
(batteries)

111. What is the digital automatic alerting system that you hear on shp VHF, MF and HF frequency bands
with information (position I.D. of Ship and type of distress) to the operator at CRS or RCC?
 Narrow Band Direct Printing
 NAVTEX
 Digital Selective Calling
 Automatic Identification System

112. On board a survival craft what GMDSS is used?


 Handheld radio
 EPIRB
 GPS
 NAVTEX

113. What is most likely the cause when you triggered SART and no active mode indication?
 Displaced magnet
 A faulty PCB
 A drift on PCB
 SART is stock in active mode

114. During transmission your radio become distorted what should you do?
 Use minimum modulation
 Cease the transmission
 Reduce tansmitter power reduce audio amplitude
 Reduce transmitter power

115. A fire occured onboard your ship and you need assistance in sending a DSC message, how will you
categorize the message?
 Distress
 Safety
 Routine
 Precautions

116. What can be the visual observation of a condition whereby it may be possible to initiate a reliable TELEX
transmission but a voice transmission may not be possible?
 An indication of a very high antenna elevation on the terminal display screen
 An idication thatthe antenna has reached its maximum travel in one direction
 An indication on a meter or on the display terminal of a high transmit power
 An indication on a meter or on the terminal of low receiver signal strength

117. what is the cause of an individual battery cell to reserve polarity?


 Insufficient charging which does not brig all of the cells up to full charge
 High discharge rate without allowing for a cool down period
 When charging wire connected in a wrong way. (the negative to the positive and the positive to the
negative)
 When discharging the battery string if a cell becomes weaker than the remaining cells, the
discharge current will effectively charge the weaker cell in reverse polarity.

118. What is the minimum transmitter power level for a medium frequency transmitter aboard a complsary
fitte vessel?
 The power atleast 100 watts single side band suppressed carrier power
 The power atleast 25 watts delivered into 50 ohmns when operated with aprimary voltage of 13.6 volts DC
 The power atleast 60 watts to communicate with the nearest public station operating a single sideband,
suppresse carrier power
 The power needed to communicate with the nearest public coast station operating on 2182 khz

119. Who is incharge of distress traffic during SAR operations when all participating ship are merhant
vessels?
 The first vessel that arrived on the scene
 Rescue Coordination Center (RCC)
 On-Scene Commader (OSC)
 Coordination Surface Search (CSS)

120. Risk of chemical burns is one of the two hazards associated with lead acid batteries that you should be
aware. What is the other hazard that you should give concern in dealing wih lead acid?
 Risk of fire
 Risk of suffocation
 Risk of poisoning
 Risk of explosion

121. What is the operational communcations conducted by radio other than distress?
 Security related communication
 Public correspondence communication
 General Communication
 Operational traffic communication

122. During abanon ship, which of the following types of EPIRBs is most likely to be used to transmit a
distress alert signal?
 S-band EPIRBs
 Class A EPIRBs
 406 MHz EPIRBs
 121.51/124.3 MHz EPIRBs

123. Your vessel is on fire on a radio traffic sea area, how will you initiate a call using VHF?
 Any STATIONS or ALL SHIPs spoken three times followed by the nature of your message?
 SECURITEE spoken three times followed by the nature of your message
 MAYDAY spoken three times and followed by the nature of your message
 PAN-PAN spoken three times followed by the nature of your message

124. A GMDSS Radio Operator license holder is prohibited to undertake the following servicing or
maintenance function?
 Replacement of the consumable items such as paper, ribbon, etc.
 Any adjustment or maintenance that may affect the proper operation of the station
 Routine battery maintenance if used as part of the GMDSS equipment
 Resest tripped circuit brake or replaced defective fuse

125. What occurs in ARQ when the information sending station (ISS) gets a signal indicating that the parity
check failed?
 The next block will be sent
 The data link will break
 The acknowledge light will illuminate
 The last block will be resent

126. IN case of power breakdown, what type of batteries most commoly used on board shipas as source of
backup power?
 Lead Battery
 Lithium Battery
 Nickel-iron (NiFe) battery
 Nickel-Cadmium (NiCd) battery

127. In the operation of 2182 kHz radiotelephone system, which of the following indicates a symptom of
mulfunction that need to be reported to the Master?
 Much lower noise level observed during daytime operation
 Failure to contact a shore station in 600 miles distance during daytime
 When testing alarm on 2182 kHz into an antenna, the distress frequency watch receiver becomes
unmuted
 No indication of power output when speaking into the microphone

128. What are the characteristics of the reserve source of energy un GMDSS?
 Can be independent of the following propelling power of the ship
 Supplies independent HF and MF installations at the same time
 Must be incorporated into the ship’s electrical system
 Must be independent of the ship’s elecrical system

129. What must be the provision for maintenance of a ship operating in sea area A1?
 Duplication of equipment
 Any one of the following
 At sea maintenance
 Shore maintenance

130. What type of initial external message you should send if it is necessary to communicate about the fire
onboard your vessel?
 Urgency message to MRCC and all ships
 Advice message to all ships and MRCC
 Alert messages to all ships
 Distress message to the MRCC and other ships

131. How do you prevent a powerful signal from ‘’smothering’’ your receiver while listening to a neigbhoring
station’s transmission?
 You should adjust the squelch
 You should adjust the volume
 Off the speaker and use headset
 You should lower the power output

132. What happens if you speak into the handheld transceiver’s microphone when the deviation adjustment
is set to high?
 It may cause digital interference to computer equipment
 It may cause interference to other stations operating nearby
 It may cause atmospheric interference in the air around the antenna
 It may cause interference to other stations on a higher frequency bands

133. In accordance with the International Convention Maritime Search and Rescue, who is responsible for
promoting the efficient organization of search and rescue service?
 Ship Eart Station (SES)
 On-Scene Commander (OSC)
 Coordination Surface Search (CSS)
 Rescue Coordination Center

134. What may form during the chemical process in battery charging?
 It may form highly explosive mixture if exposed to sunlight
 It may form highly explosive if mixed with water
 It may form highly explosive mixture if exposed to circuit
 It may form highly explosive mixture if exposed to chemical

135. What is the most likely caused of SART Red LED not turning on?
 Faulty PCB
 Drift on PCB
 SART in active mode
 Stock in active mode

136. What frequencies in GMDSS equipment you should use on an ‘’On Scene’’ communication between
vessel in distress and the SAR Aircraft?
 VHF Ch-16, 4125kHz, J3E, 3023 kHz, J3E
 VHF Ch-12, 4125kHz, JF1B, 5680 kHz, J3E
 VHF Ch-70, 4125kHz, F1B, 5680 kHz, J3E
 VHF Ch-70, 4125kHz, J3E, 5680 kHz, J3E

137. What is most likely the cause that SART do not indicate standby mode?
 The SART PBCS is drifting
 The SART switch ring magnet is displaced
 The SART is on active mode
 The SART battery is dead

138. When your equipment fails, how will you decide on the requirements for parts and repair materials for
the said GMDSS equipment?
 By IMO circular ‘’Equipment Spares’’
 The GMDSS maintainer requirements
 The recommendation of the manufacturer
 The recommendation of PSC inspector
139. Why is it necessary to send distress alert when you have fire onboard?
 The distress alert message is necessary so that other vessel can approach you and give assistance
 The distress alert message is not necessary if you doo not need immediate assistance
 The distress alert message is not necessary because your situation is still on uncertainty phase
 The distress alert message is necessary to alert other vessel and authorities

140. How often would you carry out a full specific gravity check of a lead acid battery?
 Monthly
 Weekly
 Annually
 Daily

141. While underway, how frequently is the DSC controller required to be tested?
 Once a month
 Twice a week
 Once a day
 Once a week

142. There are no signals or static heard in the receiver of the SBB Transceiver, although aloud noise is
heard if the volume control is advanced all he way up, whch of the following is ‘’NOT’’ the possible cause?
 The antenna of feed line is damaged
 A defective receiver
 The frequency is not properly set
 The antenna switch has been transfered to the test position

143. What’s the unit of measurement used to express the inductive reactance of a circuit?
 Mhos
 Henrys
 Farads
 Ohms

144. What is the main reason the radio transmission tend to be useful only for short-distance communication
during daylight hours?
 Because of auroral propagation
 Because of lack of activity
 Because of D region absorption
 Because of magnetic flux

145. What is the effect of the temperature to the specific gravity of lead acid batteries?
 A higher temperature results in a higher specific gravity
 The temperature has no effect on specific gravity reading
 A higher temperature results in lower specific gravity reading
 A lower temperature results in lower specific gravity reading

146.What would be an indication of a malfunction on a GMDSS equipment station with a 24 VDC battery
system?
 An indication on a meter or on the terminal of low receiver signal strength
 After a testing the station battery power the ammeter reading indicates a high rate of charge that then
declines
 A constant 30 volt reading on the GMDSS console
 After testing the battery power a voltmeter reading of 30 volts for brief period followed by 26 volt reading

147. Which of the following is a symptoms of a malfunction on a GMDSS station with a 24 VDC battery
system?
 The ammeter measurement indicates a high rate of charging that lowers after testing the station battery
power
 A voltmeter reading 30 volts for a brief duration followed by a constant 26 volt reading after testing the
station on battery power
 All symptoms indicate a potential battery charger malfunction
 A constant 30 volt reading on the GMDSS console volt meter

148. In case of power breakdown a reserve power source must be able to power all radio equipment plus as
emergency light system for how long?
 6 hours
 5 hours
 7 hours
 4 hours

149. What is the characteristics of the Reserve Source of Energy under GMDSS?
 Can be independent of the propelling power of the ship
 Must be independent of the Ship’s electrical system
 Must be incorporated into the ship’s electrical system
 Supplies independent HF and MF installations at the same time

150. In which Sea Area the radio coverage of VHF coast stations in which continuous alerting by Digital
Selective Calling (DSC) is available?
 Sea Area A2
 Sea Area A1
 In all the base area is available
 Sea Area A3

151. Which of the following frequencies is ‘’NOT’’ allocated in the Global Maritime Distress and Safety
System?
 415 kHz and 526.5 kHz
 1626.5 kHz (INMARSAT)
 406 MHz (EPIR)
 DSC on 156.52MHz (channel 70) and various MF/HF frequencies

152. What distress alert are based on the use of transmission via satellites (from a ship earth stationor
satellite EPIRB) and terrestial services (from ship stations and EPIRBs)?
 Ship-to-shore distress alert
 Shore-to-ship distress alert
 Ship-to-ship distress alert
 Shore-to-shore distress alert

153. Which system is most likely to be affected by atmospheric disturbances?


 VHF DSC
 Safety NET
 INMARSAT Standard-C
 MF/HF radiotelephony

154. What is the system that provides priority access to sattelite channel in an emergency situations and alert
to the LES personnel?
 Maritime Safety Information System
 Safety Net System
 Inmarsat System
 Enagnce Group Call System

155. What is the best way to minimize or prevent intrference to other stations?
 By using oinidirectional antenna pointed away from other station
 By reducing power to a level that will not affect other frequency communcation
 Determine the frequency is not use by monitoing the frequency before transmitting
 By changing frequency when notified that your radio communication interfer

156. When you received a distress alert at DSC 2187.5 kHz and your ship is capable to provide assistance,
how will you acknowledge the received MAYDAY?
 You should acknowledge at 2187.5 kHz and render assistance as necessary
 You should acknowledge at 2182 kHz and render assistance as necessary
 You should acknowledge at 6312.5 kHz and assist as necessary
 You should acknowledge at 407.5 kHz and assist as necessary

157. What is a distress communication?


 In radiotelegraphy SOS sent as a single chracter and in radio telephony spkeaking the word ‘’MAYDAY’’
 All message relative to the immediate assistance required by a ship, aircraft or other vehicle in
immediate danger
 Health and welfare messages concerning the immediat protection of property and safety of human life
 International recogized communications relating to emergency situation

158. What is usually the first step for GMDSS radio operator to take when initiating a distress priority
message, such as onboard via INMARSAT?
 Contacting the CES operator using the radiotelephone distress procedure Mayday …etc.
 Pressing a Distress key on the equipment
 Contacting the CES operator and announcing a distress condition is in existence
 Dialing the correct code on the telephone remote unit

159. You received a DSC fire alert on 8414.5 kHz in a Sea Area A3. Whatshould you do if no
acknowledgment from coast station within 3 minutes?
 Acknowledge the distress alert by DSC
 Acknowledge the distress alert by ang apppropriate GMDSS equipment
 Transmit a DSC distress relay alert
 No action to be done just proceed to your destination

160. Which statement is false regarding the maintenance of GMDSS equipment at sea?
 The GMDSS maintainer may not be the person designated to have primary responsibility for radio
communications during Distress incidents even if licensed as an operator
 ship must carry atleast one person who qualifies as GMDSS maintainer for the maintenance and repair of
equipment if at sea maintenance option is selected
 All the sea maintenenance and repairs must be performed by or under the supervision of a person holding
a GMDSS Maintainer License
 The GMDSS maintainer may b the person responsible for ensuring that the watches are properly
maintained

161. You noticed interference on one or two station while watching TV onboard, what is most likely cause of
interference?
 Harmonic transmission from your transmitter
 The TV receiver frot-end overboard
 De-ionization of the ionosphere near your area
 To much low pass filtering on the transmitter

162. What would you do if you notice a transmission problem during transmission?
 Switch-off the power supply of the equipment and re-start again
 The transmission should immediately be suspended
 Complete the transmission it will automatically stop if trouble persist
 Ignore the defect malfunction and continue with transmission
163. Your MF-HF whip antenna breaks off and is carried away by storm. What would you do to regain MF-HF
frequencies?
 Rig a wire antenna 10-15 ft long from the antenna tuner to the highest vertical support
 Rig a wire antenna approximately 35-75 log per the equipment instruction mannual
 Rig a horizontal center fed dipole antenna 40 feet long
 Rig a long wire antenna at least 100 feet long

164. What happen if your vessel is on fire and you begin a DSC distress transmission but do not include a
message?
 The transmission will not be initiated and “ERROR” will indicate on the display read out
 The transmission aborted and alarm sound will indicated that will advice you to provide informationThe t
 The transmission will be made “default” information provided automatically
 The receiving station will poll the DSC unit of your vessel to download the necessary information

165. What is the distress telegraphy message to be used when your vessel is threatened by a serious
imminent danger and is need of immediate assistance?
 PAN-PAN
 TTT
 SOS
 MAYDAY

166. Which is the best protecton must be worn when you are carrying out maintenance on batteries?
 Eye protection and cover all
 Gloves and cover all to protect the hand
 Glove and eye protector is enough
 Eye protection gloves and apron

167. You had abandon your vessel of distress situation, which of the following should be taken on board a
survival craft?
 Program the SART and EPIRB to transmit the vessel location and nature of distress
 Nothing to worry as the SART and EPIRB will both float free and operate automatically
 Place the SART and EPIRB in the “ON” position and secure them to the survival craft
 Throw the SART and EPIRB in the water to activate automatically

168. What must be the characteristics of the Rerserve Source of Energy under GMDSS?
 It must be independent of the ship’s electrical system
 It must have power a supply independent to HF and MF installations at the same time
 It must be independent of the propelling power of the ship
 It must be incorporated into the ship’s electrical system

169. Which of the following statements about maintainance requirement for GDMSS Equipment is FALSE?
 If shore based maintainance is employed, maintainance services are not required to be completed
or performance confirmed unless the vessel to foreign ports
 Compulsary vessel’s sailing in Sea Area A3 and A4 must provide two or three maintenance options which
are duplication of equipment, shore-based or at sea maintainance capability
 If compulary vessels operatig in Sea Areas A4 are required to carry at least three licensed GMDSS Radio
Operator
 Compulsary vessels sailing in Sea Area A1 and A2 must provide any one of the three maintainance
options which are duplication of equipment shore-base, or at sea maintainance capability

You might also like